Explained Mcqs in Cardiology

December 10, 2016 | Author: As'ad Bhana | Category: N/A
Share Embed Donate


Short Description

cardiology...

Description

www.apnastudent.blogspot.com Which of the following arrhythmias is unusual in digoxin toxicity? Non-paroxysmal atrial tachycardia with varying block First-degree heart block Type-II second-degree heart block

Correct answer

Wenkebach phenomenon

Your answer

Bidirectional ventricular tachycardia

Digoxin toxicity can result in any abnormal cardiac rhythm except type-I I second-degree atrioventricular (AV) block. The characteristic arrhythmia includes non-paroxysmal atrial tachycardia with varying block. The other common arrhythmias include first-degree heart block, Wenkebach (type-I second-degree block) and complete (third degree) heart block, ventricular ectopics and ventricular tachycardia (VT) (uni- and bi-directional). Ventricular fibrillation (VF) can also rarely occur. Another unusual arrhythmia is atrial flutter. Factors predisposing to digitalis toxicity are advanced age, hypoxia, hypokalaemia, hypomagnesaemia, hypercalcaemia, hypothyroidism, amyloidosis and renal failure. The most common precipitating factor is hypokalaemia. The earliest signs of digitalis toxicity include nausea, vomiting and anorexia. Chronic toxicity causes exacerbation of heart failure, weight loss, gynaecomastia and yellow vision.

A 28-year-old woman who is known to have a cardiac murmur becomes pregnant. I t is noted that the intensity of her murmur diminishes during her pregnancy. Which cardiac abnormality is she likely to have? Aortic stenosis Aortic regurgitation Tricuspid stenosis Pulmonary stenosis Fallot’s tetralogy A 28-year-old woman who is known to have a cardiac murmur becomes pregnant. I t is noted that the intensity of her murmur diminishes during her pregnancy. Which cardiac abnormality is she likely to have? Aortic stenosis Aortic regurgitation

Your answer

Tricuspid stenosis Pulmonary stenosis Fallot’s tetralogy

of 179

6/30/2011 4:56 PM

During pregnancy, cardiac output and blood volume increase from the second month up to the thirtieth week to 30–50% above the normal levels. The average increase in blood volume during pregnancy amounts to 1600 ml, and there is also an increased metabolic workload. This produces warm extremities, a tachycardia with a large-volume pulse and a slight rise in venous pressure. The diastolic blood pressure is lower due to vasodilatation, and this is responsible for the fading of the aortic regurgitation murmur. The apex beat is displaced, because of cardiomegaly and a raised diaphragm. The increased blood flow may produce a pulmonary systolic murmur and a third heart sound. All stenotic murmurs become more prominent.

A 63-year-old man with known chronic heart failure is admitted with symptoms at rest. Examination reveals pitting oedema to his knees, elevated jugular venous pressure and basal crepitations. He is in sinus rhythm at a rate of 80 bpm and his blood pressure is 100/60 mmHg. Current medication includes bisoprolol 10 mg once daily, frusemide 80 mg once daily and ramipril 2.5 mg twice daily. Blood tests reveal a sodium concentration of 133 mmol/l, potassium 4.9 mmol/l and creatinine of 169 µmol/l. The admitting doctor commences him on iv frusemide 80 mg twice daily and increases his ramipril to 5 mg twice daily. When you review him the following day what other drug would be most appropriate to add in? Amiloride 5 mg od Bendrofluazide 2.5 mg od Bumetanide 2 mg bd Metolazone 5 mg od Spironolactone 25 mg od

A 63-year-old man with known chronic heart failure is admitted with symptoms at rest. Examination reveals pitting oedema to his knees, elevated jugular venous pressure and basal crepitations. He is in sinus rhythm at a rate of 80 bpm and his blood pressure is 100/60 mmHg. Current medication includes bisoprolol 10 mg once daily, frusemide 80 mg once daily and ramipril 2.5 mg twice daily. Blood tests reveal a sodium concentration of 133 mmol/l, potassium 4.9 mmol/l and creatinine of 169 µmol/l. The admitting doctor commences him on iv frusemide 80 mg twice daily and increases his ramipril to 5 mg twice daily. When you review him the following day what other drug would be most appropriate to add in? Amiloride 5 mg od Bendrofluazide 2.5 mg od Bumetanide 2 mg bd Metolazone 5 mg od Spironolactone 25 mg od

Your answer

This man has decompensated CHF with symptoms at rest (New York Heart Association class I V). Examination has revealed significant fluid retention. The initial management plan of changing to iv frusemide is sensible since coexistent gut wall oedema is likely to impinge on oral absorption. Increasing the vasodilators in the form of ramipril is again a sensible approach. The addition of a thiazide (inhibiting sodium reabsorption in distal tubule) may work in synergy with a loop diuretic; the same is true for metolazone. Spironolactone, an aldosterone antagonist, has been shown to improve the mortality rate and symptoms and reduce hospitalisation in patients with severe CHF already on conventional treatment. Benefits are in addition to its diuretic effect since aldosterone itself has adverse effects on myocardial structure and function. Clearly, careful monitoring of renal function and biochemistry is important in such patients.

A 72-year-old Caucasian man is referred to out-patients with a 6-month history of progressive exertional dyspnoea. His ankles swell as the day progresses. There is no associated chest discomfort. He is an ex-smoker of 3 years and drinks 12 pints of beer per week. He has not seen his GP in the previous 15 years. The only past history is that of mild asthma as a child. His father died of a myocardial infarct aged 65 years. Blood pressure is 150/86 mmHg. Results of investigations are as follows: renal function, normal; cholesterol, 6.8 mmol/l; ECG, sinus rhythm LBBB; echo, dilated and impaired left ventricular function with ejection fraction of 30%, mild to moderate mitral regurgitation, no LVH. What is the most likely underlying aetiology? Alcohol

www.apnastudent.blogspot.com Coronary artery disease Hypertension Valvular heart disease Viral myocarditis A 72-year-old Caucasian man is referred to out-patients with a 6-month history of progressive exertional dyspnoea. His ankles swell as the day progresses. There is no associated chest discomfort. He is an ex-smoker of 3 years and drinks 12 pints of beer per week. He has not seen his GP in the previous 15 years. The only past history is that of mild asthma as a child. His father died of a myocardial infarct aged 65 years. Blood pressure is 150/86 mmHg. Results of investigations are as follows: renal function, normal; cholesterol, 6.8 mmol/l; ECG, sinus rhythm LBBB; echo, dilated and impaired left ventricular function with ejection fraction of 30%, mild to moderate mitral regurgitation, no LVH. What is the most likely underlying aetiology? Alcohol

Your answer

Coronary artery disease

Correct answer

Hypertension Valvular heart disease Viral myocarditis

It is important to remember that chronic heart failure is not an absolute diagnosis; there is a cause to be found. In the UK the commonest aetiology is underlying ischaemic heart disease (around 50–66% of all cases). This patient has several risk factors for ischaemic heart disease, which adds weight to the likelihood of this being the diagnosis. It is of course important to exclude other potentially reversible causes such as thyroid disease, ethanol, nutritional deficiencies, primary valvular pathology and uncontrolled prolonged tachycardia. The mitral regurgitation in this patient is most likely to be secondary to left ventricular dilatation and subsequent annular dilatation of the mitral valve ring. This will contribute to the development of pulmonary hypertension.

A 40-year-old woman presents with a 3-month history of fatigue, weight loss, night sweats and a degree of exertional dyspnoea. Her past history includes a prosthetic mitral valve replacement 2.5 years ago. She is pyrexial with evidence of mitral regurgitation and splinter haemorrhages. Echo confirms moderate paravalvular mitral regurgitation. Blood cultures are taken and a diagnosis of infective endocarditis made. What is the most likely infecting organism in this case? Coxiella burnetii Enterococcus spp Staphylococcus aureus Staphylococcus epidermidis Streptococcus viridans A 40-year-old woman presents with a 3-month history of fatigue, weight loss, night sweats and a degree of exertional dyspnoea. Her past history includes a prosthetic mitral valve replacement 2.5 years ago. She is pyrexial with evidence of mitral regurgitation and splinter haemorrhages. Echo confirms moderate paravalvular mitral regurgitation. Blood cultures are taken and a diagnosis of infective endocarditis made. What is the most likely infecting organism in this case? Coxiella burnetii Enterococcus spp Staphylococcus aureus Staphylococcus epidermidis Streptococcus viridans

of 179

Your answer

6/30/2011 4:56 PM

The commonest infective cause of native valve endocarditis in the UK is still Strep. viridans, accounting for around 40% of all cases. Enterococcus spp accounts for approximately 10% of cases and is more prevalent in the elderly. Staph. spp

www.apnastudent.blogspot.com account for around 25% of cases of endocarditis. In the first year following prosthetic valve replacement the spectrum of infecting organisms is somewhat different, with coagulase-negative staphylococci being the most common (around 50%). The majority of these are Staph. epidermidis. After the first year following valvular surgery the spectrum of infecting organisms is very similar to that for native valve endocarditis.

A 50-year-old man with long-standing hypertension presents acutely with severe chest pain radiating through to his back. He looks unwell, with a resting tachycardia (110 bpm) and blood pressure of 150/96 mmHg. There are no murmurs and neurological examination is normal. An urgent CT scan of his chest confirms type-A aortic dissection. The local cardiothoracic centre is contacted and urgent transfer arranged. He has received appropriate opiate analgesia. What additional drug treatment should be instigated as part of his immediate treatment plan? Intravenous GTN Intravenous labetalol Intravenous nitroprusside Oral amlodipine Oral enalapril A 50-year-old man with long-standing hypertension presents acutely with severe chest pain radiating through to his back. He looks unwell, with a resting tachycardia (110 bpm) and blood pressure of 150/96 mmHg. There are no murmurs and neurological examination is normal. An urgent CT scan of his chest confirms type-A aortic dissection. The local cardiothoracic centre is contacted and urgent transfer arranged. He has received appropriate opiate analgesia. What additional drug treatment should be instigated as part of his immediate treatment plan? Intravenous GTN Intravenous labetalol

Correct answer

Intravenous nitroprusside Oral amlodipine

Your answer

Oral enalapril

Type-A dissection involves the ascending aorta, and treatment with urgent surgical intervention is recommended unless severe co-morbidity would preclude the patient from surgery. I mmediate management includes the liberal use of opiates for complete pain relief, thereby decreasing the sympathetic drive. Meticulous blood pressure control is vital while awaiting surgery/transfer, in an attempt to reduce the chances of extension or rupture. Short-acting intravenous b-blockers (eg labetalol) are the first choice drugs as they reduce both blood pressure and force of ejection. A short half-life is important since haemodynamics can change rapidly. If b-blockers are contraindicated then sodium nitroprusside or calcium-channel blockers may be appropriate alternatives.

A 40-year-old woman presents with a 1-year history of increasing exertional dyspnoea and swollen ankles. Her GP has performed an ECG, which shows evidence of right ventricular hypertrophy and right axis deviation. Examination in clinic reveals the following: body mass index of 30, sinus rhythm 90 bpm, blood pressure 110/60 mmHg, elevated jugular venous pressure, left parasternal heave, peripheral oedema and a loud P2. There is no prior medical or family history of note. She is a non-smoker and drinks little alcohol. Whilst she denies current drug use, she has in the past used stimulant drugs purchased over the Internet to help her lose weight (previous body mass index of 34). What is the likely diagnosis? Isolated Cor pulmonale Chronic pulmonary emboli Primary pulmonary hypertension Mitral stenosis Tricuspid valve endocarditis

A 40-year-old woman presents with a 1-year history of increasing exertional dyspnoea and swollen ankles. Her GP has performed an ECG, which shows evidence of right ventricular hypertrophy and right axis deviation. Examination in clinic reveals the following: body mass index of 30, sinus rhythm 90 bpm, blood pressure 110/60 mmHg, elevated jugular venous pressure, left parasternal heave, peripheral oedema and a loud P2. There is no prior medical or family history of note. She is a non-smoker and drinks little alcohol. Whilst she denies current drug use, she has in the past used stimulant drugs

of 179

6/30/2011 4:56 PM

purchased over the Internet to help her lose weight (previous body mass index of 34). What is the likely diagnosis?

www.apnastudent.blogspot.com Isolated Cor pulmonale Chronic pulmonary emboli Primary pulmonary hypertension

Your answer

Mitral stenosis Tricuspid valve endocarditis

This woman has clinical features of pulmonary hypertension. The onset of symptoms (eg dyspnoea, syncope, chest pain) is usually insidious. Other clinical findings may include murmurs of tricuspid or pulmonary regurgitation, ascites and hepatomegaly. It is important to exclude a secondary cause such as chronic pulmonary emboli, underlying collagen vascular disease, left-sided heart disease and chronic obstructive pulmonary disease. Primary pulmonary hypertension is diagnosed by exclusion and is around three times more common in females. The aetiology is uncertain but a genetic component with additional triggering factors seems likely. There appears to be a clear-cut relationship between the use of appetite suppressants (fenfluramine) and the development of primary pulmonary hypertension. This would fit in with the clinical scenario described above.

Cardiac catheterisation is performed on a 25-year-old man with a systolic murmur but no symptoms. ECG and chest X-ray are normal. The findings are as follows (pressures mmHg): aorta, 125/70; left ventricle, 120/12; right atrium, mean 8; right ventricle, 40/8; pulmonary artery, 44/14; pulmonary capillary wedge, mean 13. Saturations (%): aorta, 97; superior vena cava, 68; right atrium, 70; right ventricle, 82; pulmonary artery, 85. What is the most likely cardiac diagnosis? ASD – primum ASD – secundum Mitral stenosis Primary pulmonary hypertension Ventricular septal defect

Cardiac catheterisation is performed on a 25-year-old man with a systolic murmur but no symptoms. ECG and chest X-ray are normal. The findings are as follows (pressures mmHg): aorta, 125/70; left ventricle, 120/12; right atrium, mean 8; right ventricle, 40/8; pulmonary artery, 44/14; pulmonary capillary wedge, mean 13. Saturations (%): aorta, 97; superior vena cava, 68; right atrium, 70; right ventricle, 82; pulmonary artery, 85. What is the most likely cardiac diagnosis? ASD – primum ASD – secundum

Your answer

Mitral stenosis Primary pulmonary hypertension Ventricular septal defect

Correct answer

The catheter data demonstrate a step up in saturations from the right atrium to the right ventricle, in keeping with a left to right shunt at the level of the ventricle. There is a mild elevation in pulmonary artery pressure in keeping with the shunt. VSD is the commonest form of congenital heart disease. In adults a small defect may present as an asymptomatic murmur; or in the extreme as Eisenmenger’s syndrome, where reversal of a left to right shunt has occurred as a consequence of advanced pulmonary hypertension. VSD is associated with an increased risk of endocarditis although recent NICE guidelines do not say that routine antibiotic prophylaxis is recommended. Management depends upon the actual size of the shunt.

A 65-year-old man presents to casualty with severe chest pain. ECG shows anterior ST-segment elevation and he receives prompt thrombolysis with reteplase with good resolution of changes. He is commenced on aspirin, a

of 179

β-blocker,

an ACE

6/30/2011 4:56 PM

inhibitor and a statin. His initial progress is complicated by further pain, worse with inspiration and movement and relieved by non-steroidal drugs. You are called to see him on day 5 postinfarct when he complains of shortness of breath on walking to the bathroom. He looks unwell with a cool periphery and resting tachycardia. Blood pressure is reduced at 90/50 mmHg. Jugular venous pressure is elevated to around 8 cm and rises with inspiration. His ECG shows preserved anterior R waves and anterolateral T-wave inversion together with sinus tachycardia. Chest X-ray shows an increase in the cardiothoracic

www.apnastudent.blogspot.com ratio but clear lung fields. What is the most likely complication to have developed to account for this deterioration? Cardiogenic shock Mitral regurgitation Pericardial tamponade Pulmonary embolism Ventricular septal defect A 65-year-old man presents to casualty with severe chest pain. ECG shows anterior ST-segment elevation and he receives prompt thrombolysis with reteplase with good resolution of changes. He is commenced on aspirin, a β-blocker, an ACE inhibitor and a statin. His initial progress is complicated by further pain, worse with inspiration and movement and relieved by non-steroidal drugs. You are called to see him on day 5 postinfarct when he complains of shortness of breath on walking to the bathroom. He looks unwell with a cool periphery and resting tachycardia. Blood pressure is reduced at 90/50 mmHg. Jugular venous pressure is elevated to around 8 cm and rises with inspiration. His ECG shows preserved anterior R waves and anterolateral T-wave inversion together with sinus tachycardia. Chest X-ray shows an increase in the cardiothoracic ratio but clear lung fields. What is the most likely complication to have developed to account for this deterioration? Cardiogenic shock Mitral regurgitation Pericardial tamponade

Your answer

Pulmonary embolism Ventricular septal defect

Devastating complications still do occur following acute myocardial infarction. Cardiogenic shock tends to occur early following a large infarct (or in the presence of already impaired left ventricular function), typically in the first 24–48 hours. The development of acute mitral regurgitation or VSD is associated with severe pulmonary oedema. While they would give rise to an elevation of jugular venous pressure, Kussmaul’s sign would not be present (increase in JVP with inspiration). Although pericarditis is common following an MI , and in particular anterior MI, tamponade occurs relatively infrequently. Chest X-ray shows a large heart with normal pulmonary vasculature. Echocardiography is vital to assist in the management of such a patient and permits easy differentiation of the possible causes of haemodynamic collapse.

A 54-year-old man is referred with increased swelling of his ankles and abdomen, and a degree of shortness of breath on exertion. His jugular venous pressure is elevated with prominent x- and y-descents. Apex beat is normal. ECG shows atrial fibrillation with widespread non-specific ST-segment abnormalities. Echo reveals preserved left ventricular systolic function with biatrial enlargement and an estimated pulmonary artery systolic pressure of around 60 mmHg. Chest X-ray shows atrial enlargement but no other abnormalities. What is the most likely cardiac diagnosis? Chronic pulmonary emboli Dilated cardiomyopathy Restrictive cardiomyopathy Secundum ASD Tricuspid regurgitation A 54-year-old man is referred with increased swelling of his ankles and abdomen, and a degree of shortness of breath on exertion. His jugular venous pressure is elevated with prominent x- and y-descents. Apex beat is normal. ECG shows atrial fibrillation with widespread non-specific ST-segment abnormalities. Echo reveals preserved left ventricular systolic function with biatrial enlargement and an estimated pulmonary artery systolic pressure of around 60 mmHg. Chest X-ray shows atrial enlargement but no other abnormalities. What is the most likely cardiac diagnosis? Chronic pulmonary emboli

of 179

6/30/2011 4:56 PM

Dilated cardiomyopathy Restrictive cardiomyopathy

Correct answer

www.apnastudent.blogspot.com Secundum ASD

Your answer

Tricuspid regurgitation

Restrictive cardiomyopathy results from fibrosis or infiltration of the endo- or myocardium. The result is failure of the ventricles to relax, with a subsequent increase in ventricular end-diastolic pressures leading on to biatrial enlargement. Systolic function is normal. Underlying causes include amyloidosis, storage disorders, sarcoidosis, haemochromatosis and endomyocardial fibrosis. Symptoms are usually those of predominant right heart failure and atrial fibrillation is common. The ECG may be normal but diffuse ST-segment and T-wave changes are commonly seen. Diuretics are the mainstay of symptomatic treatment. I t can be very difficult to differentiate restrictive cardiomyopathy from constrictive pericarditis. In restriction the pulmonary artery systolic pressure is usually elevated to > 45 mmHg, while it is lower in constriction. Right and left heart catheter may aid differentiation.

A 70-year-old woman is admitted to hospital with a swollen left leg 4 weeks after undergoing an elective total hip replacement. An above-knee DVT is diagnosed by ultrasound. She is in sinus rhythm at 60 bpm and her blood pressure is 160/80 mmHg. She is commenced on the appropriate dose of low molecular weight heparin and warfarin loading. The following day she becomes acutely short of breath. Examination reveals a resting tachycardia (110 bpm) with blood pressure of 100/60 mmHg. Her JVP is elevated at 7 cm above the sternal notch. Arterial blood gas measurement reveals her to be hypoxaemic with a p a(O2 ) of 7 mmHg. What would be the first-line therapy after administering high-flow oxygen? Aspirin Intravenous heparin Surgical embolectomy Thrombolysis with reteplase Vena caval filter A 70-year-old woman is admitted to hospital with a swollen left leg 4 weeks after undergoing an elective total hip replacement. An above-knee DVT is diagnosed by ultrasound. She is in sinus rhythm at 60 bpm and her blood pressure is 160/80 mmHg. She is commenced on the appropriate dose of low molecular weight heparin and warfarin loading. The following day she becomes acutely short of breath. Examination reveals a resting tachycardia (110 bpm) with blood pressure of 100/60 mmHg. Her JVP is elevated at 7 cm above the sternal notch. Arterial blood gas measurement reveals her to be hypoxaemic with a p a(O2 ) of 7 mmHg. What would be the first-line therapy after administering high-flow oxygen? Aspirin Intravenous heparin Surgical embolectomy Thrombolysis with reteplase

Your answer

Vena caval filter

This patient has clinical features of a massive pulmonary embolus. This results from significant obstruction of the pulmonary arteries causing haemodynamic compromise – namely shock or systemic hypotension (systolic blood pressure < 90 mmHg or a drop of > 40 mmHg for > 15 minutes). The initial treatment of choice is thrombolysis using a recognised protocol. Whilst she is only 4 weeks out from her hip replacement, the benefits fo thrombolysis would outweigh the risks in this case. I notropic support and the judicious use of fluids may also be required in the interim. Subsequent intravenous unfractionated heparin should then be commenced.

A 70-year-old woman with long-standing hypertension is referred to out-patients with a diagnosis of asymptomatic atrial fibrillation. Echocardiography demonstrates normal left ventricular function, mild LVH and normal mitral valve structure. The left atrium is slightly enlarged (4.2 cm). She is not keen on cardioversion and her rate is well controlled at 70 bpm. What would be the optimal strategy for long-term anticoagulation? Aspirin

of 179

6/30/2011 4:56 PM

Clopidogrel

www.apnastudent.blogspot.com Dipyridamole Low molecular weight heparin Warfarin A 70-year-old woman with long-standing hypertension is referred to out-patients with a diagnosis of asymptomatic atrial fibrillation. Echocardiography demonstrates normal left ventricular function, mild LVH and normal mitral valve structure. The left atrium is slightly enlarged (4.2 cm). She is not keen on cardioversion and her rate is well controlled at 70 bpm. What would be the optimal strategy for long-term anticoagulation? Aspirin Clopidogrel Dipyridamole Low molecular weight heparin Warfarin

Your answer

Atrial fibrillation (AF) is common and affects around 2–5% of the population who are over 60 years old. It confers an approximately fivefold increased risk of stroke. The absolute risk of stroke is related to the coexistence of other cardiovascular disease. In patients with AF and additional risk factors for stroke, such as hypertension, warfarin has been shown to be superior to antiplatelet therapy (primarily aspirin). This patient has evidence of structural cardiac disease with LVH and an enlarged left atrium, thereby reflecting a higher risk of developing a thromboembolic complication. A Cochrane systemic review has concluded clear superiority of warfarin over anti-platelet agents in non-valvular AF; warfarin could be expected to prevent 30 strokes at the expense of 6 major bleeding events. Aspirin had some benefit in stroke prevention but was clearly not as efficacious in patients at high risk of stroke. The use of low-dose warfarin or low-dose warfarin combined with aspirin was found to be of little benefit for stroke prevention. Newer agents such as low molecular weight heparin appear to be beneficial and require further study.

A 40-year-old woman is admitted with a stroke after a prolonged pyrexial illness. On examination she is in sinus rhythm, has splenomegaly and a pansystolic murmur at the apex. Blood cultures confirm an infective endocarditis. Which of the following is the most common causative organism? Streptococcus viridans Staphylococcus aureus Streptococcus bovis Gram-negative bacilli Staphylococcus epidermidis A 40-year-old woman is admitted with a stroke after a prolonged pyrexial illness. On examination she is in sinus rhythm, has splenomegaly and a pansystolic murmur at the apex. Blood cultures confirm an infective endocarditis. Which of the following is the most common causative organism? Streptococcus viridans

Your answer

Staphylococcus aureus Streptococcus bovis Gram-negative bacilli Staphylococcus epidermidis

Infective endocarditis on native valves – prevalence of organisms: Streptococci

of 179

Viridans group

30–40%

Enterococci

10–15%

Other

20–25%

6/30/2011 4:56 PM

Staphylococci

Staphylococcus aureus

9–27%

www.apnastudent.blogspot.com Coagulase-negative

1–3%

Gram-negative bacilli

Haemophilus spp.

3–8%

Anaerobes

Rickettsia/fungi

less than 2%

Members of the viridans group of streptococci are the commonest cause of subacute endocarditis on native valves. These commensals of the upper respiratory tract may enter the bloodstream on chewing, tooth brushing or at the time of dental treatment. Staphylococcus aureus is a common cause of acute endocarditis originating from skin infections, abscesses, vascular access sites or intravenous drug misuse. Staphylococcus epidermidis is the most common organism causing postoperative endocarditis following cardiac surgery.

A 30-year-old woman is routinely seen by her GP 24 weeks into her first pregnancy. She is well without adverse symptoms. Her blood pressure is 150/96 mmHg. Her baseline blood pressure at booking was 136/84 mmHg. No other abnormalities are found. What drug therapy would you prescribe? Bendrofluazide Moxonidine Labetalol Losartan Ramipril A 30-year-old woman is routinely seen by her GP 24 weeks into her first pregnancy. She is well without adverse symptoms. Her blood pressure is 150/96 mmHg. Her baseline blood pressure at booking was 136/84 mmHg. No other abnormalities are found. What drug therapy would you prescribe? Bendrofluazide Moxonidine Labetalol

Your answer

Losartan Ramipril

Hypertension in pregnancy is defined as a blood pressure > 140/90 mmHg or a rise of 25 mmHg of systolic and/or 15 mmHg of diastolic pressure above baseline. I t is seen in around 10% of all pregnancies. Gestational hypertension is more common than pre-eclampsia, which is associated with maternal organ dysfunction. While the latter is treated by delivery, drug therapy is often required to treat gestational hypertension. First-line agents, proven to be safe in pregnancy, include labetalol and methyldopa. Second-line agents include nifedipine, hydralazine and prazosin. Thiazides, ACE inhibitors and angiotensin receptor blockers are not recommended for use in pregnancy due to the risk of adverse effects on the developing fetus.

A 65-year-old man is referred to out-patients with resistant hypertension. He is already taking bendrofluazide 2.5 mg once daily, lisinopril 20 mg once daily and amlodipine 10 mg once daily. He is an ex-smoker with a past history of uncomplicated myocardial infarction. Blood pressure is 170/100 mmHg in both arms. The only other abnormality on examination is a left femoral bruit. Results of investigations are as follows: LVH on ECG; creatinine, 165 µmol/l ; sodium, 138 mmol/l; potassium, 5.2 mmol/l; chest X-ray, normal; 24-hour blood pressure, sustained systolic and diastolic hypertension with no evidence of nocturnal dip. What is the most likely underlying aetiology for his hypertension? Coarctation Conn’s syndrome Cushing’s syndrome Polycystic kidney disease Renal artery stenosis

of 179

6/30/2011 4:56 PM

A 65-year-old man is referred to out-patients with resistant hypertension. He is already taking bendrofluazide 2.5 mg once

www.apnastudent.blogspot.com daily, lisinopril 20 mg once daily and amlodipine 10 mg once daily. He is an ex-smoker with a past history of uncomplicated myocardial infarction. Blood pressure is 170/100 mmHg in both arms. The only other abnormality on examination is a left femoral bruit. Results of investigations are as follows: LVH on ECG; creatinine, 165 µmol/l ; sodium, 138 mmol/l; potassium, 5.2 mmol/l; chest X-ray, normal; 24-hour blood pressure, sustained systolic and diastolic hypertension with no evidence of nocturnal dip. What is the most likely underlying aetiology for his hypertension? Coarctation Conn’s syndrome Cushing’s syndrome Polycystic kidney disease Renal artery stenosis

Your answer

A secondary cause for hypertension is more likely in patients with resistant hypertension and in those who fail to show a nocturnal dip (usually an approximately 20% drop). Other clinical and investigative findings may raise the index of suspicion. In this case, renovascular disease should be suspected since he has documented evidence of co-morbid vascular disease and arterial bruit. A discrepancy in renal size on ultrasound would add further weight to the diagnosis. Further imaging, such as angiography or magnetic resonance angiography, should be considered in patients with a high index of suspicion for renovascular disease.

A 70-year-old man is referred by his GP for advice regarding optimisation of secondary prevention. He has a history of angina, with excellent control of symptoms on a combination of aspirin, dipyridamole MR, atenolol 50 mg od, simvastatin 40 mg od and isosorbide mononitrate 20 mg bd. His pulse rate is 70 bpm and blood pressure is 144/86 mmHg. The only other relevant past history includes an ischaemic stroke 2 years ago from which he made a complete recovery. What additional therapy would you consider adding? Bendroflumethiazide Diltiazem Doxazosin Nicorandil Perindopril A 70-year-old man is referred by his GP for advice regarding optimisation of secondary prevention. He has a history of angina, with excellent control of symptoms on a combination of aspirin, dipyridamole MR, atenolol 50 mg od, simvastatin 40 mg od and isosorbide mononitrate 20 mg bd. His pulse rate is 70 bpm and blood pressure is 144/86 mmHg. The only other relevant past history includes an ischaemic stroke 2 years ago from which he made a complete recovery. What additional therapy would you consider adding? Bendroflumethiazide

Your answer

Diltiazem Doxazosin Nicorandil Perindopril

Correct answer

Most clinicians would now recommend the addition of ACE inhibitors for patients with vascular disease, irrespective of left ventricular function. This is based on evidence from large trials, such as PROGRESS (perindopril) and HOPE (ramipril). Favourable outcomes were found to be independent of a blood pressure effect. As such, the benefit of ACE inhibition seems to be not purely related to a reduction in blood pressure; beneficial local vascular and myocardial effects are also seen. Blood pressure is not yet optimised in this patient and further antihypertensive therapy is warranted. The addition of an ACE inhibitor should bring this to the desired level (< 140/85 mmHg).

An 81-year-old woman is referred to cardiology out-patients with a history of dizzy episodes and one episode of syncope.

of 179

6/30/2011 4:56 PM

She is known to have long-standing atrial fibrillation. A 24-hour tape confirms atrial fibrillation, with rates varying from 30

www.apnastudent.blogspot.com to 140 bpm. There are several daytime pauses of over 3 seconds. She is listed for a permanent pacemaker. Which of the following would be the most appropriate device? DDD DDI

VOO VVI VDD An 81-year-old woman is referred to cardiology out-patients with a history of dizzy episodes and one episode of syncope. She is known to have long-standing atrial fibrillation. A 24-hour tape confirms atrial fibrillation, with rates varying from 30 to 140 bpm. There are several daytime pauses of over 3 seconds. She is listed for a permanent pacemaker. Which of the following would be the most appropriate device? DDD

Your answer

DDI VOO VVI

Correct answer

VDD

Pacemaker types are defined by an international code, which uses three or more letters in a set sequence. The first letter is related to the chamber that is to be paced: A = atrium, V = ventricle, D = both. The second letter refers to the chamber that is sensed (A, V or D). The third letter refers to the response to a sensed beat by the pacemaker: I = inhibits, T = trigger or D = both (ie either inhibits or triggers). VOO is a fixed output setting (eg pacing at 60 beats per minute irrespective of intrinsic activity); the ‘O’ as second and third symbol implies that the chamber is not sensed and therefore there is no response to a sensed beat. A fourth letter refers to whether or not the pacemaker has rate-adaptive properties (R). This woman has atrial fibrillation and as such there is no need for an atrial lead. VVI means there is one lead in the ventricle (pacing and sensing the ventricle). If the pacemaker senses an intrinsic QRS complex then pacing is inhibited (I). This means the pacemaker will only deliver a stimulus when there is a significant pause.

An 18-year-old young man presents to A&E having developed palpitations while playing football. ECG shows rapid atrial fibrillation with a ventricular rate of around 250 bpm. QRS duration is prolonged at around 130 ms. DC cardioversion is performed. Subsequent ECG in sinus rhythm demonstrates a PR interval of 100 ms, positive R wave in V1 and the presence of a delta wave. What further treatment would you recommend? Atrial defibrillator implantation Intravenous and then oral loading with amiodarone Radiofrequency ablation of the accessory pathway Radiofrequency ablation of the AV node Surgical ablation of the accessory pathway An 18-year-old young man presents to A&E having developed palpitations while playing football. ECG shows rapid atrial fibrillation with a ventricular rate of around 250 bpm. QRS duration is prolonged at around 130 ms. DC cardioversion is performed. Subsequent ECG in sinus rhythm demonstrates a PR interval of 100 ms, positive R wave in V1 and the presence of a delta wave. What further treatment would you recommend? Atrial defibrillator implantation Intravenous and then oral loading with amiodarone

Your answer

Radiofrequency ablation of the accessory pathway

Correct answer

Radiofrequency ablation of the AV node

of 179

6/30/2011 4:56 PM

Surgical ablation of the accessory pathway

www.apnastudent.blogspot.com

This young man has Wolff–Parkinson–White (WPW) syndrome. The most common arrhythmia is an atrioventricular re-entry tachycardia (AVRT). This is a narrow complex with anterograde conduction through the AV node and retrograde conduction via the accessory pathway. Patients who develop AF are at risk of rapid anterograde conduction to the ventricles via the accessory pathway, and this may subsequently degenerate to VF. The extremely rapid conduction with broad QRS duration is typical of this complication. Radiofrequency ablation of the accessory pathway is recommended in this setting and is potentially curative.

A 70-year-old man is brought into A&E. He is unwell with a cool periphery and blood pressure of 70/40 mmHg. ECG shows a regular broad-complex tachycardia with rate of 150 bpm. He is unable to provide a clear history, but a recent prescription in his wallet shows that he is taking aspirin, ramipril, frusemide and spironolactone. What is the likely arrhythmia? Atrial fibrillation Atrial flutter with a 2:1 block SVT with aberrant conduction VT Wolff–Parkinson–White syndrome A 70-year-old man is brought into A&E. He is unwell with a cool periphery and blood pressure of 70/40 mmHg. ECG shows a regular broad-complex tachycardia with rate of 150 bpm. He is unable to provide a clear history, but a recent prescription in his wallet shows that he is taking aspirin, ramipril, frusemide and spironolactone. What is the likely arrhythmia? Atrial fibrillation Atrial flutter with a 2:1 block SVT with aberrant conduction VT

Your answer

Wolff–Parkinson–White syndrome

This patient is haemodynamically compromised as a consequence of the arrhythmia. The prescription would suggest the presence of underlying cardiac disease with probable cardiac dysfunction. In the context of known ischaemic heart disease or left ventricular dysfunction, a broad-complex tachycardia should be assumed to be VT until proved otherwise. Many ECG criteria exist to aid the differentiation of VT and SVT with aberrancy and include: A–V dissociation; capture beats; fusion beats; extreme QRS axis; concordance across ventricular leads. A previous ECG can be exceedingly helpful. Subtle clinical signs, such as intermittent cannon waves in the JVP and a variable first heart sound may be present, in patients with A–V dissociation.

A 68-year-old man is admitted with syncope. He is known to have ischaemic cardiomyopathy. His medications include: aspirin 75 mg od, frusemide 80 mg bd and lisinopril 10 mg od. An initial ECG shows sinus bradycardia (50 bpm) and RBBB. Results of blood tests are as follows: sodium, 134 mmol/l; potassium, 3.5 mmol/l; creatinine 124 µmol/l. He has recurrent syncopal episodes on the CCU, where monitoring shows episodes of non-sustained torsades de pointes (polymorphic VT). Which of the following would be your initial line of treatment? DC cardioversion Intravenous amiodarone Intravenous magnesium Oral metoprolol Temporary pacing A 68-year-old man is admitted with syncope. He is known to have ischaemic cardiomyopathy. His medications include: aspirin 75 mg od, frusemide 80 mg bd and lisinopril 10 mg od. An initial ECG shows sinus bradycardia (50 bpm) and RBBB.

of 179

6/30/2011 4:56 PM

Results of blood tests are as follows: sodium, 134 mmol/l; potassium, 3.5 mmol/l; creatinine 124 µmol/l. He has recurrent syncopal episodes on the CCU, where monitoring shows episodes of non-sustained torsades de pointes (polymorphic VT).

www.apnastudent.blogspot.com Which of the following would be your initial line of treatment? DC cardioversion Intravenous amiodarone Intravenous magnesium

Correct answer

Oral metoprolol Temporary pacing

Your answer

Torsades de pointes (polymorphic VT with QRS complexes of different amplitude twisting around isoelectric line) occurs in patients with a prolonged QT interval. Any cause of QT prolongation can predispose to the arrhythmia. These include: congenital (the Jervell–Lange-Neilsen or Romano–Ward syndromes); with QRS complexes of different amplitude, metabolic (hypo-calcaemia, -magnesaemia or –kalaemia); drugs (eg amiodarone, tricyclic antidepressants, phenothiazines); ischaemic heart disease; mitral valve prolapse. The arrhythmia often occurs in the context of bradycardia. The key here is that amiodarone may exacerbate the situation. Intravenous magnesium (even if the serum magnesium concentration is normal) is the first-line therapy. Temporary pacing at higher rates with or without β-blockers is the next line of therapy. DC shock would not be helpful since episodes are non-sustained.

A 65-year-old man is admitted via A&E with acute shortness of breath. His past medical history includes an anterior MI 5 years ago. He is usually short of breath after walking around 400 m (0.25 mile), but is not on regular treatment. Clinically he is distressed: respiratory rate 30/min, basal crepitations to mid-zones, saturations 90%, pulse rate 110 sinus, blood pressure 180/100 mm Hg. The casualty officer has already given iv diamorphine 5 mg, iv metoclopramide and iv frusemide 40 mg (twice) but the patient remains short of breath, although saturations have increased to 94% with high-flow oxygen. CXR confirms pulmonary oedema. What further intravenous therapy would you commence? Atenolol iv Dobutamine iv Dopamine iv GTN iv Milrinone iv A 65-year-old man is admitted via A&E with acute shortness of breath. His past medical history includes an anterior MI 5 years ago. He is usually short of breath after walking around 400 m (0.25 mile), but is not on regular treatment. Clinically he is distressed: respiratory rate 30/min, basal crepitations to mid-zones, saturations 90%, pulse rate 110 sinus, blood pressure 180/100 mm Hg. The casualty officer has already given iv diamorphine 5 mg, iv metoclopramide and iv frusemide 40 mg (twice) but the patient remains short of breath, although saturations have increased to 94% with high-flow oxygen. CXR confirms pulmonary oedema. What further intravenous therapy would you commence? Atenolol iv Dobutamine iv

Your answer

Dopamine iv GTN iv

Correct answer

Milrinone iv

This patient has severe pulmonary oedema. Initial treatment includes sitting the patient up and administering high-flow oxygen. High catecholamine levels and activation of the renin–angiotensin–aldosterone systems drive the peripheral vasoconstriction, which increases myocardial oxygen demand. Traditionally, treatment has been with diuretics, but the benefits probably relate to their vasodilatatory actions. Early use of more powerful vasodilators represents a more attractive strategy and includes iv diamorphine and iv GTN (short half-life). Early administration of an oral ACE inhibitor would also be recommended. Inotropic agents should be considered in the context of cardiogenic shock (low blood pressure with impaired tissue perfusion).

of 179

6/30/2011 4:56 PM

A 78-year-old woman presents to A&E with three episodes of syncope in the last 24 hours. There is no history of chest pain.

www.apnastudent.blogspot.com She is taking frusemide 80 mg od and ramipril 10 mg od for known hypertension. She is conscious with a blood pressure of 100/40 mmHg. Potassium is 5.3 mmol/l. Her ECG shows complete heart block with rate of 40 bpm. QRS duration is 150 ms with a right bundle-branch block configuration. What is the optimum initial management? Dobutamine Isoprenaline Intravenous calcium chloride Temporary transvenous pacing Withhold medication and observe A 78-year-old woman presents to A&E with three episodes of syncope in the last 24 hours. There is no history of chest pain. She is taking frusemide 80 mg od and ramipril 10 mg od for known hypertension. She is conscious with a blood pressure of 100/40 mmHg. Potassium is 5.3 mmol/l. Her ECG shows complete heart block with rate of 40 bpm. QRS duration is 150 ms with a right bundle-branch block configuration. What is the optimum initial management? Dobutamine Isoprenaline Intravenous calcium chloride Your answer Temporary transvenous pacing Correct answer Withhold medication and observe

This woman has complete heart block with an unstable escape rhythm. The latter is exemplified by the fact that she has already had three syncopal episodes. Her QRS duration is prolonged (normal up to 120 ms), and this is generally more unstable than an escape rhythm of normal duration (ie < 120 ms) since this originates from around the His bundle. Her blood pressure is low, particularly with a background of hypertension. In addition, it is important to remember that cardiac output will be influenced by heart rate. In the elderly, cerebral vascular dysregulation may compound the effect thereby contributing to cerebral hypoperfusion. Ideally she should receive a transvenous temporary pacemaker. If further acute problems occur while waiting for a transvenous temporary pacemaker (eg awaiting transfer to a room with fluoroscopy), then external pacing can be instituted in the short term with appropriate sedation.

A 50-year-old man presents with a 1-hour history of severe central chest pain. There is no significant past medical history. He is haemodynamically stable with pulse rate of 90 bpm and blood pressure of 120/70 mm Hg. ECG shows 5 mm of ST-segment elevation in the anterior leads (V2–V4). He received aspirin 300 mg in the ambulance and 5 mg diamorphine. What would be the next line of treatment? Clopidogrel 75mg Enoxaparin GII b/II a blocker Percutaneous coronary intervention Tissue plasminogen activator A 50-year-old man presents with a 1-hour history of severe central chest pain. There is no significant past medical history. He is haemodynamically stable with pulse rate of 90 bpm and blood pressure of 120/70 mm Hg. ECG shows 5 mm of ST-segment elevation in the anterior leads (V2–V4). He received aspirin 300 mg in the ambulance and 5 mg diamorphine. What would be the next line of treatment? Clopidogrel 75mg Enoxaparin GII b/II a blocker

of 179

6/30/2011 4:56 PM

Percutaneous coronary intervention Tissue plasminogen activator

Your answer

www.apnastudent.blogspot.com

This relatively young man has presented early with acute anterior myocardial infarction. The key therapeutic aim is early reperfusion in an attempt to save myocardium. In centres with rapid access to primary angioplasty this would be the optimum strategy and guidelines now suggest this should be the norm in the UK. I f angioplasty is not available then thrombolysis with TPA is the next best alternative.

A 38-year-old woman is seen in A&E with a history of collapse. She recalls rushing for the bus before feeling faint. Her brother recently died suddenly due to a heart problem. On examination she has a ‘jerky’ pulse, a thrusting cardiac impulse and a mid-systolic murmur. What is the likely diagnosis? Dilated cardiomyopathy Hypertrophic cardiomyopathy Mitral valve prolapse Aortic stenosis Pericarditis

A 38-year-old woman is seen in A&E with a history of collapse. She recalls rushing for the bus before feeling faint. Her brother recently died suddenly due to a heart problem. On examination she has a ‘jerky’ pulse, a thrusting cardiac impulse and a mid-systolic murmur. What is the likely diagnosis? Dilated cardiomyopathy Hypertrophic cardiomyopathy

Your answer

Mitral valve prolapse Aortic stenosis Pericarditis

Hypertrophic cardiomyopathy (HCM) is the commonest form of cardiomyopathy, with a prevalence of about 100 per 100,000. It is a genetic disorder with autosomal-dominant transmission, a high degree of penetrance and variable expression. Symptoms and signs are similar to those of aortic stenosis, except that the character of the pulse in HCM is jerky. The age of the patient and her family history make HCM the likely diagnosis. Sudden death can be a presenting symptom. It typically occurs during or just after vigorous physical activity. Risk factors for sudden death in HCM are: • a history of previous cardiac arrest or sustained ventricular tachycardia • recurrent syncope • an adverse genotype and/or family history • exercise-induced hypotension • multiple episodes of non-sustained ventricular tachycardia on ambulatory ECG • a marked increase in the thickness of the left ventricular wall

A 52-year-old man undergoes Bruce-protocol exercise testing 6 weeks following an uncomplicated inferior myocardial infarction. He was ECHOed prior to his exercise test, where abnormal inferior wall motion was demonstrated. He is currently on aspirin 75 mg od, simvastatin 40 mg od, lisinopril 20 mg od and atenolol 25 mg od. Resting heart rate is 72 bpm and blood pressure is 130/70 mmHg. He achieves 4 minutes 15 seconds, stopping secondary to chest pain and associated ST-segment depression in the inferolateral leads. What would be the next stage in his management? Add diltiazem and review in clinic

of 179

6/30/2011 4:56 PM

Arrange a stress echocardiogram

www.apnastudent.blogspot.com Increase atenolol 50 mg od and repeat the exercise test Refer for coronary angiography Refer for a myocardial perfusion scan A 52-year-old man undergoes Bruce-protocol exercise testing 6 weeks following an uncomplicated inferior myocardial infarction. He was ECHOed prior to his exercise test, where abnormal inferior wall motion was demonstrated. He is currently on aspirin 75 mg od, simvastatin 40 mg od, lisinopril 20 mg od and atenolol 25 mg od. Resting heart rate is 72 bpm and blood pressure is 130/70 mmHg. He achieves 4 minutes 15 seconds, stopping secondary to chest pain and associated ST-segment depression in the inferolateral leads. What would be the next stage in his management? Add diltiazem and review in clinic Arrange a stress echocardiogram Increase atenolol 50 mg od and repeat the exercise test

Your answer

Refer for coronary angiography

Correct answer

Refer for a myocardial perfusion scan

The purpose of the exercise test postmyocardial infarction is twofold: risk stratification and patient self-confidence. Patients who are unable to perform two stages of the Bruce protocol (< 6 minutes) are at higher risk of adverse cardiovascular events. This youngish man has an early positive exercise test, as exemplified by symptoms and associated ECG changes. He should be referred for coronary angiography to accurately determine whether he has prognostic disease warranting coronary artery bypass grafting (left main stem disease, proximal three-vessel disease and proximal two-vessel disease including the left anterior descending artery). Percutaneous intervention can also be considered if he has ongoing symptoms (likely in view of his symptoms on exercise testing). I ncreasing his atenolol dose would be a sensible amendment to his current medical therapy.

As medical registrar on call you are summoned to assist with a cardiac arrest on CCU. A 60-year-old man is being resuscitated having presented with unstable angina 3 days before. He has had three unsuccessful shocks for ventricular fibrillation. An anaesthetist is looking after his airway. He has a large-bore iv access in his antecubital fossa. What additional therapy would you consider at this point? Amiodarone Bretylium Calcium chloride Lidocaine Sodium bicarbonate As medical registrar on call you are summoned to assist with a cardiac arrest on CCU. A 60-year-old man is being resuscitated having presented with unstable angina 3 days before. He has had three unsuccessful shocks for ventricular fibrillation. An anaesthetist is looking after his airway. He has a large-bore iv access in his antecubital fossa. What additional therapy would you consider at this point? Amiodarone

Correct answer

Bretylium Calcium chloride Lidocaine

Your answer

Sodium bicarbonate

of 179

6/30/2011 4:56 PM

Current recommendations for advanced life support in the UK are provided by the European Resuscitation Council and the Resuscitation Council UK. In patients with refractory ventricular fibrillation or pulseless VT (ie after three initial shocks) intravenous amiodarone should be considered. The standard dose used is a 300-mg bolus. I f central access is available this is the desired route. However, if this is not the case then a large-bore peripheral access can be used. Lidocaine (lignocaine) is used when amiodarone is unavailable.

A 72-year-old Caucasian woman is referred to out-patients for advice regarding her hypertension management. She has been on treatment in the form of perindopril 4 mg od for the past 3 years. However, on repeated measurements, her readings have been > 160 mmHg systolic, with diastolic readings being in the order of 80–85 mmHg. Renal function is normal as is urine dipstick testing. There is no evidence of left ventricular hypertrophy on ECG. She is obese with a BMI of 33. What would you consider adding as your next drug? Atenolol Bendrofluazide Doxazosin Amlodipine Spironolactone

A 72-year-old Caucasian woman is referred to out-patients for advice regarding her hypertension management. She has been on treatment in the form of perindopril 4 mg od for the past 3 years. However, on repeated measurements, her readings have been > 160 mmHg systolic, with diastolic readings being in the order of 80–85 mmHg. Renal function is normal as is urine dipstick testing. There is no evidence of left ventricular hypertrophy on ECG. She is obese with a BMI of 33. What would you consider adding as your next drug? Atenolol Bendrofluazide Doxazosin Amlodipine

Your answer

Spironolactone

This woman has hypertension resistant to a single agent. It is increasingly recognised that more than one agent is required to adequately control blood pressure. Whilst guidelines suggest either a thiazide or calcium channel antagonist could be added next, in view of the fact she is obese, a calcium channel antagonist may be the better choice. Meta-analyses have shown that both thiazides and beta blockers are associated with an increased risk of the development of Type 2 diabetes in at risk patients. Indeed, the ASCOT study did suggest that the combination of ACE inhibitor and calcium antagonist was associated with the development of less type 2 diabetes than a beta blocker/thiazide alternative.

A 53-year-old bus driver presents with a history of chest pain at rest. Initial ECG shows minor ST-segment depression in the lateral leads. Cardiac enzymes, including troponin, are normal. He has known angina, with angiography 3 years previously demonstrating minor right coronary artery disease. He is commenced on aspirin, a b-blocker and a statin. His symptoms settle over 24 hours, 12 hr troponin is normal, and following mobilisation he is discharged home. He needs to know how this episode might affect his future employment. Assuming his condition remains stable with no further symptoms, what would be the next stage of investigation? Angiography Echocardiogram Exercise testing

www.apnastudent.blogspot.com Myocardial perfusion imaging Review in outpatients after 6 weeks

of 179

6/30/2011 4:56 PM

A 53-year-old bus driver presents with a history of chest pain at rest. Initial ECG shows minor ST-segment depression in the lateral leads. Cardiac enzymes, including troponin, are normal. He has known angina, with angiography 3 years previously demonstrating minor right coronary artery disease. He is commenced on aspirin, a b-blocker and a statin. His symptoms settle over 24 hours, 12 hr troponin is normal, and following mobilisation he is discharged home. He needs to know how this episode might affect his future employment. Assuming his condition remains stable with no further symptoms, what would be the next stage of investigation? Angiography Echocardiogram Exercise testing

Your answer

Myocardial perfusion imaging Review in outpatients after 6 weeks

This man holds a group 2 licence, and hence he should be advised to inform the DVLA of his recent presentation with unstable angina and stop work until re-licensing can be performed. Up-to-date information can be obtained from the DVLA ( http://www.dvla.gov.uk). For group 2 licence holders, all acute coronary syndromes are considered relevant and this disqualifies the individual from driving for at least 6 weeks. Re-licensing may be permitted if a suitable exercise test is achieved (need to complete three stages of the Bruce protocol or equivalent safely, without antianginal medication for 48 hours and without significant symptoms, ECG or haemodynamic abnormalities) and there are no other disqualifying conditions. Although angiography is not required for re-licensing, if it is performed then specific guidelines are available on the DVLA website. I n this case he would be exercised at the 6 week stage.

A 50-year-old woman is referred to out-patients for a previously asymptomatic atrial septal defect (ASD). She is new to the area and was last seen around 6 years ago in her previous local hospital. She is a smoker but without other significant medical history. She now complains of shortness of breath on exertion, together with peripheral oedema. Clinical examination reveals her to be clubbed and cyanosed. Her pulse rate is 90 bpm and blood pressure 98/60 mmHg. Echo demonstrates a dilated right heart with an estimated right ventricular pressure of 90 mmHg and significant tricuspid and pulmonary regurgitation. What is the likely diagnosis? Cor pulmonale Eisenmenger’s syndrome Infective endocarditis Primary pulmonary hypertension Pulmonary emboli disease A 50-year-old woman is referred to out-patients for a previously asymptomatic atrial septal defect (ASD). She is new to the area and was last seen around 6 years ago in her previous local hospital. She is a smoker but without other significant medical history. She now complains of shortness of breath on exertion, together with peripheral oedema. Clinical examination reveals her to be clubbed and cyanosed. Her pulse rate is 90 bpm and blood pressure 98/60 mmHg. Echo demonstrates a dilated right heart with an estimated right ventricular pressure of 90 mmHg and significant tricuspid and pulmonary regurgitation. What is the likely diagnosis? Cor pulmonale Eisenmenger’s syndrome Infective endocarditis Primary pulmonary hypertension Pulmonary emboli disease

Your answer

www.apnastudent.blogspot.com This woman has developed massive irreversible pulmonary hypertension as a consequence of a previous left to right shunt. Pulmonary pressures have now reached systemic level. The reversal of a left to right shunt as a consequence of pulmonary hypertension is known as Eisenmenger’s syndrome, and is generally the result of a previously undiagnosed ASD, ventricular septal defect (VSD) or patent ductus arteriosus. I t may also result from an incompletely corrected

of 179

6/30/2011 4:56 PM

Fallot’s tetralogy or Ebstein’s anomaly. Prognosis is poor, although a few patients may be candidates for heart–lung transplantation. Symptomatic treatment is directed towards right heart failure. Complications include polycythaemia, bleeding disorders and cerebral embolism or abscess. Since there is a particularly high risk in those of childbearing age, patients should be given appropriate advice and information to avoid pregnancy.

A 40-year-old man is referred by his GP for advice with regard to primary prevention of cardiovascular disease. He is a smoker with a strong family history of premature death from ischaemic heart disease. Following a period of lifestyle modification, his fasting cholesterol concentration is 7.2 mmol/l. On consultation of the local guidelines you find that his estimated 10-year risk of a coronary heart disease event is > 30%. What would you advise? Cholestyramine Dietician advice Fibrate Nicotinic acid Statin A 40-year-old man is referred by his GP for advice with regard to primary prevention of cardiovascular disease. He is a smoker with a strong family history of premature death from ischaemic heart disease. Following a period of lifestyle modification, his fasting cholesterol concentration is 7.2 mmol/l. On consultation of the local guidelines you find that his estimated 10-year risk of a coronary heart disease event is > 30%. What would you advise? Cholestyramine Dietician advice Fibrate Nicotinic acid Statin

Your answer

The National Service Framework for Coronary Heart Disease (2000) and the Joint British Society Guidelines recommend targeting individuals with a 10-year risk of a coronary heart disease event of > 30%. Individuals should be offered interventions to address all modifiable risk factors, including dietary advice, smoking cessation advice and support, moderation of alcohol consumption and weight reduction where appropriate. In respect of lipid management, non-pharmacological and pharmacological interventions should be utilised to achieve a total cholesterol concentration < 5.0 mmol/l and an LDL cholesterol concentration of < 3.0 mmol/l. The results of several important trials support the use of statins in primary prevention, these include: WOSCOPS (pravastatin) AFCAPS/TEXCAPS (LEVASTATIN).

Which one of the following statements BEST describes primary pulmonary hypertension? The familial form is inherited as sex-linked recessive Chronic thromboembolic disease can be identified in 30% of primary cases Spontaneous remission is the rule in more than half the cases Cannabis inhalation may induce similar disease The risk for subacute bacterial endocarditis is low and antibiotic prophylaxis is seldom required Which one of the following statements BEST describes primary pulmonary hypertension? The familial form is inherited as sex-linked recessive Chronic thromboembolic disease can be identified in 30% of primary cases

Your answer

Spontaneous remission is the rule in more than half the cases Cannabis inhalation may induce similar disease The risk for subacute bacterial endocarditis is low and antibiotic prophylaxis is seldom required

Correct answer

www.apnastudent.blogspot.com

One of the diagnostic criteria includes a mean pulmonary artery pressure of more than 25 mmHg at rest or more than 30 mmHg with exercise. Recurrent thromboembolism is one cause of secondary pulmonary hypertension (not primary). Fenfluramine, cocaine inhalation and HIV infection can cause pulmonary vascular disease with clinical and pathological features similar to those of primary pulmonary hypertension. The familial form is inherited as autosomal dominant. The medium period of survival is two to three years after the diagnosis. Recent improvement in diagnosis and newer forms of treatment have improved survival, but the prognosis is generally very poor and most patients gradually succumb to progressive right-sided heart failure.

Which one of the following statements is MOST accurate regarding coarctation of the aorta? The coarctation is proximal to the left subclavian artery origin if the right arm blood pressure is significantly higher than in the left arm Continuous murmur over the thoracic spine usually originates from extensive collaterals Rib notching on plain chest X-ray can be identified as early as three months after birth Atrial septal defect (ASD) is the commonest associated congenital abnormality The risk for bacterial endocarditis means that antibiotic prophylaxis is required prior to all dental procedures Which one of the following statements is MOST accurate regarding coarctation of the aorta? The coarctation is proximal to the left subclavian artery origin if the right arm blood pressure is significantly higher than in the left arm

Your answer

Continuous murmur over the thoracic spine usually originates from extensive collaterals Rib notching on plain chest X-ray can be identified as early as three months after birth Atrial septal defect (ASD) is the commonest associated congenital abnormality The risk for bacterial endocarditis means that antibiotic prophylaxis is required prior to all dental procedures

The commonest site of discrete obstruction of the aortic lumen is just distal to the origin of the left subclavian artery. The systolic arterial pressure in the arms exceeds that in the leg. However, if the systolic arterial pressure in the right arm is higher than that of the left arm by more than 30 mmHg, the left subclavian is involved in the coarctation (i.e. coarctation is proximal to the origin of the subclavian as in this case). A continuous murmur over the thoracic spine usually originates from small, tight coarctation (< 2 mm). Other cardiac malformations are frequent, the commonest being a bicuspid aortic valve. Notching of the inferior border of the ribs from collateral vessels is common and usually manifest in adults and older children. Patients with unrepaired coarctation are at risk of bacterial endocarditis and should be advised about this possibility. Antibiotic prophylaxis is no longer advised given the poor evidence that this is effective in reducing the risks of infection following dental or other procedures.

Which one of the following is characteristic of atrial myxoma? Usually originates in the right atrium Fragments of tumour easily break off and grow in its peripheral sites Echocardiogram is diagnostic in most cases The clinical signs can mimic severe mitral regurgitation Recurrence is frequent even after successful surgical removal of the tumour Which one of the following is characteristic of atrial myxoma? Usually originates in the right atrium Fragments of tumour easily break off and grow in its peripheral sites Echocardiogram is diagnostic in most cases

of 179

Your answer

6/30/2011 4:56 PM

The clinical signs can mimic severe mitral regurgitation Recurrence is frequent even after successful surgical removal of the tumour

www.apnastudent.blogspot.com

Atrial myxoma is a benign tumour of the heart. Approximately 75% originate in the left atrium. The clinical features are characterised by a triad of embolism, intracardiac obstruction and constitutional symptoms. The clinical signs can mimic mitral stenosis and the murmur may vary with body position. Fragments of tumour easily break off but do not grow in its peripheral sites. After complete and careful removal of the tumour recurrence is very rare.

A 57-year-old female is admitted with Gram-negative septicaemia. She is given intravenous antibiotics and normal saline. Two days later she becomes anxious, tachypnoeic, and short of breath. An emergency chest X-ray demonstrates diffuse, bilateral interstitial and alveolar infiltrates. Her past medical history revealed hypertension and that she has been on regular antihypertensive treatment for seven years. She has never had any evidence of congestive heart failure. In this case, adult respiratory distress syndrome can be distinguished from cardiogenic pulmonary oedema by? Asymmetrical hypertrophy of the interventricular septum is revealed on echocardiography Calculation of the alveolar-arterial p O2 difference Measurement of pulmonary artery wedge pressure Measurement of lung compliance Measurement of ejection fraction A 57-year-old female is admitted with Gram-negative septicaemia. She is given intravenous antibiotics and normal saline. Two days later she becomes anxious, tachypnoeic, and short of breath. An emergency chest X-ray demonstrates diffuse, bilateral interstitial and alveolar infiltrates. Her past medical history revealed hypertension and that she has been on regular antihypertensive treatment for seven years. She has never had any evidence of congestive heart failure. In this case, adult respiratory distress syndrome can be distinguished from cardiogenic pulmonary oedema by? Asymmetrical hypertrophy of the interventricular septum is revealed on echocardiography Calculation of the alveolar-arterial p O2 difference Measurement of pulmonary artery wedge pressure

Your answer

Measurement of lung compliance Measurement of ejection fraction

The adult respiratory distress syndrome (ARDS) is a clinical triad of hypoxaemia, diffuse lung infiltrates, and reduced lung compliance not attributable to congestive cardiac failure. This has been reported as a complication of apparently unrelated conditions. Examples include sepsis, lung contusion and drug overdose. Increase in lung water in ARDS occurs as a result of an increase in alveolar capillary permeability and is not due to an increase in hydrostatic forces. Clinically and radiographically, ARDS closely resembles severe haemodynamic pulmonary oedema due to heart failure. The distinction between these disorders is often apparent from the clinical circumstances associated with the onset of respiratory distress, whereas differentiation by radiographic means alone is often extremely difficult. As in cardiac pulmonary oedema, the increase in lung water associated with ARDS produces interstitial oedema and alveolar collapse, and so the affected lung becomes stiff and the alveolar-arterial oxygen tension difference widens. The central venous pressure and ejection fraction may alter but would not reflect the underlying pathophysiological mechanism. A Swan-Ganz catheter should be placed if the mechanism of oedema formation cannot be discerned with confidence. A pulmonary capillary wedge pressure < 18 mmHg favours acute lung injury over haemodynamic pulmonary oedema. In clinical practice, determination of pulmonary artery wedge pressure is the most helpful discriminate between ARDS and cardiac failure.

A 48-year-old man is admitted with a prolonged episode of chest pain at rest. The ECG shows ST depression in the lateral leads and his troponin T level is just above the normal range. Which of the following is the most appropriate combination of drugs for initial treatment? Aspirin, ramipril, unfractionated heparin, diltiazem Aspirin, warfarin, low molecular weight heparin, atenolol Aspirin, clopidogrel, low molecular weight heparin, atenolol

of 179

6/30/2011 4:56 PM

Aspirin, losartan, unfractionated heparin, atenolol Aspirin, ramipril, low molecular weight heparin, nicorandil

www.apnastudent.blogspot.com A 48-year-old man is admitted with a prolonged episode of chest pain at rest. The ECG shows ST depression in the lateral leads and his troponin T level is just above the normal range. Which of the following is the most appropriate combination of drugs for initial treatment? Aspirin, ramipril, unfractionated heparin, diltiazem Aspirin, warfarin, low molecular weight heparin, atenolol Aspirin, clopidogrel, low molecular weight heparin, atenolol

Your answer

Aspirin, losartan, unfractionated heparin, atenolol Aspirin, ramipril, low molecular weight heparin, nicorandil

The initial treatment of unstable angina (UA) should include bed rest, anti-platelet therapy, anticoagulation and a β-blocker. A systematic review found that aspirin alone (75–325 mg/day) reduces the risk of death and myocardial infarction in patients with UA. A large, randomised, control trial (RCT) has shown that the combination of clopidogrel (75 mg/day) and aspirin is superior to aspirin alone. Many RCTs have found that treating those patients at risk of UA with low molecular weight heparin (LMWH) is more effective than aspirin alone. The advantages of LMWH over unfractionated heparin include its ease of administration and no need for monitoring. Diltiazem or verapamil can be used if are contraindicated.

β-blockers

Patients with a high risk of UA should be considered for revascularisation. Those who undergo coronary angioplasty should also be considered for treatment with an intravenous glycoprotein II b/II Ia inhibitor such as abciximab, tirofiban or eptifibatide.

A 30-year-old woman presents with a three month history of chest pain. On auscultation, there is a midsystolic click and a late systolic murmur. Her electrocardiogram shows T-wave inversions in leads I I, II I, and aVF. Which of the following statements concerning her condition is true? The woman’s chest pain is almost certainly due to associated coronary artery disease The click and murmur is likely to occur earlier in systole when the patient stands An exercise stress test would most likely be positive Asymmetrical hypertrophy of the interventricular septum is revealed on echocardiography Prophylactic measures to prevent subacute bacterial endocarditis are warranted

A 30-year-old woman presents with a three month history of chest pain. On auscultation, there is a midsystolic click and a late systolic murmur. Her electrocardiogram shows T-wave inversions in leads I I, II I, and aVF. Which of the following statements concerning her condition is true? The woman’s chest pain is almost certainly due to associated coronary artery disease The click and murmur is likely to occur earlier in systole when the patient stands

Your answer

An exercise stress test would most likely be positive Asymmetrical hypertrophy of the interventricular septum is revealed on echocardiography Prophylactic measures to prevent subacute bacterial endocarditis are warranted

The systolic click-murmur syndrome is associated with mitral valve prolapse. It occurs in approximately 4% of the normal asymptomatic population. It can place excessive stress on the papillary muscles and lead to ischaemia and chest pain. Although often associated with inferior T-wave changes, the systolic click-murmur syndrome only occasionally results in an ischaemic response to exercise. On standing or during the Valsalva manoeuvre, as ventricular volume gets smaller, the click and murmur move earlier in systole. Echocardiography reveals mid-systolic prolapse of the posterior mitral leaflet or, on occasion, both mitral leaflets into the left atrium. Asymmetrical hypertrophy of the interventricular septum is a feature of hypertrophic obstructive cardiomyopathy (HOCM). According to recent guidelines (American Heart Association, 2007) prophylaxis against infective endocarditis (I E) is no longer recommended for patients with mitral valve prolapse (MVP).

of 179

6/30/2011 4:56 PM

http://circ.ahajournals.org/cgi/reprint/116/15/1736

www.apnastudent.blogspot.com Aortic stenosis in adults is commonly the result of which one of the following? Bicuspid aortic valve disease Left ventricular membrane Hypertrophic obstructive cardiomyopathy (HOCM) Rheumatic fever Cystic medial necrosis Aortic stenosis in adults is commonly the result of which one of the following? Bicuspid aortic valve disease

Your answer

Left ventricular membrane Hypertrophic obstructive cardiomyopathy (HOCM) Rheumatic fever Cystic medial necrosis

Approximately 1% of the general population has a bicuspid aortic valve defect. The bicuspid aortic valve may function normally throughout life, with late stenosis resulting from fibrocalcific thickening. Aortic stenosis resulting from bicuspid valve disease occurs from increasing rigidity of the abnormal aortic valve and increasing calcification. The congenital form of bicuspid valve disease is conjoined anteriorly.

Left bundle branch block is associated with which one of the following conditions? Ischaemic heart disease Mitral stenosis Pericarditis Pulmonary embolism Tricuspid stenosis Left bundle branch block is associated with which one of the following conditions? Ischaemic heart disease

Correct answer

Mitral stenosis Pericarditis Pulmonary embolism

Your answer

Tricuspid stenosis

Mitral stenosis, tricuspid stenosis and secondary pulmonary hypertension due to pulmonary embolism are associated with right ventricular strain and hypertrophy with partial or complete right bundle branch block. Pericarditis is not associated with bundle branch block.

Normal pregnancy is associated with which one of the following haemodynamic changes? A 20% reduction in blood volume and cardiac output A 10 mmHg drop in diastolic blood pressure during the second trimester Bradycardia with a radial pulse rate between 45 and 55 beats per minute Grade 2/6 diastolic murmur at the mitral area

of 179

6/30/2011 4:56 PM

Pulsus alternans Normal pregnancy is associated with which one of the following haemodynamic changes? A 20% reduction in blood volume and cardiac output

www.apnastudent.blogspot.com A 10 mmHg drop in diastolic blood pressure during the second trimester

Your answer

Bradycardia with a radial pulse rate between 45 and 55 beats per minute Grade 2/6 diastolic murmur at the mitral area Pulsus alternans

Despite an expansion of the plasma volume and cardiac output of 50%, mean and diastolic blood pressures fall by approximately 15% owing to a reduction in peripheral vascular resistance. There is little change in systolic blood pressure, but diastolic pressure is reduced (5-10 mmHg) from about 12-16 weeks. Diastolic pressure usually increases thereafter to pre-pregnancy levels by about 36 weeks. Tachycardia rather than bradycardia is a recognised physiological change during pregnancy. I t is a consequence of reduced peripheral vascular resistance and fall in blood pressure levels. The heart may be slightly enlarged and may be displaced outward because of the high diaphragm. A pulmonary systolic murmur from a high blood flow is common and there may be a physiological third heart sound. Diastolic murmurs are generally pathological and at the mitral area may signify mitral stenosis. The presence of pulsus alternans usually signifies advanced heart failure.

A 60-year-old man underwent a coronary angiogram for unstable angina. The next day whilst recovering in hospital he complains of severe pain in his right foot and partial loss ofsight in the left eye. On examination the lower limb peripheral pulses are present and of good volume. There is gangrene of the lateral two toes on the right foot. Fundoscopy reveals cholesterol emboli in a branch of the central retinal artery in the left eye. Which one of the following is the most probable diagnosis in this case? Atheroembolic disease Polyarteritis nodosa Buerger’s disease Arterial thromboembolism Disseminated intravascular coagulopathy A 60-year-old man underwent a coronary angiogram for unstable angina. The next day whilst recovering in hospital he complains of severe pain in his right foot and partial loss ofsight in the left eye. On examination the lower limb peripheral pulses are present and of good volume. There is gangrene of the lateral two toes on the right foot. Fundoscopy reveals cholesterol emboli in a branch of the central retinal artery in the left eye. Which one of the following is the most probable diagnosis in this case? Atheroembolic disease

Correct answer

Polyarteritis nodosa Buerger’s disease Arterial thromboembolism

Your answer

Disseminated intravascular coagulopathy

Although each of the mentioned options is a valid possible underlying cause behind this presentation, it is clear that the picture is more typical of atheroembolic disease. I t is due to cholesterol emboli lodged in peripheral arteries, commonly as a result of angiographic or other surgical vascular procedures. Clearly the clinical features will depend on the site of embolisation. The most common clinical findings are cutaneous features, renal failure and worsening hypertension. The presence of foot pulses with gangrenous toes should suggest cholesterol embolisation. The retina provides a unique opportunity to visualise the cholesterol emboli. Renal failure may manifest as gradual deterioration of renal function following angiography or may be acute (this may mimic acute dissection of the renal artery during renal angiography). Eosinophilia, eosinophiluria, a raised ESR and hypocomplementinaemia have been found in atheroembolic disease. Arterial thromboembolism is related to distal embolisation of proximal, pre-existing atheroma.

Which one of the following features is MORE common in constrictive pericarditis than in cardiac tamponade?

of 179

6/30/2011 4:56 PM

Pulsus paradoxus Kussmaul’s sign

www.apnastudent.blogspot.com Prominent x trough 4-chamber diastolic equilibrium Hypotension Which one of the following features is MORE common in constrictive pericarditis than in cardiac tamponade? Pulsus paradoxus Kussmaul’s sign

Your answer

Prominent x trough 4-chamber diastolic equilibrium Hypotension

An inspiratory increase in venous pressure (Kussmaul’s sign) and a steep y descent in the jugular pulse are features of constrictive pericarditis. Pericardial knock in early diastole is often seen in constrictive pericarditis. Both conditions cause failure of either side of the heart and the diastolic pressure in all cardiac chambers are equal. A paradoxical pulse and prominent x trough in the jugular pulse are more common in tamponade than in constrictive pericarditis.

A 30-year-old man with known hypertrophic obstructive cardiomyopathy (HOCM) presents to casualty with an episode of witnessed syncope: a passer-by provided initial resuscitation. On admission he is unwell with pulse rate of 160 bpm, blood pressure 70/40 mmHg and decreased conscious level. ECG confirms ventricular tachycardia. Sinus rhythm is restored with a DC shock. What would be the most appropriate strategy for the long term? Amiodarone Automatic implantable cardioverter defibrillator Dual-chamber pacemaker Sotalol Verapamil

A 30-year-old man with known hypertrophic obstructive cardiomyopathy (HOCM) presents to casualty with an episode of witnessed syncope: a passer-by provided initial resuscitation. On admission he is unwell with pulse rate of 160 bpm, blood pressure 70/40 mmHg and decreased conscious level. ECG confirms ventricular tachycardia. Sinus rhythm is restored with a DC shock. What would be the most appropriate strategy for the long term? Amiodarone Automatic implantable cardioverter defibrillator

Your answer

Dual-chamber pacemaker Sotalol Verapamil

This man has survived an out-of-hospital cardiac arrest and therefore an automatic implantable cardioverter defibrillator (AICD) is warranted. Overall, patients with HOCM have an annual mortality rate of around 1%. I dentifying those at greatest risk of sudden cardiac death (SCD) is challenging. However, several factors have been identified that are associated with an increased risk: maximum wall thickness > 30 mm non-sustained ventricular tachycardia on a 48-hour tape a history of SCD in a relative under 45 years of age and a history of syncope resting, left ventricular outflow-tract gradient > 30 mmHg abnormal blood-pressure response to exercise. Although a single risk factor does not, on its own, have a particularly high positive-predictive accuracy, the presence of

of 179

6/30/2011 4:56 PM

two or more risk factors does identify a much higher risk population. Dual-chamber pacing, be used to reduce symptoms in patients with a left ventricular outflow-tract obstruction.

β-blockers or

verapamil may

www.apnastudent.blogspot.com A 45-year-old woman is being investigated for heart disease. It is found that the pressure–volume curve of the left ventricle is shifted to the left. What is the most likely diagnosis in this case? Aortic regurgitation Mitral stenosis Aortic stenosis Mitral regurgitation Tricuspid stenosis

A 45-year-old woman is being investigated for heart disease. It is found that the pressure–volume curve of the left ventricle is shifted to the left. What is the most likely diagnosis in this case? Aortic regurgitation Mitral stenosis Aortic stenosis

Your answer

Mitral regurgitation Tricuspid stenosis

The pressure–volume curve denotes the pumping mechanics of the heart chambers. Most studies refer to the left ventricle, as this is the main chamber. In aortic stenosis, there is pressure overload leading to concentric hypertrophy of the left ventricle. This causes increased contractility and decreased compliance of the chamber. More pressure is thus exerted to eject the same volume of blood. The pressure–volume curve therefore shifts to the left. In aortic and mitral regurgitation, volume overload occurs leading to a dilated left ventricle. This causes decreased contractility and increased compliance and shifts the pressure–volume curve to the right. In mitral and tricuspid stenosis, there is pressure overload in the left atrium and not the left ventricle. There would thus be no change in the pressure–volume curve of the left ventricle.

The pressure–volume curve in a patient with heart failure is shifted to the right. What is the most important feature in cardiovascular dynamics responsible for this right shift? Increased contractility of the chamber Increased sympathetic activity Concentric hypertrophy of the chamber Increased compliance of the chamber Pressure overload in the chamber The pressure–volume curve in a patient with heart failure is shifted to the right. What is the most important feature in cardiovascular dynamics responsible for this right shift? Increased contractility of the chamber Increased sympathetic activity Concentric hypertrophy of the chamber Increased compliance of the chamber

Your answer

Pressure overload in the chamber

of 179

6/30/2011 4:56 PM

Shifting of the pressure–volume curve to the right occurs when there is volume overload, as in aortic and mitral regurgitation. There is decreased contractility and increased compliance. The ventricle dilates to accommodate the increased volume. Hypertrophy of the cardiac muscle does not occur. I ncreased sympathetic activity, catecholamine administration or exercise would shift the pressure–volume curve to the left.

A 65-year-old man with angina pectoris undergoes serum lipid testing. Which of the following abnormalities is most likely to be found? Increased triglyceride levels Increased low-density lipoprotein cholesterol levels Increased high-density lipoprotein cholesterol levels Increased chylomicrons Increased intermediate-density lipoprotein cholesterol levels A 65-year-old man with angina pectoris undergoes serum lipid testing. Which of the following abnormalities is most likely to be found? Increased triglyceride levels Increased low-density lipoprotein cholesterol levels

Your answer

Increased high-density lipoprotein cholesterol levels Increased chylomicrons Increased intermediate-density lipoprotein cholesterol levels

LDL particles are the main carriers of cholesterol. These particles can deposit lipid into the walls of the peripheral vasculature. There is a strong association between both total- and LDL-cholesterol concentration and coronary heart risk. There is a relatively weak independent link between raised concentrations of (triglyceride-rich) VLDL (very low-density lipoprotein) particles and cardiovascular risk. Very highly raised triglyceride levels (> 6 mmol/l) cause a greatly increased risk of acute pancreatitis and retinal vein thrombosis. Higher HDL concentrations protect against cardiovascular disease. HDL also has effects on the function of platelets and of the haemostatic cascade. These properties may favourably influence thrombogenesis. Excess chylomicrons do not confer an excess cardiovascular risk but do raise the total plasma triglyceride concentration.

A 58-year-old-woman suffers a cardiac arrest while on the ward. A rhythm strip shows VF. What is the strength (in joules) recommended for the monophasic shock used for defibrillation? 50J 100J 200J 300J 360J A 58-year-old-woman suffers a cardiac arrest while on the ward. A rhythm strip shows VF. What is the strength (in joules) recommended for the monophasic shock used for defibrillation? 50J 100J

www.apnastudent.blogspot.com 200J

Your answer

300J 360J

of 179

Correct answer

6/30/2011 4:56 PM

Three-quarters of arrests are due to ventricular fibrillation. Only a small proportion is due to pulseless electrical activity (PEA), the rest being due to asystole. PEA may have a potentially reversible cause: • hypovolaemia • hypoxia • hyperkalaemia • hypokalaemia • hypothermia • tension pneumothorax • tamponade • toxicity due to drugs • thromboembolism Defibrillation is used to convert VF to sinus rhythm. The recommendation is initially a 360-joule shock.

A 70-year-old obese man is admitted with a 6-hour history of chest pain. An ECG reveals an inferior wall myocardial infarction. Measurement of which of the following would best confirm the diagnosis? Creatine kinase Creatine kinase MB Cardiac-specific troponin T Aspartate aminotransferase Lactate dehydrogenase A 70-year-old obese man is admitted with a 6-hour history of chest pain. An ECG reveals an inferior wall myocardial infarction. Measurement of which of the following would best confirm the diagnosis? Creatine kinase Creatine kinase MB

Your answer

Cardiac-specific troponin T

Correct answer

Aspartate aminotransferase Lactate dehydrogenase

Troponin T and troponin I are regulatory proteins with a very high specificity for cardiac injury. They are released early (2–4 h) and can persist for up to 7 days. Most hospitals check levels at 6 and 12hrs after admission. They are more sensitive and cardiospecific than CKMB. The latter is a cardiac-specific isoform of creatine kinase and allows greater diagnostic accuracy than creatine kinase. Both aspartate aminotransferase and lactate dehydrogenase are non-specific enzymes that are rarely used nowadays for the diagnosis of myocardial infarction. LDH peaks at 3–4 days and remains elevated for up to 10 days following a cardiac event, and can thus be useful in confirming myocardial infarction in patients presenting several days after an episode of chest pain.

A 40-year-old salesman presents with frequent flushing of his face and neck, abdominal pain and watery diarrhoea, fatigue, breathlessness, anorexia and nausea. On examination, there is jugular venous distension with prominent v waves, hepatomegaly and dependent oedema. On auscultation, a blowing pansystolic murmur is heard on inspiration at the lower left sternal edge. What is the most likely cardiac abnormality in this case? Mitral regurgitation

www.apnastudent.blogspot.com Tricuspid incompetence Tricuspid stenosis Pulmonary stenosis Prolapsing mitral valve A 40-year-old salesman presents with frequent flushing of his face and neck, abdominal pain and watery diarrhoea, fatigue, breathlessness, anorexia and nausea. On examination, there is jugular venous distension with prominent v waves, hepatomegaly and dependent oedema. On auscultation, a blowing pansystolic murmur is heard on inspiration at the lower left sternal edge. What is the most likely cardiac abnormality in this case? Mitral regurgitation Tricuspid incompetence

Your answer

Tricuspid stenosis Pulmonary stenosis Prolapsing mitral valve

This patient most probably has carcinoid syndrome, which can occur in 5% of patients with carcinoid tumours when there are liver metastases. Cardiac abnormalities are found in 50% of patients, and consist of pulmonary stenosis or tricuspid incompetence. The auscultatory findings in this case are suggestive of tricuspid incompetence. In pulmonary stenosis, the characteristic auscultatory finding is a harsh mid-systolic ejection murmur best heard on inspiration to the left of the sternum in the second intercostal space. A right ventricular fourth sound and a prominent jugular venous a wave are both present when the stenosis is moderately severe. A rumbling mid-diastolic murmur is characteristic of tricuspid stenosis along with a prominent jugular venous a wave. Mitral regurgitation does not usually occur due to carcinoid syndrome. In mitral regurgitation, a pansystolic murmur is heard loudest at the apex and radiating widely over the precordium and into the axilla. A prominent third heart sound may be present. Prolapsing (floppy) mitral valve is commonly seen in young women and has a familial incidence. It may be associated with rheumatic or ischaemic heart disease, Marfan’s syndrome and thyrotoxicosis. The most common sign is a mid-systolic click followed by a late systolic murmur due to some regurgitation.

A patient who has been inadvertently given an intravenous injection of potassium chloride, develops ventricular tachycardia. His pulse is 150 beats per minute and blood pressure 60/40 mmHg. What would be the best line of treatment in this case? Lidocaine Insulin 10 units and 50 ml of 50% glucose Amiodarone DC cardioversion 10 ml of 10% calcium gluconate A patient who has been inadvertently given an intravenous injection of potassium chloride, develops ventricular tachycardia. His pulse is 150 beats per minute and blood pressure 60/40 mmHg. What would be the best line of treatment in this case? Lidocaine Insulin 10 units and 50 ml of 50% glucose Amiodarone DC cardioversion 10 ml of 10% calcium gluconate

of 179

Correct answer Your answer

6/30/2011 4:56 PM

Since the patient is haemodynamically compromised, the emergency step would be to carry out DC cardioversion as utherwise there is a risk of death. Injection of 10 ml of 10% calcium gluconate would help to protect the myocardium

www.apnastudent.blogspot.com against hyperkalaemia. Calcium ions protect the cell membranes from the effects of hyperkalaemia but do not alter the potassium concentration. Insulin drives potassium into the cell and must be accompanied by glucose to avoid hypoglycaemia. Lidocaine and amiodarone are only useful in stable cases.

A 65-year-old man with chronic renal failure has a serum potassium level of 7.1 mmol/l (normal 3.5-5.5 mmol/l). What would be the most characteristic finding on ECG? Reduced P waves Prolonged QT intervals Prominent U waves Narrow QRS complexes T-wave inversion

A 65-year-old man with chronic renal failure has a serum potassium level of 7.1 mmol/l (normal 3.5-5.5 mmol/l). What would be the most characteristic finding on ECG? Reduced P waves

Correct answer

Prolonged QT intervals Prominent U waves

Your answer

Narrow QRS complexes T-wave inversion

Hyperkalaemia causes hyperpolarisation of cell membranes, leading to decreased cardiac excitability, hypotension, bradycardia and eventual asystole. The ECG shows characteristic tall, peaked T waves with widened QRS complexes. There is a progressive diminution in the amplitude of the P wave, which eventually disappears. Prominent U waves are seen in hypokalaemia while T-wave inversion occurs in ischaemic heart disease. Prolonged QT intervals are seen in acute myocardial infarction, hypocalcaemia, hypothermia and procainamide administration.

A patient with acute inferior wall myocardial infarction develops shock. Auscultation does not reveal any murmurs. Which of the following complications of his MI is most likely to be the cause? Cardiac rupture Interventricular septal perforation Papillary muscle rupture Right ventricular infarction Atrial fibrillation

A patient with acute inferior wall myocardial infarction develops shock. Auscultation does not reveal any murmurs. Which of the following complications of his MI is most likely to be the cause? Cardiac rupture

Your answer

Interventricular septal perforation Papillary muscle rupture Right ventricular infarction

Correct answer

Atrial fibrillation

of 179

6/30/2011 4:56 PM

Right ventricular infarction occurs in one-third of cases of inferior wall myocardial infarction, which leads to pooling of blood in the right ventricle and a consequent decreased preload in the left ventricle. Hypovolaemic shock results. Papillary muscle rupture and atrial fibrillation are other complications which are also more common in inferior myocardial infarction, interventricular septal rupture is commoner in anterior myocardial infarction. Atrial fibrillation occurs as a complication in 10% of patients with myocardial infarction. It does not give rise to shock.

A 12-year-old boy with known heart disease is being advised regarding the risks of infective endocarditis. Which cardiac lesion is most likely to be prone to infection? Atrial septal defect Aortic regurgitation

Mitral

stenosis Mitral valve prolapse without regurgitation Mitral regurgitation A 12-year-old boy with known heart disease is being advised regarding the risks of infective endocarditis. Which cardiac lesion is most likely to be prone to infection? Atrial septal defect Aortic regurgitation Mitral stenosis

Correct answer

Your answer

Mitral valve prolapse without regurgitation Mitral regurgitation

Patients at high risk for infective (bacterial) endocarditis include those with prosthetic materials (valves, patches, conduits or shunts), particularly in the first 6 months after placement i.e. before these materials become endothelialised. Other risk factors include complex cyanotic heart disease and previous episode(s) of endocarditis. Endocarditis more commonly affects the left (high pressure) side of the heart than the right. I nfection of previously abnormal valves most commonly involves the aortic valves. Mitral regurgitation and mitral valve prolapse with regurgitation present a moderate risk, while mitral valve prolapse without regurgitation is a low risk. Infective endocarditis in pure mitral stenosis and atrial septal defect is uncommon. Patients at risk of infective endocarditis should be made aware of presenting symptoms and counseled to seek medical advice if they become unwell. If the diagnosis is considered it is essential that repeated sets of blood cultures are obtained before antibiotics are started. Recent UK guidelines no longer support the use of antibiotic prophylaxis to prevent endocarditis after dental or other procedures. See http://www.nice.org.uk/Guidance/CG64/Guidance/pdf/English

A 69-year-old man presents to a small district general hospital during the night with a 3-hour history of chest pain. ECG shows an inferior wall infarction with ST elevation of 3 mm. There is no history of diabetes mellitus, injury or previous surgery. Blood pressure is 132/70 mmHg with a pulse of 58/min. Which of the following treatments would be most appropriate? Tissue plasminogen activator Aspirin 2b3a inhibitor Heparin Metoprolol

www.apnastudent.blogspot.com A 69-year-old man presents to a small district general hospital during the night with a 3-hour history of chest pain. ECG shows an inferior wall infarction with ST elevation of 3 mm. There is no history of diabetes mellitus, injury or previous surgery. Blood pressure is 132/70 mmHg with a pulse of 58/min. Which of the following treatments would be most appropriate?

of 179

6/30/2011 4:56 PM

Tissue plasminogen activator

Your answer

Aspirin 2b3a inhibitor Heparin Metoprolol

In ST elevation MI (STEMI ), patients with persistent ST elevation should be considered for reperfusion therapy (thrombolysis or primary PCI). When PCI cannot be provided within 90 minutes ("door to balloon time") then thrombolysis is appropriate. Many large trials have shown that thrombolysis within 12 hours reduces the extent of ventricular damage and the mortality rate. Tissue plasminogen activator (TPA) achieves higher reperfusion rates than streptokinase but may be associated with a higher risk of bleeding stroke than PTCI . TPA tends to be given in preference to streptokinase in patients under 50 years of age with anterior wall myocardial infarctions where the blood pressure is low (systolic < 100 mmHg), and in those patients who have previously received streptokinase. TPA also appears to be more effective than streptokinase if it is administered within 4 hours of the onset of chest pain. I ntravenous heparin may be given after the initial thrombolytic therapy though its role is doubtful. Aspirin (300 mg tablet) is usually recommended. Following initiation of thrombolysis, an intravenous the heart rate is > 100 beats per minute with persistent pain.

β-blocker such

as metoprolol is given, especially if

See also SIGN guidelines on acute coronary syndromes http:/www.sign.ac.uk/pdf/qrgchd.pdf

A 72-year-old diabetic man is admitted to A&E with a 40 minute history of central, crushing chest pain. The pain eases after an hour with bedrest, oxygen and morphine. ECG shows mild anterior ST flattening. The troponin T level is slightly raised. What would be the optimal management of the underlying cause of his chest pain be besides usual medical measures? Discharge home with referral to the outpatients department Low molecular weight heparin Abciximab Thrombolysis with tissue plasminogen activator Urgent coronary angiography A 72-year-old diabetic man is admitted to A&E with a 40 minute history of central, crushing chest pain. The pain eases after an hour with bedrest, oxygen and morphine. ECG shows mild anterior ST flattening. The troponin T level is slightly raised. What would be the optimal management of the underlying cause of his chest pain be besides usual medical measures? Discharge home with referral to the outpatients department Low molecular weight heparin Abciximab Thrombolysis with tissue plasminogen activator Urgent coronary angiography

Your answer

This patient falls into the high-risk category of acute coronary syndrome (also called ‘unstable angina’ or ‘myocardial infarction without ST-segment elevation’). Acute coronary syndrome is a medical emergency, which, if untreated, will progress to myocardial infarction in over 10% of cases. High-risk patients may have one or more of the following features: Prolonged, ongoing (> 20 min) rest pain

www.apnastudent.blogspot.com Pulmonary oedemaor angina with hypotension Rest angina with dynamicST changes > 1 mm Raised troponin (I and T) Age> 70 years Diabetes mellitus

of 179

6/30/2011 4:56 PM

Those at high-risk should proceed promptly to angiography with a view to proceeding to revascularisation, where appropriate, during that admission.

A 47-year-old man with chest pain of 1-hour duration is diagnosed as having acute myocardial infarction. Which of the following features, if present, would most contraindicate thrombolytic therapy? Blood pressure 160/110 mmHg History of likely ischaemic stroke within the past month ST-segment elevation in ECG Previous aspirin therapy Elevated serum cholesterol A 47-year-old man with chest pain of 1-hour duration is diagnosed as having acute myocardial infarction. Which of the following features, if present, would most contraindicate thrombolytic therapy? Blood pressure 160/110 mmHg History of likely ischaemic stroke within the past month

Your answer

ST-segment elevation in ECG Previous aspirin therapy Elevated serum cholesterol

There is an approximate 1% risk of stroke and a 0.7% risk of major haemorrhage associated with the use of thrombolysis. Recombinant tissue plasminogen activator (TPA) is connected with a lower rate of intracranial haemorrhage than streptokinase and hence is the drug of choice for thrombolysis in acute myocardial infarction of less than 12 hours’ duration. A list of the absolute contraindications to thrombolytic use is shown below; Aortic dissection Previous cerebral haemorrhage Known history of cerebral aneurysm or arteriovenous malformation Known intracranial neoplasm Recent (within the past 6 months) thromboembolic stroke Active internal bleeding (excluding menstruation) Patients previously treated with streptokinase or an isolated plasminogen streptokinase activator complex (APSAC or anistreplase) should receive recombinant tissue plasminogen activator, reteplase, or tenecteplase.

A 20-year-old woman presents with a history of dyspnoea on exertion. On examination she has a wide, fixed, split-second sound with an ejection systolic murmur in the left second intercostal space. Her ECG shows left axis deviation. What is the most probable diagnosis? Ostium primum septal defect Tricuspid incompetence Ostium secondum septal defect Pulmonary stenosis Aortic stenosis A 20-year-old woman presents with a history of dyspnoea on exertion. On examination she has a wide, fixed, split-second sound with an ejection systolic murmur in the left second intercostal space. Her ECG shows left axis deviation. What is the most probable diagnosis? Ostium primum septal defect Tricuspid incompetence

Correct answer

www.apnastudent.blogspot.com Ostium secondum septal defect

Your answer

Pulmonary stenosis

of 179

6/30/2011 4:56 PM

Aortic stenosis

Wide, fixed splitting of S 2 with an ejection systolic murmur in the left second intercostal space points to a diagnosis of atrial septal defect. Left axis deviation occurs in ostium primum atrial septal defect, whereas right axis deviation is seen in ostium secondum septal defect. The ejection systolic murmur is due to a large volume of blood passing through the pulmonary valves into the pulmonary artery. Aortic stenosis is associated with an ejection systolic murmur that is usually diamond-shaped (crescendo– decrescendo). There may be a systolic ejection click. Tricuspid incompetence presents with a blowing pansystolic murmur, best heard on inspiration at the lower left sternal edge. I n pulmonary stenosis, there is a harsh mid-systolic ejection murmur, best heard on inspiration to the left of the sternum in the second intercostal space.

A 42-year-old-man, known to be hypertensive, ran out of his medication two days ago. He presented to Casualty feeling short of breath and dizzy. His blood pressure on admission was 230/140 mmHg. Fundoscopy showed blurred disc margins. His chest revealed bibasal crepitations. With therapy, what blood pressure should you aim for in the next 1 hour in such cases? < 130/80 mm Hg < 140/90 mmHg Decrease in mean arterial pressure (MAP) by 50% Decrease in MAP by 25% Decrease in MAP by 75% A 42-year-old-man, known to be hypertensive, ran out of his medication two days ago. He presented to Casualty feeling short of breath and dizzy. His blood pressure on admission was 230/140 mmHg. Fundoscopy showed blurred disc margins. His chest revealed bibasal crepitations. With therapy, what blood pressure should you aim for in the next 1 hour in such cases? < 130/80 mm Hg < 140/90 mmHg Decrease in mean arterial pressure (MAP) by 50%

Your answer

Decrease in MAP by 25%

Correct answer

Decrease in MAP by 75%

The patient has a hypertensive emergency with markedly elevated blood pressure and evidence of target organ damage. This situation requires immediate attention to prevent disability or death. Here, the aim is to reduce the blood pressure promptly but partially to prevent end-organ damage without compromising tissue perfusion. The initial target is to lower the MAP by no more than 25%, or reduce the diastolic blood pressure by one-third. MAP = diastolic blood pressure + [(systolic BP – diastolic BP)/3]. Even in the presence of heart failure or hypertensive encephalopathy, a controlled reduction, to a level of about 150/90 mmHg, over a period of 24–36 hours is ideal. In most patients, blood pressure can be brought down with bed rest and oral medication. I ntravenous labetalol (2 mg/min to a maximum of 200 mg), intravenous glyceryl trinitrate (0.6–1.2 mg/h), intravenous sodium nitroprusside (0.3–1.0 mg/kg per min) or intramuscular hydralazine (5 or 10 mg repeated at half-hourly intervals) are all effective but require close monitoring.

A 67-year-old man is admitted with chronic congestive heart failure. Based on this history, what is the most important factor to be kept in mind when prescribing drugs for this patient? Loop diuretic administration would result in a decrease in mortality

www.apnastudent.blogspot.com Digoxin is more effective than ACE inhibitors in reducing cardiovascular events

of 179

6/30/2011 4:56 PM

Administration of a

β-blocker reduces the

time spent in hospital

Administration of spironolactone has no effect on the incidence of sudden cardiac death Angiotensin II -receptor antagonists have a better response rate than ACE inhibitors A 67-year-old man is admitted with chronic congestive heart failure. Based on this history, what is the most important factor to be kept in mind when prescribing drugs for this patient? Loop diuretic administration would result in a decrease in mortality Digoxin is more effective than ACE inhibitors in reducing cardiovascular events Administration of a

β-blocker reduces the

Your answer

time spent in hospital

Administration of spironolactone has no effect on the incidence of sudden cardiac death Angiotensin II -receptor antagonists have a better response rate than ACE inhibitors

Beta-adrenoceptor blocking agents (metoprolol, bisoprolol and carvedilol) have been found to be useful in patients with chronic stable heart failure. The studies MERI T and CIBIS 2, using the β-blockers metoprolol and bisoprolol, respectively, have shown improved symptomatic class, exercise tolerance, left ventricular function and reduced mortality in heart failure of any cause. The rapid decrease in symptoms reduces the time spent in hospital. Diuretic administration is associated with a rapid decrease in symptoms, but mortality rates are unchanged. Angiotensin-converting enzyme (ACE) inhibitors and diuretics are recommended in all patients with clinical heart failure as ACE inhibitors reduce mortality rates by 20%. Spironolactone greatly reduces the mortality and sudden cardiac death rates and should be added to the treatment. A recent trial comparing an angiotensin II-receptor antagonist (losartan) with an ACE inhibitor (enalapril) has shown no benefit of the former over the latter. Angiotensin II -receptor antagonists should be used when ACE inhibitors are contraindicated or cause side-effects (eg persistent cough).

During a routine medical check-up, a 2-year-old boy has been found to have a continuous machinery murmur on auscultation just below the left clavicle. Given the likely diagnosis, what would be the most characteristic investigative finding in this patient? Dilated left ventricle on echocardiogram Right ventricular hypertrophy on ECG Hilar haziness on chest X-ray Prominent pulmonary artery and pulmonary plethora on chest X-ray Polycythaemia During a routine medical check-up, a 2-year-old boy has been found to have a continuous machinery murmur on auscultation just below the left clavicle. Given the likely diagnosis, what would be the most characteristic investigative finding in this patient? Dilated left ventricle on echocardiogram

Correct answer

Right ventricular hypertrophy on ECG Hilar haziness on chest X-ray Prominent pulmonary artery and pulmonary plethora on chest X-ray

Your answer

Polycythaemia

This boy has a persistent ductus arteriosus. Because the aortic pressure exceeds the pulmonary artery pressure

www.apnastudent.blogspot.com throughout a cardiac cycle, a persistent ductus produces a continuous left to right shunting. This leads to increased pulmonary venous return to the left heart and an increased left ventricular volume load. The echocardiogram shows a dilated left atrium and left ventricle. Right heart changes are apparent in late disease.

of 179

6/30/2011 4:56 PM

Hilar haziness occurs in pulmonary oedema due to congestive cardiac failure. A prominent pulmonary artery may be seen on chest X-ray in persistent ductus, but the presence of pulmonary plethora is more suggestive of atrial septal defect. Polycythaemia may occur if the shunt is reversed (Eisenmenger’s syndrome).

A 3-month-old boy with a cyanotic heart lesion is found to have a patent ductus arteriosus (PDA). What is the best treatment for maintaining patency the PDA prior to surgery? Indometacin Surgical ligation Angiographic ligation of the pulmonary artery Prostaglandin E 1 administration No treatment A 3-month-old boy with a cyanotic heart lesion is found to have a patent ductus arteriosus (PDA). What is the best treatment for maintaining patency the PDA prior to surgery? Indometacin Surgical ligation Angiographic ligation of the pulmonary artery Prostaglandin E 1 administration

Your answer

No treatment

The ductus arteriosus in neonates and infants is highly sensitive to vasodilatation by PGE1. Patency of the ductus is necessary in patients with cyanotic heart disease until surgical correction of the heart problem is undertaken. This will ensure additional oxygenation of the blood. Administration of PGE 1 has been found to be highly effective in such cases.

An Asian boy with a known history of rheumatic heart disease presents with low-grade fever for the past month. He received a course of antibiotics from his GP a week ago. Which of the following investigations would be most useful in the diagnosis? Blood culture Serological testing Echocardiogram C-reactive protein Full blood count An Asian boy with a known history of rheumatic heart disease presents with low-grade fever for the past month. He received a course of antibiotics from his GP a week ago. Which of the following investigations would be most useful in the diagnosis? Blood culture Serological testing Echocardiogram C-reactive protein Full blood count

Your answer

www.apnastudent.blogspot.com

Echocardiography is extremely useful in allowing vegetations in infective endocarditis to be seen. Although blood cultures are a key diagnostic test in this condition, they may be negative if patients have recently received antibiotic therapy. The same reasoning applies to serological tests for Coxiella, Bartonella, Legionella, Chlamydia and Brucella spp that may also cause infective endocarditis. Both CRP and polymorphonuclear leucocytosis are non-specific tests.

A 64-year-old man with Wolff–Parkinson–White syndrome presents with uneasiness and palpitations. The ECG shows fine oscillations of the baseline and no clear P waves. The QRS rhythm is rapid and irregular. The ventricular rate is 120 beats per minute. His blood pressure is 90/60 mmHg. Which of the following interventions would be most appropriate in this case? Digoxin Verapamil DC Cardioversion Metoprolol Procainamide A 64-year-old man with Wolff–Parkinson–White syndrome presents with uneasiness and palpitations. The ECG shows fine oscillations of the baseline and no clear P waves. The QRS rhythm is rapid and irregular. The ventricular rate is 120 beats per minute. His blood pressure is 90/60 mmHg. Which of the following interventions would be most appropriate in this case? Digoxin Verapamil DC Cardioversion

Your answer

Metoprolol Procainamide

This patient most probably has atrial fibrillation superimposed on WPW syndrome. The aim of treatment is to suppress the conduction ability of the abnormal pathway. This is achieved by using class-I and -III antiarrhythmic drugs but not by verapamil and digoxin, which may allow a higher rate of conduction over the abnormal pathway and precipitate ventricular fibrillation. Thus neither verapamil nor digoxin should be used to treat atrial fibrillation associated with WPW syndrome. Previous guidelines suggested that use of adenosine was an acceptable option in these patients, but now D/C cardioversion in unstable situations is seen as the intervention of choice, with procainamide an alternative.

A 60-year-old man complains of dizziness and palpitations. An ECG shows tachycardia, broad QRS complexes, AV dissociation and the presence of capture beats. What is the most probable diagnosis? Sustained ventricular tachycardia Ventricular fibrillation Torsades de pointes Ventricular premature beats Atrial tachycardia A 60-year-old man complains of dizziness and palpitations. An ECG shows tachycardia, broad QRS complexes, AV dissociation and the presence of capture beats. What is the most probable diagnosis?

of 179

6/30/2011 4:56 PM

Sustained ventricular tachycardia

Correct answer

www.apnastudent.blogspot.com Ventricular fibrillation

Your answer

Torsades de pointes Ventricular premature beats Atrial tachycardia

The features are highly suggestive of sustained ventricular tachycardia. In ventricular fibrillation, there is very rapid and irregular ventricular activation with no mechanical effect. The patient is pulseless and rapidly becomes unconscious. The ECG shows shapeless rapid oscillations with no hint of organised complexes. In torsades de pointes, ventricular repolarisation is greatly prolonged (long QT syndrome). It is characterised on ECG by rapid, irregular, sharp complexes that continuously change from an upright to an inverted position. Prolonged QT intervals are also seen between spells of tachycardia or immediately preceding the onset of tachycardia. Broad QRS complexes may be seen in ventricular premature beats, but, following a premature beat, there is usually a compensatory pause. This condition is usually asymptomatic. In atrial tachycardia, the P waves are abnormally shaped and occur in front of the QRS complexes.

A 65-year-old man is admitted with a broad complex tachycardia. Which one of the following features would suggest a diagnosis of supraventricular tachycardia with aberrancy and help to exclude ventricular tachycardia? Capture beats on the electrocardiogram (ECG) Past history of ischaemic heart disease Right bundle branch block morphology with left axis deviation on the ECG Temporary alleviation by carotid sinus massage Variable intensity of the first heart sound A 65-year-old man is admitted with a broad complex tachycardia. Which one of the following features would suggest a diagnosis of supraventricular tachycardia with aberrancy and help to exclude ventricular tachycardia? Capture beats on the electrocardiogram (ECG) Past history of ischaemic heart disease Right bundle branch block morphology with left axis deviation on the ECG Temporary alleviation by carotid sinus massage

Your answer

Variable intensity of the first heart sound

Ventricular tachycardia (VT) may be distinguished from supraventricular tachycardia (SVT) by ECG features that indicate AV dissociation (ie that the atria and ventricles are no longer linked in rate and rhythm). The three characteristic features of AV dissociation are irregular notching of the QRS complex, capture beats and fusion beats. A variable intensity of the first heart sound in a regular tachycardia suggests AV dissociation causing variable filling of the ventricles from the atria. Atrial fibrillation is the commonest cause of variable intensity of the first heart sound. VT does not involve the AV node and cannot therefore be affected by adenosine or carotid sinus massage, which temporarily blocks the AV node. A past history of ischaemic heart disease is associated with a >95% chance that broad complex tachycardia is VT. A patient attending the cardiology clinic requires dental treatment. Which of the following conditions merit antibiotic prophylaxis? Atrial septal defect Hypertrophic cardiomyopathy Patent ductus arteriosus

of 179

6/30/2011 4:56 PM

All of the above None of the above

www.apnastudent.blogspot.com A patient attending the cardiology clinic requires dental treatment. Which of the following conditions merit antibiotic prophylaxis? Atrial septal defect Hypertrophic cardiomyopathy Patent ductus arteriosus All of the above None of the above

Your answer

Patent ductus arteriosus carries a high risk of endocarditis, but there is no robust evidence that antibiotic prophylaxis reduces the risk. The other ‘high-risk’ lesions are small ventricular septal defects and aortic regurgitation. The risk of endocarditis is highest where there are high-velocity jets of blood that damage the endothelium. Hypertrophic cardiomyopathy may be associated with high–velocity flow in the left ventricular outflow tract (LVOT) when there is marked LVOT obstruction, although, in practice, the risk of endocarditis is small. Atrial septal defects (ASDs) are large holes in a ‘low-pressure’ system and therefore carry a low risk of endocarditis and do not normally require prophylaxis. Mitral valve prolapse only carries appreciable risk where there is associated mitral regurgitation. Previous recommendations for antibiotic prophylaxis prior to dental procedures or instrumentation of the GI/GU tracts have been withdrawn as there is little evidence that these have been effective in preventing infection. Endocarditis may follow transient bacteraemia with organisms from oral or other mucosal flora, but this is much more likely to result from normal daily activity (chewing, brushing teeth) than from a visit to the dentist and antibiotic prophylaxis for such events is impractical and unnecessary. For the 2008 NI CE guidelines on the prevention of antibiotic prophylaxis see http:www.nice.org.uk/Guidance /CG64/Guidance/pdf/English

A 67-year-old lady, post-myocardial infarction, is suspected to have a left ventricular apical thrombus. Her neurological status has deteriorated and you want to exclude the need for thrombolysis. What is the most suitable imaging technique for confirming this diagnosis? Cardiac MR Left ventricular angiography Multiple uptake gated acquisition scanning Transoesophageal echocardiography Transthoracic echocardiography A 67-year-old lady, post-myocardial infarction, is suspected to have a left ventricular apical thrombus. Her neurological status has deteriorated and you want to exclude the need for thrombolysis. What is the most suitable imaging technique for confirming this diagnosis? Cardiac MR Left ventricular angiography Multiple uptake gated acquisition scanning

Your answer

Transoesophageal echocardiography Transthoracic echocardiography

Correct answer

Although an excellent technique for imaging the posterior cardiac structures (atria, left atrial appendage, valves and pulmonary veins), transoesophageal echo is less useful for imaging the structure and function of the ventricles, especially the left ventricular apex, which is better imaged by conventional two-dimensional (2D) echo. Atypical thrombus may be apparent on contrast left ventriculography, but this technique carries a risk of dislodging and embolising interventricular thrombus. Cardiac MR is effective in detecting mural thrombus but may not be easily available. A 30-year-old postman with hypertension but normally in good health presents to the emergency department with sudden severe breathlessness and sweating. Chest examination reveals bilateral basal crackles. He improves with diamorphine and

of 179

6/30/2011 4:56 PM

frusemide (furosemide). Electrocardiograms (ECGs) and cardiac enzymes are normal. He develops two further episodes of

www.apnastudent.blogspot.com pulmonary oedema which respond well to diuretics. The most likely cause of pulmonary oedema is? Dilated cardiomyopathy Myocarditis Ischaemic heart disease Phaeochromocytoma Renal artery stenosis A 30-year-old postman with hypertension but normally in good health presents to the emergency department with sudden severe breathlessness and sweating. Chest examination reveals bilateral basal crackles. He improves with diamorphine and frusemide (furosemide). Electrocardiograms (ECGs) and cardiac enzymes are normal. He develops two further episodes of pulmonary oedema which respond well to diuretics. The most likely cause of pulmonary oedema is? Dilated cardiomyopathy Myocarditis Ischaemic heart disease Phaeochromocytoma Renal artery stenosis

Your answer

The differential diagnosis of acute pulmonary oedema includes severe left ventricular (LV) dysfunction, paroxysmal arrhythmias, three-vessel or left main stem coronary disease and, in the context of hypertension, renal artery stenosis and phaeochromocytoma. Renal artery stenosis is more common. This man is a postman and therfore would be expected to have reasonable exercise tolerance, given his age this makes cardiomyopathy, myocarditis and ischaemic heart disease extremely unlikely. Renal artery stenosis in a 30-year-old is much more common than phaeochromocytoma.

A 30-year-old-man presents to the outpatient clinic with a 2-month history of progressive effort intolerance. Some three weeks ago he experienced an episode of shortness of breath at rest, suggestive of paroxysmal nocturnal dyspnoea. Examination reveals a JVP raised up to his earlobes, a soft tender hepatomegaly and a bilateral pitting oedema up to his knees. Chest examination reveals bibasal crepitations, and an audible S3 on auscultation of the heart. The chest X-ray shows cardiomegaly with interstitial infiltrates. Echocardiography shows global left ventricular hypokinesia with an ejection fraction of 25–30%. Which of the following is the LEAST likely aetiological factor? Alcohol abuse Genetic factor Adenovirus Eosinophilic states HIV infection A 30-year-old-man presents to the outpatient clinic with a 2-month history of progressive effort intolerance. Some three weeks ago he experienced an episode of shortness of breath at rest, suggestive of paroxysmal nocturnal dyspnoea. Examination reveals a JVP raised up to his earlobes, a soft tender hepatomegaly and a bilateral pitting oedema up to his knees. Chest examination reveals bibasal crepitations, and an audible S3 on auscultation of the heart. The chest X-ray shows cardiomegaly with interstitial infiltrates. Echocardiography shows global left ventricular hypokinesia with an ejection fraction of 25–30%. Which of the following is the LEAST likely aetiological factor? Alcohol abuse Genetic factor Adenovirus Eosinophilic states

of 179

Your answer

6/30/2011 4:56 PM

HIV infection

www.apnastudent.blogspot.com

Dilated cardiomyopathy encompasses a heterogeneous group of conditions. Alcohol abuse is an important aetiological factor in a significant number of patients. About 25% of cases are inherited as an autosomal-dominant trait. A substantial group is due to a late autoimmune reaction to viral myocarditis. Up to 10% of patients with advanced HI V infection develop dilated cardiomyopathy. Eosinophilic states are associated with obliterative cardiomyopathy.

A 46-year-old Asian man with a past history of coronary artery bypass grafting presents with breathlessness. The jugular venous pressure (JVP) shows prominent x and y descents. The most likely cause is? Constrictive pericarditis Dilated cardiomyopathy Pericardial effusion Restrictive cardiomyopathy Severe mitral regurgitation A 46-year-old Asian man with a past history of coronary artery bypass grafting presents with breathlessness. The jugular venous pressure (JVP) shows prominent x and y descents. The most likely cause is? Constrictive pericarditis

Correct answer

Dilated cardiomyopathy Pericardial effusion

Your answer

Restrictive cardiomyopathy Severe mitral regurgitation

A prominent x descent in the jugular venous pressure (JVP) may occur in constrictive pericarditis or pericardial effusion. The y descent is lost in tamponade but prominent in constrictive pericarditis. Constrictive pericarditis was classically caused by tuberculosis, but today is more commonly associated with cardiac surgery, renal failure or following infective pericarditis. Restrictive cardiomyopathy may produce clinical features similar to constriction, but is less common.

A 57-year-old man with ischaemic heart disease, and a recent transient ischaemic attack, is prescribed clopidogrel. How would the mechanism of action of this drug be best described? Blocks glycoprotein IIb/III a receptors Blocks thrombin receptors Blocks thromboxane production Blocks platelet ADP receptors Potentiates antithrombin-II I action A 57-year-old man with ischaemic heart disease, and a recent transient ischaemic attack, is prescribed clopidogrel. How would the mechanism of action of this drug be best described? Blocks glycoprotein IIb/III a receptors Blocks thrombin receptors Blocks thromboxane production Blocks platelet ADP receptors

Your answer

Potentiates antithrombin-II I action

of 179

6/30/2011 4:56 PM

Clopidogrel blocks platelet ADP receptors, while aspirin blocks thromboxane production, hence the complementary actions of the two drugs when given together following coronary stenting. The final common pathway for platelet aggregation is through the glycoprotein IIb/III a receptor. Hence, the most powerful antiplatelet drugs are the glycoprotein I Ib/II Ia blockers such as abciximab and tirofiban. Hirudins act by blocking thrombin receptors but have no current indication in cardiac disease.

A 47-year-old female patient attends the cardiology clinic for her symptoms of fatigue and ankle oedema. 2-D echocardiography shows diffuse ventricular wall thickening and marked dilatation of both atria, with granular sparkling of the left ventricular myocardium. She has been advised to avoid taking digoxin. Which of the following conditions is she most likely to have? Hypertrophic cardiomyopathy Dilated cardiomyopathy Restrictive cardiomyopathy Amyloid heart disease Constrictive pericarditis A 47-year-old female patient attends the cardiology clinic for her symptoms of fatigue and ankle oedema. 2-D echocardiography shows diffuse ventricular wall thickening and marked dilatation of both atria, with granular sparkling of the left ventricular myocardium. She has been advised to avoid taking digoxin. Which of the following conditions is she most likely to have? Hypertrophic cardiomyopathy Dilated cardiomyopathy Restrictive cardiomyopathy

Your answer

Amyloid heart disease

Correct answer

Constrictive pericarditis

This lady has cardiac amyloid, characterised by heart failure with a restrictive infiltrative pattern. Her echocardiogram appearance is typical of cardiac amyloid and excludes other conditions such as HOCM (in which digoxin is usually also contraindicated). Bradycardia with 2:1 or complete heart block is common in amyloidosis and is much more likely to occur with digoxin. In addition, the risk of digoxin toxicity is high in amyloidosis. Digoxin increases the risk of ventricular tachyarrhythmias in patients with impaired left ventricular (LV) function but this is not a definite contraindication.

A neonate is noted to be cyanosed within the 24 h following delivery. Which cardiac abnormality would be the most likely cause? Ebstein’s anomaly Eisenmenger ventricular septal defect Hypoplastic left heart Tetralogy of Fallot Transposition of the great vessels A neonate is noted to be cyanosed within the 24 h following delivery. Which cardiac abnormality would be the most likely cause? Ebstein’s anomaly Eisenmenger ventricular septal defect Hypoplastic left heart

www.apnastudent.blogspot.com Tetralogy of Fallot Transposition of the great vessels

Your answer

Although tetralogy of Fallot is much more common than transposition, the right ventricular (RV) outflow tract gradient, which is the major determinant of cyanosis in Fallot’s, does not become maximal until 6–9 months after birth. Many babies with Fallot’s are, therefore, pink at birth but gradually become cyanosed over the first few months of life. Ebstein’s anomaly is a congenital abnormality of the tricuspid valve associated with in utero exposure to lithium, which does not normally cause cyanosis.

A 49-year-old man is noted to have shortening of the QT interval on the ECG. Which drug is most likely to be responsible? Amiodarone Atenolol Digoxin Flecainide Sotalol A 49-year-old man is noted to have shortening of the QT interval on the ECG. Which drug is most likely to be responsible? Amiodarone Atenolol Digoxin Correct answer Flecainide Your answer Sotalol

The cardiac glycosides (digoxin and ouabain) shorten the QT interval. Class I a (eg disopyramide), class Ic (eg flecainide) and class II I drugs (eg amiodarone and sotalol) all prolong the QT interval. on the QT interval but are effective at stabilising the QT in long QT syndromes.

β-blockers have

a neutral effect

A 32-year-old lady is noted to have a loud first heart sound with reversed splitting of the second heart sound on auscultation. Which cardiological diagnosis is she most likely to have? Hypertrophic cardiomyopathy Left bundle branch block Mitral stenosis Right bundle branch block Wolff–Parkinson–White syndrome type B

A 32-year-old lady is noted to have a loud first heart sound with reversed splitting of the second heart sound on auscultation. Which cardiological diagnosis is she most likely to have?

of 179

6/30/2011 4:56 PM

Hypertrophic cardiomyopathy Left bundle branch block Mitral stenosis Right bundle branch block

Your answer

www.apnastudent.blogspot.com Wolff–Parkinson–White syndrome type B

Correct answer

A loud first heart sound (S1) is typical of mitral stenosis if the valve is pliable, but splitting of the second heart sound should be normal. The second heart sound (S2) is caused by closure of the aortic valve (A2) followed by closure of the pulmonary valve (P2). Reversed splitting of S2 is caused either by delayed A2 (eg left bundle branch block (LBBB), aortic stenosis, hypertrophic obstructive cardiomyopathy HOCM) or early P2 (eg Wolff–Parkinson–White (WPW) type B where the right–sided accessory pathway causes early RV depolarisation). Right bundle branch block (RBBB) causes wide splitting of S2 because it delays P2. S1 is soft if closure of the mitral valve is delayed (eg LBBB, long PR), but loud if mitral or tricuspid closure is early (eg WPW type B).

A 60-year-old woman is found to have a systolic murmur at a routine medical. When you see her she appears asymptomatic. Electrocardiography (ECG) shows marked left ventricular hypertrophy with strain. Echocardiography shows a peak aortic valve gradient of 90 mmHg, and decreased LV systolic function. What is the correct management? Aortic valvuloplasty Anticoagulation Regular out-patient review Routine aortic valve replacement Urgent aortic valve replacement

A 60-year-old woman is found to have a systolic murmur at a routine medical. When you see her she appears asymptomatic. Electrocardiography (ECG) shows marked left ventricular hypertrophy with strain. Echocardiography shows a peak aortic valve gradient of 90 mmHg, and decreased LV systolic function. What is the correct management? Aortic valvuloplasty Anticoagulation Regular out-patient review Routine aortic valve replacement Your answer Urgent aortic valve replacement Correct answer

Surgery for aortic valve replacement is indicated in symptomatic patients (angina, exertional breathlessness, syncope) as the risk of sudden death increases dramatically with the onset of symptoms, or, in those with severe asymptomatic disease (peak outflow gradient greater than around 50 mmHg). Patients with a gradient of less than 25 mmHg have a 20% chance of needing surgical intervention within 15 years. Valvuloplasty is used only in patients with critical aortic stenosis who are unfit for surgery as the benefits are usually short-lived.

A 58-year-old man is having his drug therapy reviewed following a myocardial infarction. Which of the following has no proven benefit on mortality following myocardial infarction (MI)? Atorvastatin Isosorbide mononitrate Ramipril Timolol Tirofiban A 58-year-old man is having his drug therapy reviewed following a myocardial infarction.

of 179

6/30/2011 4:56 PM

Which of the following has no proven benefit on mortality following myocardial infarction (MI)? Atorvastatin Isosorbide mononitrate

Your answer

www.apnastudent.blogspot.com Ramipril Timolol Tirofiban

The glycoprotein I Ib/I IIa antagonist tirofiban (PRISM-PLUS), timolol (TI MI trials) and ramipril (AIRE) have all been shown to reduce mortality following myocardial infarction. The recent MIRACL study showed that atorvastatin reduced cardiovascular events by 17% when given for three months post-MI . Isosorbide mononitrate showed no benefit in the ISIS 4 study.

A 70-year-old lady, who underwent mitral valve replacement surgery 2 years ago. She appeared to make a good recovery initially, but now presents with infective endocarditis. What causal organism would be most likely in her case? E. coli Staphylococcus aureus Staphylococcus epidermidis Streptococcus faecalis Streptococcus viridans A 70-year-old lady, who underwent mitral valve replacement surgery 2 years ago. She appeared to make a good recovery initially, but now presents with infective endocarditis. What causal organism would be most likely in her case? E. coli Staphylococcus aureus Staphylococcus epidermidis Streptococcus faecalis Streptococcus viridans

Your answer

Organisms causing infective endocarditis can be divided into three groups: native valve endocarditis, early prosthetic endocarditis (6 to12 months post-surgery). Native valve endocarditis is mainly streptococcal, predominantly streptococcus viridans, although staphylococcus aureus is also common and often affects previously normal valves. Early prosthetic endocarditis is most commonly due to staphylococcus epidermidis, with some gram-negative organisms and fungi. Late prosthetic endocarditis is similar to native valve endocarditis (ie most commonly streptococcus viridans) except with a higher incidence of staphylococcal infection.

The plateau phase of the myocardial action potential is mediated by? ATP-sensitive potassium current Delayed rectifier potassium current Fast sodium inward current L-type calcium current Slow calcium inward current The plateau phase of the myocardial action potential is mediated by? ATP-sensitive potassium current

of 179

6/30/2011 4:56 PM

Delayed rectifier potassium current Fast sodium inward current L-type calcium current

Your answer

www.apnastudent.blogspot.com Slow calcium inward current

Correct answer

Except in specialised regions of the heart such as the SA and AV nodes, the rapid upstroke of the cardiac action potential results from a fast sodium inward current generated by the opening of voltage-sensitive sodium channels. This current rapidly inactivates. The plateau of the action potential is maintained by (i) the slow calcium inward current, which activates at more depolarised potentials and inactivates more slowly than the fast sodium current, and (ii) an inward current generated by the sodium/calcium exchanger. Repolarisation is affected by outward potassium and chloride currents. The ATP-sensitive potassium channel mediates action potential shortening (and hence surface electrocardiogram (ECG) changes) during ischaemia. I t is the site of action of potassium channel openers such as nicorandil and has been implicated in the phenomenon of ischaemic preconditioning. Activation of the contractile myofilaments occurs as a result of calcium binding to troponin C. The calcium is partly derived from calcium entry via L-type calcium channels during the action potential and partly from calcium released from intracellular stores (chiefly the sarcoplasmic reticulum) in response to calcium entry.

A 35-year-old-woman gives a history of progressive exertional dyspnoea and fatigue over the last year. Examination reveals features of right-sided heart failure with pulmonary hypertension, but there are no crackles to suggest fibrosis. Pulmonary function testing rules out obstructive airways disease. Lung perfusion scanning and pulmonary angiography fail to detect pulmonary thromboembolic disease. An echocardiogram shows enlarged right heart chambers. ANCA testing is negative. What is the likely diagnosis? Pulmonary vasculitis Mitral valve prolapse Primary pulmonary hypertension Mitral stenosis Dilated cardiomyopathy

A 35-year-old-woman gives a history of progressive exertional dyspnoea and fatigue over the last year. Examination reveals features of right-sided heart failure with pulmonary hypertension, but there are no crackles to suggest fibrosis. Pulmonary function testing rules out obstructive airways disease. Lung perfusion scanning and pulmonary angiography fail to detect pulmonary thromboembolic disease. An echocardiogram shows enlarged right heart chambers. ANCA testing is negative. What is the likely diagnosis? Pulmonary vasculitis Mitral valve prolapse Primary pulmonary hypertension

Your answer

Mitral stenosis Dilated cardiomyopathy

Pulmonary hypertension can be subdivided into primary (idiopathic) and secondary types. Primary pulmonary hypertension (PPH) is a diagnosis of exclusion where no cause of pulmonary hypertension is discernible. A number of theories have been put forward to explain the origin of PPH; none has yet gained ascendancy. Secondary pulmonary hypertension can be caused by: vasoconstriction due to chronic hypoxia; loss of pulmonary vessels due to chronic obstructive pulmonary disease, pulmonary fibrosis, or vasculitis; pulmonary thromboembolic disease; congenital left-to-right shunting; left heart failure; and mitral valve disease. I nvestigations here largely rule out valve disease or significant cardiac dysfunction, making primary pulmonary hypertension the most likely diagnosis.

The first-line treatment for a 50-year-old man with known poor left ventricular function who presents with a

of 179

6/30/2011 4:56 PM

broad complex tachycardia at a rate of 150 beats/min (bpm) and a blood pressure of 120/70 mmHg is? Amiodarone

www.apnastudent.blogspot.com

β-blockers Flecainide Lidocaine Verapamil The first-line treatment for a 50-year-old man with known poor left ventricular function who presents with a broad complex tachycardia at a rate of 150 beats/min (bpm) and a blood pressure of 120/70 mmHg is? Amiodarone

Your answer

β-blockers Flecainide Lidocaine Verapamil

In the presence of poor left ventricular function, broad complex tachycardia is highly likely to be caused by ventricular tachycardia (VT). Verapamil may precipitate circulatory collapse in VT and is therefore contraindicated. In the presence of severe left ventricular (LV) dysfunction, negative inotropes such as lidocaine and β-blockers are undesirable while flecainide may rarely cause degeneration of stable tachycardias to ventricular fbrillation (VF). Amiodarone is, therefore, the most appropriate choice.

An 18-year-old man with Marfan’s syndrome is reviewed in the cardiology clinic after a screening ECG is found to be abnormal, with left axis deviation and prominent Q waves in I, I II, aVF and V3-V6. Which cardiac abnormality is most likely to be found? Aortic regurgitation Atrial septal defect Dilated cardiomyopathy Pulmonary regurgitation Persistent ductus arteriosus An 18-year-old man with Marfan’s syndrome is reviewed in the cardiology clinic after a screening ECG is found to be abnormal, with left axis deviation and prominent Q waves in I, I II, aVF and V3-V6. Which cardiac abnormality is most likely to be found? Aortic regurgitation

Correct answer

Atrial septal defect Dilated cardiomyopathy

Your answer

Pulmonary regurgitation Persistent ductus arteriosus

Marfan’s syndrome is characteristically associated with progressive aortic root dilatation leading to aortic regurgitation and an increased risk of dissection. Other skeletal manifestations include tall stature, scoliosis, chest wall malformations, high arched palate and lens dislocation. Mitral valve prolapse is also common but there is no association with other congenital malformations or cardiomyopathy.

A 67-year-old man with chronic heart failure is reviewed in terms of his drug therapy.

of 179

6/30/2011 4:56 PM

Which of the following treatments has no proven mortality benefit? Bisoprolol Digoxin

www.apnastudent.blogspot.com Enalapril Nitrates and hydralazine Spironolactone A 67-year-old man with chronic heart failure is reviewed in terms of his drug therapy. Which of the following treatments has no proven mortality benefit? Bisoprolol Digoxin

Your answer

Enalapril Nitrates and hydralazine Spironolactone

Bisoprolol (CIBI S II), spironolactone (RALES), enalapril (CONSENSUS) and nitrates and hydralazine (V-HEFT) have all been shown to improve mortality in chronic heart failure. Digoxin reduces the risk of death due to heart failure but overall cardiovascular mortality is similar to that on placebo, probably reflecting a small increase in the risk of arrhythmic death with digoxin therapy.

What is the most likely lipid abnormality in a 48-year-old Asian man with Type 2 diabetes who has good glycaemic control? Elevated high-density lipoprotein (HDL) Elevated low-density lipoprotein (LDL) Elevated LDL/elevated triglycerides Low HDL/elevated LDL Low HDL/elevated triglycerides What is the most likely lipid abnormality in a 48-year-old Asian man with Type 2 diabetes who has good glycaemic control? Elevated high-density lipoprotein (HDL) Elevated low-density lipoprotein (LDL) Elevated LDL/elevated triglycerides Low HDL/elevated LDL Low HDL/elevated triglycerides

Your answer

Asians do not have classical LDL–related risk for ischaemic heart disease. Their profile includes low HDL and elevated triglycerides, meaning that measurement of LDL alone may underestimate their risk.

A 72-year-old man presents with 15 min of central crushing chest pain. ECG shows 0.5 mm ST elevation in leads V1 and V2. You are in a peripheral hospital with no acute cardiac catheterisation lab. What is the most appropriate treatment? Accelerated tissue plasminogen activator (tPA) + aspirin Aspirin + heparin and repeat electrocardiogram (ECG) in 15 min Heparin only No treatment and repeat ECG in 15 min Streptokinase + aspirin

of 179

6/30/2011 4:56 PM

A 72-year-old man presents with 15 min of central crushing chest pain. ECG shows 0.5 mm ST elevation in leads V1 and V2. You are in a peripheral hospital with no acute cardiac catheterisation lab.

www.apnastudent.blogspot.com What is the most appropriate treatment? Accelerated tissue plasminogen activator (tPA) + aspirin Aspirin + heparin and repeat electrocardiogram (ECG) in 15 min

Correct answer

Heparin only No treatment and repeat ECG in 15 min

Your answer

Streptokinase + aspirin

The criteria for thrombolysis are ³1 mm ST elevation in two or more limb leads or ³2 mm ST elevation in adjacent chest leads, so thrombolysis is not indicated here. However, in the context of a good history of cardiac pain and borderline ECG, an acute coronary event should be strongly suspected and aspirin and heparin given prophylactically, with a repeat ECG in 15 min.

A 56-year-old man has known tricuspid regurgitation. Which part of the jugular venous waveform is likely to be most prominent? a wave c wave v wave x descent y descent A 56-year-old man has known tricuspid regurgitation. Which part of the jugular venous waveform is likely to be most prominent? a wave c wave v wave

Your answer

x descent y descent

Tricuspid regurgitation characteristically causes loss of the x descent in the JVP causing fusion of the c and v waves to produce ‘giant’ v waves, also known as ‘cv’ waves or ‘s’ waves.

A 50-year-old woman presents with an acute myocardial infarction, and thrombolysis is being considered. Which one of the following would be an absolute therapy?

contraindication for thrombolytic

Background diabetic retinopathy Past history of a minor stroke 5 years ago with full recovery and no evidence of underlying cerebrovascular lesion Menstruation Dyspeptic symptoms Resting blood pressure 220/130 mmHg A 50-year-old woman presents with an acute myocardial infarction, and thrombolysis is being considered.

of 179

6/30/2011 4:56 PM

Which one of the following would be an absolute therapy?

contraindication for thrombolytic

Background diabetic retinopathy Past history of a minor stroke 5 years ago with full recovery and no evidence of underlying cerebrovascular lesion

Your answer

www.apnastudent.blogspot.com Menstruation Dyspeptic symptoms Resting blood pressure 220/130 mmHg

Correct answer

A proven (i.e. computed tomography (CT) scan-proven) intracranial haemorrhage (ICH) is an absolute contraindication to thrombolysis. A fully recovered previous TIA would not prove an absolute contraindication. There remains however some debate over the risk of ICH in the context of a previous stroke. In the American College of Cardiology guidelines (2004), a history of ischaemic stroke within 3 months is given as an absolute contraindication, as is any history of intracranial haemorrhage. Contraindications consistent with the NI CE guidelines 2007 are detailed below. • Normal ECG or ECG not available • Unconsciousness without known cause • Cerebrovascular accident within the last six months • Possible aortic dissection • Known intracranial pathology • Active pulmonary disease with cavitation • Severe hypertension with blood pressure above 180/110 mmHg • Coagulation defects, bleeding diatheses or anticoagulants • Surgery (including dental extraction), major trauma, dental extraction or haemorrhage (including heavy vaginal bleeding) within the last two weeks • Acute gastrointestinal bleeding, recent peptic ulcer, acute pancreatitis, oesophageal varices or severe liver disease • Laser therapy for retinopathy within the last week • Infective endocarditis, pericarditis • Pregnancy or up to 18 weeks postnatal • Prolonged or traumatic CPR • In the case of streptokinase, previous allergic reactions to either streptokinase or anistreplase (no longer available) • Prolonged persistence of antibodies to streptokinase may reduce the effectiveness of subsequent treatment, therefore streptokinase should not be used if recently used 5 days to 12 months previously.

A 56-year-old lady has a known ventricular septal defect. Which of the following clinical signs would most indicate the presence of established pulmonary hypertension? Loud systolic murmur Raised jugular venous pressure (JVP) Single loud second heart sound Systolic thrill Displaced apex beat A 56-year-old lady has a known ventricular septal defect. Which of the following clinical signs would most indicate the presence of established pulmonary hypertension? Loud systolic murmur Raised jugular venous pressure (JVP)

Your answer

Single loud second heart sound Systolic thrill Displaced apex beat

The systolic murmur and thrill of the ventricular septal defect (VSDs) are absent once Eisenmenger's complex has developed. Under these circumstances there are just signs of pulmonary hypertension and cyanosis. Cardiomegaly may occur with Eisenmenger's complex because of right venticular (RV) enlargement but the left ventricle (LV) is not usually significantly enlarged and the apex beat is not typically displaced. A single second heart sound is characteristically

of 179

6/30/2011 4:56 PM

associated with Fallot’s tetralogy where pulmonary stenosis protects from developing pulmonary hypertension.

A 45-year-old man with a strong family history of ischaemic heart disease presents with atypical chest pains. Electrocardiographic (ECG) exercise testing shows J point depression of 1 mm with a heart rate of 120 beats/min (bpm).

www.apnastudent.blogspot.com What is the most appropriate next step? Coronary angiography Dobutamine stress echocardiography Radionuclide myocardial perfusion scanning Reassure and discharge Repeat ECG exercise testing on anti-anginal medication A 45-year-old man with a strong family history of ischaemic heart disease presents with atypical chest pains. Electrocardiographic (ECG) exercise testing shows J point depression of 1 mm with a heart rate of 120 beats/min (bpm). What is the most appropriate next step? Coronary angiography Dobutamine stress echocardiography Radionuclide myocardial perfusion scanning Reassure and discharge Repeat ECG exercise testing on anti-anginal medication

Correct answer Your answer

J point depression is a physiological response to an increase in heart rate and ST segments should, therefore, be measured 80 ms post-J. J point depression produces upward sloping ST depression which has little predictive value for coronary artery disease. Horizontal and downward sloping segments are more predictive than upward sloping ones. In a young man with atypical chest pain, this exercise test would be reassuring and there would be no indication for further investigation or treatment.

A 56-year-old man presents with a cardiac rhythm disorder. Which one of the following scenarios would be an indication for temporary transvenous cardiac pacemaker insertion? Asymptomatic 2.8 s sinus pauses A short period of complete heart block complicating inferior myocardial infarction, (pre-thrombolysis) with blood pressure 110/70 mmHg Asymptomatic complete heart block with broad complex ventricular complexes at 35 bpm Mobitz I I AV block complicating anterior myocardial infarction with blood pressure 110/70 mmHg Bifascicular block prior to aortic aneurysm repair A 56-year-old man presents with a cardiac rhythm disorder. Which one of the following scenarios would be an indication for temporary transvenous cardiac pacemaker insertion? Asymptomatic 2.8 s sinus pauses A short period of complete heart block complicating inferior myocardial infarction, (pre-thrombolysis) with blood pressure 110/70 mmHg Asymptomatic complete heart block with broad complex ventricular complexes at 35 bpm

of 179

Mobitz I I AV block complicating anterior myocardial infarction with blood pressure 110/70 mmHg

Correct answer

Bifascicular block prior to aortic aneurysm repair

Your answer

6/30/2011 4:56 PM

Asymptomatic sinus node disease, even with prolonged pauses, carries a low risk of sudden death and can usually be managed without a temporary wire. Chronic complete heart block has a higher risk but if asymptomatic and with a stable escape rhythm can also be managed in most cases without a temporary wire. Bifascicular block carries a slightly higher risk of high–grade AV block but, in asysmptomatic patients, this risk is sufficiently low to obviate the need for perioperative pacing. In myocardial infarction, AV block and even complicating MI should be managed conservatively if asymptomatic and causing no haemodynamic compromise. However, second- or third-degree heart block complicating anterior MI requires pacing.

A 78-year-old-man presents to Casualty with a history of syncope. An ECG shows complete heart block. Which of the following physical signs is consistent with the diagnosis? Cannon ‘a’ waves on JVP at regular intervals Soft first heart sound Low-volume pulse Basal systolic murmur Loud second heart sound A 78-year-old-man presents to Casualty with a history of syncope. An ECG shows complete heart block. Which of the following physical signs is consistent with the diagnosis? Cannon ‘a’ waves on JVP at regular intervals Soft first heart sound Low-volume pulse Basal systolic murmur Loud second heart sound

Correct answer

Your answer

Complete heart block produces a slow regular pulse (25–50/min) that doesn’t vary with exercise. Usually, there is a compensatory increase in stroke volume with a large-volume pulse and systolic flow murmurs. Cannon ‘a’ waves are irregularly seen, and the intensity of the first and second heart sound varies due to the loss of atrioventricular synchrony.

A 40-year-old man is noted to have palmar crease xanthomas. Which form of lipid disorder is most likely? Familial hypercholesterolaemia Familial mixed hypercholesterolaemia Hyperchylomicronaemia Familial hypertriglyceridaemia Broad b disease A 40-year-old man is noted to have palmar crease xanthomas. Which form of lipid disorder is most likely? Familial hypercholesterolaemia Familial mixed hypercholesterolaemia Hyperchylomicronaemia

www.apnastudent.blogspot.com Familial hypertriglyceridaemia Broad b disease Correct answer

of 179

Your answer

6/30/2011 4:56 PM

Palmar crease xanthomas are pathognomonic of type II I hyperlipidaemia also known as broad b-disease. Familial hypercholesterolaemia (type IIa hyperlipidaemia) has high total cholesterol and low-density lipoprotein (LDL) values due to impaired LDL receptor activity. It is associated with severe atherosclerosis and premature coronary heart disease, xanthomas, xanthelasmata and corneal arcus. Hyperchylomicronaemia is type I hyperlipidaemia due to lipoprotein lipase enzyme deficiency. Familial hypertriglyceridaemia is a form of type I V hyperlipidaemia.

A 15-year-old patient undergoes echocardiography, which shows a right-sided aortic arch. Which cardiac condition is most likely? Coarctation of the aorta Ebstein’s anomaly Tetralogy of Fallot Hypoplastic left ventricle Noonan’s syndrome A 15-year-old patient undergoes echocardiography, which shows a right-sided aortic arch. Which cardiac condition is most likely? Coarctation of the aorta

Your answer

Ebstein’s anomaly Tetralogy of Fallot

Correct answer

Hypoplastic left ventricle Noonan’s syndrome

Approximately 25% of patients with Fallot’s tetralogy have a right–sided aortic arch. Coarctation is associated with bicuspid aortic valve and Noonan’s syndrome with pulmonary stenosis. Ebstein’s anomaly is a congenital abnormality of the tricuspid valve which has an association with right–sided accessory pathways.

A 36-year-old old woman presents with a cerebral infarct following treatment for a deep vein thrombosis. Cardiovascular examination is entirely normal. The most likely underlying cardiac abnormality is? Partial anomalous pulmonary venous drainage Ostium primum atrial septal defect Ostium secundum Common atrium Patent foramen ovale A 36-year-old old woman presents with a cerebral infarct following treatment for a deep vein thrombosis. Cardiovascular examination is entirely normal. The most likely underlying cardiac abnormality is? Partial anomalous pulmonary venous drainage Ostium primum atrial septal defect Ostium secundum Common atrium Patent foramen ovale

Your answer

www.apnastudent.blogspot.com

The incidence and importance of patent foramen ovale (PFO) remain controversial but up to 25% of people have a PFO which may allow passage of a thrombus from the venous to systemic circulation when the right heart pressures are increased – characteristically with Valsalva or following a pulmonary embolus. PFOs are not associated with clinical signs and cannot normally be identified on transthoracic echo. Use of agitated saline contrast during echo is helpful in identifying PFOs. Other types of ASDs are much less common than PFOs and abnormal clinical signs are usually present. Partial anomalous pulmonary venous drainage means that between one and three pulmonary veins open into the right atrium rather than the left atrium. There is no increased risk of right-to-left shunting.

A 70-year-old man undergoes successful DC cardioversion for atrial fibrillation (AF). Which one of the following factors best predicts long-term maintenance of sinus rhythm following this procedure? Age under 75 years Normal left ventricular function Warfarin therapy No alcohol intake AF duration less than 6 months prior to cardioversion A 70-year-old man undergoes successful DC cardioversion for atrial fibrillation (AF). Which one of the following factors best predicts long-term maintenance of sinus rhythm following this procedure? Age under 75 years Normal left ventricular function Warfarin therapy No alcohol intake AF duration less than 6 months prior to cardioversion

Your answer

Cardioversion has a much higher success rate in patients with structurally normal hearts but the left atrial size is a better predictor than left ventricular function. AF is likely to be persistent where the left atrial dimension is >5 cm. Age is much less important than the duration of AF. The success of cardioversion drops off significantly after 6 months of persistent AF and long-term sinus rhythm is unlikely to be restored if AF has been persistent for more than 12 months. Alcohol is an important aetiological factor but less significant than AF duration. Warfarin is important to reduce stroke risk but does not help to restore or maintain sinus rhythm.

A 60-year-old lady has evidence of left ventricular dysfunction. Which of the following causes an increase in end–diastolic left ventricular dimension? Patent ductus arteriosus Hypertrophic cardiomyopathy Severe mitral regurgitation Pericardial effusion Mitral stenosis A 60-year-old lady has evidence of left ventricular dysfunction. Which of the following causes an increase in end–diastolic left ventricular dimension? Patent ductus arteriosus

of 179

6/30/2011 4:56 PM

Hypertrophic cardiomyopathy Severe mitral regurgitation

Your answer

www.apnastudent.blogspot.com Pericardial effusion Mitral stenosis

Increased left ventricular dimensions are characteristic in dilated cardiomyopathy. Any other condition which causes increased left ventricular work (eg to overcome left ventricular outflow obstruction in aortic stenosis) or increased left ventricular end-diastolic pressure (eg in end-stage mitral regurgitation) will eventually cause LV dilatation. In hypertrophic cardiomyopathy, severe left ventricular hypertrophy prevents significant increase in intra-cavity dimensions such as LV end-diastolic dimensions. In pericardial effusion, pericardial fluid prevents LV dilatation by external compression. Mitral stenosis and patent ductus arteriosus do not affect left ventricular work.

A 38-year-old man of Chinese descent who smokes 60 cigarettes per day presents to his GP. He is developing pain at rest in his legs, and is unable to walk more than a few yards due to ischaemic pain. On examination there is prolonged capillary refill and necrotic ulcers at the tips of his toes. There is also evidence of thrombophlebitis. What diagnosis fits best with this clinical picture? Buerger’s disease Simple peripheral vascular disease Polyarteritis nodosa Familial hypercholesterolaemia Temporal arteritis A 38-year-old man of Chinese descent who smokes 60 cigarettes per day presents to his GP. He is developing pain at rest in his legs, and is unable to walk more than a few yards due to ischaemic pain. On examination there is prolonged capillary refill and necrotic ulcers at the tips of his toes. There is also evidence of thrombophlebitis. What diagnosis fits best with this clinical picture? Buerger’s disease

Your answer

Simple peripheral vascular disease Polyarteritis nodosa Familial hypercholesterolaemia Temporal arteritis

Buerger’s disease (thromboangiitis obliterans) is an occlusive inflammatory disease of small- to medium-sized arteries of the upper and lower extremities. Histopathology examination of affected arteries reveals fresh inflammatory thrombus within both small- and medium-sized arteries and veins, with giant cells surrounding the thrombus. The disease is very closely associated with heavy smoking; continued smoking after diagnosis invariably leads to a poor outlook, gangrene and multiple amputations. Prevalence is higher in men and people of Far-Eastern origin. The main goal of therapy is elimination of tobacco smoking. Bypass surgery is of variable success due to the distal nature of the occlusions. Sympathectomy may be useful in increasing distal blood flow and relieving pain. Amputation of gangrenous digits is frequently required.

A 42-year-old painter presents to A&E with symptoms of vertigo, diplopia and gait unsteadiness at the end of a busy afternoon painting the interior of a property. On examination there is a markedly lower blood pressure in the left arm. What diagnosis fits best with this clinical picture? Anterior circulation transient ischaemic attack Subclavian steal syndrome Vestibular neuronitis

of 179

6/30/2011 4:56 PM

Posterior circulation transient ischaemic attack Unexplained cardiac arrhythmia A 42-year-old painter presents to A&E with symptoms of vertigo, diplopia and gait unsteadiness at the end of a busy

www.apnastudent.blogspot.com afternoon painting the interior of a property. On examination there is a markedly lower blood pressure in the left arm. What diagnosis fits best with this clinical picture? Anterior circulation transient ischaemic attack Subclavian steal syndrome

Your answer

Vestibular neuronitis Posterior circulation transient ischaemic attack Unexplained cardiac arrhythmia

Subclavian steal syndrome results from occlusion or stenosis of the proximal subclavian artery, leading to decreased antegrade or retrograde flow in the ipsilateral vertebral artery. Subclavian steal produces symptoms by flow-related phenomena rather than embolic. When an atherosclerotic lesion in the proximal subclavian artery progresses to cause hemodynamically significant stenosis, collateral vessels from the subclavian artery gradually enlarge. The upper extremity becomes dependent on these large collateral blood vessels that originate from the subclavian artery distal to the obstruction. The collateral vessels serve as points of reentry for blood flowing retrograde into the arm from the head, shoulder, and neck, thereby providing the extremity with adequate perfusion. When the arm is exercised, the blood vessels dilate to enhance perfusion to the ischemic muscle, thus lowering the resistance in the outflow vessels. Blood is siphoned from the head, neck, and shoulder through collateral vessels to supply this low-resistance vascular bed, satisfying increased oxygen demand by the exercising muscles of the upper extremity. This results in posterior cerebral circulation neurological symptoms. Many patients are asymptomatic, upper extremity symptoms include fatigue, aching, coolness of the affected arm and some numbness. Neurological symptoms may include vertigo, diplopia, decreased vision, nystagmus and gait unsteadiness and occur in around 25% of patients. Symptoms may be precipitated by extreme exercise on the affected side such as cricket bowling, use of an underarm crutch or painting a wall. Appropriate imaging studies include non-invasive arterial flow studies, Doppler and arteriography. Most patients require no intervention, although surgical reconstruction may be required where symptoms are severe.

A 64-year-old woman presents with an episode of syncope while out shopping. On more direct questioning she also reports a few episodes of fast palpitations, which she is able to tap out on her hand. She is discharged to await an outpatient 24-h ECG Holter recording. Unfortunately she is readmitted after suffering a fit while in bed, her husband felt her pulse at the time and claims that she was pulseless for a few seconds. What diagnosis fits best with this clinical picture? Paroxysmal atrial fibrillation Atrial flutter Sick-sinus syndrome Multiple transient ischaemic attacks Epilepsy

A 64-year-old woman presents with an episode of syncope while out shopping. On more direct questioning she also reports a few episodes of fast palpitations, which she is able to tap out on her hand. She is discharged to await an outpatient 24-h ECG Holter recording. Unfortunately she is readmitted after suffering a fit while in bed, her husband felt her pulse at the time and claims that she was pulseless for a few seconds. What diagnosis fits best with this clinical picture? Paroxysmal atrial fibrillation Atrial flutter Sick-sinus syndrome

Your answer

Multiple transient ischaemic attacks Epilepsy

of 179

6/30/2011 4:56 PM

Sick-sinus syndrome is characterised by periods of sinus bradycardia, sinus arrest, a combination of sinoatrial or atrioventricular conduction defects and supraventricular tachycardias. In adults the disease is often associated with

www.apnastudent.blogspot.com atherosclerosis, but may occur in the presence of a normal heart. Presentation may be with light-headedness, syncope or palpitations, or patients may present with stroke associated with atrial fibrillation. The cause is fibrosis or fatty infiltration of the sinus node, atrioventricular node, His bundle, or its branches. Work-up includes ECG and ambulatory cardiac rhythm monitoring. A permanent pacemaker is the treatment of choice. Drug treatment of tachyarrhythmias may result in an increased risk of heart block.

A 17-year-old youth is brought to the GP by his mother. He was previously seen 2 weeks earlier suffering from acute pharyngitis. His teeth are in generally poor condition, but otherwise there is no previous medical history. On examination he is febrile with a temperature of 38.2 °C, and has a polyarthritis affecting his knees, ankles, wrists and elbows. He also appears to have subcutaneous nodules over his elbows, and mitral regurgitation on cardiovascular examination. What diagnosis fits best with this clinical picture? Bacterial endocarditis Juvenile rheumatoid arthritis Scarlet fever Rheumatic fever Congenital valvular heart disease

A 17-year-old youth is brought to the GP by his mother. He was previously seen 2 weeks earlier suffering from acute pharyngitis. His teeth are in generally poor condition, but otherwise there is no previous medical history. On examination he is febrile with a temperature of 38.2 °C, and has a polyarthritis affecting his knees, ankles, wrists and elbows. He also appears to have subcutaneous nodules over his elbows, and mitral regurgitation on cardiovascular examination. What diagnosis fits best with this clinical picture? Bacterial endocarditis Juvenile rheumatoid arthritis Scarlet fever Rheumatic fever

Your answer

Congenital valvular heart disease

This patient’s clinical condition is highly suggestive of rheumatic fever. Physical findings suggestive of rheumatic fever include the history of previous pharyngitis, fever, polyarthritis, carditis (including the mitral regurgitation murmur) and the presence of subcutaneous extensor surface nodules. Laboratory testing suggestive of the diagnosis would include a positive anti-streptolysin O titre (peaks at 4–5 weeks after a streptococcal throat infection), raised erythrocyte sedimentation rate (ESR), C-reactive protein (CRP) and a leucocytosis is also suggestive of rheumatic fever. Acute treatment includes a course of penicillin to eradicate throat carriage of group A streptococci; where there is carditis or arthritis, aspirin or prednisolone may be added, but specialist advice is advised. Erythromycin may be used in penicillin-allergic patients.

A 65-year-old woman with a history of heavy smoking presents for review. She has woken during the early hours of the morning for the second time with shortness of breath so bad that she had to fling open the windows. On examination there are crackles in the lung bases, her chest X-ray shows bilateral fluffy perihilar shadowing. ECG reveals small anterior Q waves and a sinus tachycardia of 105 bpm. What diagnosis fits best with this clinical picture? Cryptogenic fibrosing alveolitis Pulmonary embolus Exacerbation of COPD Sarcoidosis Pulmonary oedema

of 179

6/30/2011 4:56 PM

A 65-year-old woman with a history of heavy smoking presents for review. She has woken during the early hours of the morning for the second time with shortness of breath so bad that she had to fling open the windows. On examination there are crackles in the lung bases, her chest X-ray shows bilateral fluffy perihilar shadowing. ECG reveals small anterior Q waves

www.apnastudent.blogspot.com and a sinus tachycardia of 105 bpm. What diagnosis fits best with this clinical picture? Cryptogenic fibrosing alveolitis Pulmonary embolus Exacerbation of COPD Sarcoidosis Pulmonary oedema

Your answer

The history of paroxysmal nocturnal dyspnoea, chest X-ray suggestive of pulmonary oedema and ECG with changes of a previous anterior myocardial infarction suggests that this woman is suffering from left ventricular failure. A history of pink frothy sputum and distended neck veins on examination would also contribute to the diagnosis. Causes of pulmonary oedema include acute myocardial infarction, hypertensive heart failure, valvular disease, ventricular septal defect, cardiac tamponade, cardiac arrhythmias, endocarditis, myocarditis and cardiomyopathy. Echocardiography is useful to determine the differential diagnoses, and provides information about valvular disease, diastolic vs systolic dysfunction, ejection fraction and estimates of right-sided pressures. The acute management of pulmonary oedema includes oxygen therapy, intravenous furosemide and vasodilator therapy with iv nitrates. Many acute wards also have intermittent positive-pressure ventilation available, a useful adjunct to medical therapy for left ventricular failure.

Which of the following statements is true regarding pulsus alternans? It is found in beriberi heart disease The pulse is irregular It is diagnosed electrocardiographically It is found in association with a third heart sound It is found in patients with a small pericardial effusion Which of the following statements is true regarding pulsus alternans? It is found in beriberi heart disease The pulse is irregular It is diagnosed electrocardiographically It is found in association with a third heart sound

Your answer

It is found in patients with a small pericardial effusion

Pulsus alternans is found in patients with acute left ventricular failure. Alternate weak and strong pulses occur, which are regularly placed. A pathological third heart sound is usually associated.The condition may be associated with heart failure or pericarditis for example, but is not usually seen in association with pericardial effusion. Beri-Beri heart disease leads to long QT, T wave inversion and low voltage complexes. Electrical alternans is diagnosed electrocardiographically. The amplitude of QRS complexes varies alternately. I t occurs due to changes in electrical depolarisation, conduction abnormalities or cardiac motion.

An 18-year-old student who has never been vaccinated against measles presents to his GP with symptoms suggestive of the disease. He is sent home and advised to rest, but later presents to A&E with anterior chest pain that is worse on inspiration and relieved by sitting forward. On examination there appears to be a rub on auscultation. What diagnosis fits best with this clinical picture?

of 179

6/30/2011 4:56 PM

Viral pleurisy Pericarditis Myocardial ischaemia

www.apnastudent.blogspot.com Pneumothorax Secondary bacterial pneumonia An 18-year-old student who has never been vaccinated against measles presents to his GP with symptoms suggestive of the disease. He is sent home and advised to rest, but later presents to A&E with anterior chest pain that is worse on inspiration and relieved by sitting forward. On examination there appears to be a rub on auscultation. What diagnosis fits best with this clinical picture? Viral pleurisy Pericarditis

Your answer

Myocardial ischaemia Pneumothorax Secondary bacterial pneumonia

Pericarditis presents with anterior pleuritic chest pain, worse on inspiration and relieved by sitting forward. I t is associated with a pericardial friction rub, which is best heard when the patient is upright and leaning forward. There may be associated cardiac tamponade, evidenced by tachycardia, low blood and pulse pressure and distended neck veins. Pericarditis may be infectious in origin (viral, bacterial or fungal), inflammatory (eg rheumatoid, related to systemic lupus erythematosus (SLE), scleroderma or vasculitis), drug-induced, myocardial infarction-related, postradiotherapy, uraemic, neoplastic, related to sarcoid, or to a host of other causes. For viral pericarditis, as in this case, limitation of activity is advised with additional pain relief using non-steroidals and opiate-based agents such as codeine phosphate. I n severe cases, oral prednisolone may be considered. Prognosis varies according to the underlying cause, but recurrence of pericarditis occurs in 10–15% of patients with pericarditis within the first year.

A 62-year-old man presents to A&E with persistent indigestion-like pain, retrosternal in nature and radiating to his jaw. He is a heavy smoker of some 40 cigarettes per day and has been previously treated for Barrett’s oesophagus. On examination he is bradycardic at 55 bpm, has a blood pressure of 100/50 mmHg and looks unwell. ECG reveals ST elevation in leads II , II I and aVF. What diagnosis fits best with this clinical picture? Acute anterior myocardial infarction Non-Q wave myocardial infarction Inferior myocardial infarction Recurrence of Barrett’s oesophagus Unstable angina A 62-year-old man presents to A&E with persistent indigestion-like pain, retrosternal in nature and radiating to his jaw. He is a heavy smoker of some 40 cigarettes per day and has been previously treated for Barrett’s oesophagus. On examination he is bradycardic at 55 bpm, has a blood pressure of 100/50 mmHg and looks unwell. ECG reveals ST elevation in leads II , II I and aVF. What diagnosis fits best with this clinical picture? Acute anterior myocardial infarction Non-Q wave myocardial infarction Inferior myocardial infarction

Your answer

Recurrence of Barrett’s oesophagus Unstable angina

This man has symptoms and signs of an acute myocardial infarction; changes in the inferior leads on ECG testing confirm the diagnosis. Myocardial infarction is said to characteristically present with central crushing chest pain

of 179

6/30/2011 4:56 PM

radiating to the arms and jaw that is not pleuritic in character. However, it is important to note that large infarcts may

www.apnastudent.blogspot.com be associated with a less typical pain distribution, and up to 20% of myocardial infarctions may not be associated with pain. Painless infarcts are commoner in the elderly and in patients with diabetes mellitus. Acute treatment involves oxygen therapy, nitrates, analgesia (including opiates and aspirin) and thrombolytic therapy. Early postinfarct treatment includes the introduction of b-blockade and angiotensin-converting enzyme (ACE) inhibition. Persistent pain and/or persistent ECG changes after thrombolysis should be discussed with the local cardiac surgery unit, ie for consideration of angiography with or without stenting.

A 43-year-old man presents with acute central chest pain radiating to his back. He is pale, sweaty and looks extremely unwell. The admitting nurse notices that his blood pressures appear to be unequal when comparing the left and right arms. Chest auscultation reveals aortic regurgitation, and on the ECG the complexes look small. What diagnosis fits best with this clinical picture? Acute myocardial infarction Pericarditis Subacute bacterial endocarditis Aortic dissection Aortic regurgitation

A 43-year-old man presents with acute central chest pain radiating to his back. He is pale, sweaty and looks extremely unwell. The admitting nurse notices that his blood pressures appear to be unequal when comparing the left and right arms. Chest auscultation reveals aortic regurgitation, and on the ECG the complexes look small. What diagnosis fits best with this clinical picture? Acute myocardial infarction Pericarditis Subacute bacterial endocarditis Aortic dissection

Your answer

Aortic regurgitation

Aortic dissection may present with hypertension or hypotension, unequal or absent pulses, aortic regurgitation, neurological abnormalities caused by vascular occlusion, Horner’s syndrome due to mass effect, or cardiac tamponade caused by dissection into the pericardial sac. Predisposing factors include hypertension, Marfan’s syndrome, congenital aortic valve abnormalities and syphilis infection. ECG may show left ventricular hypertrophy, or small complexes suggestive of cardiac tamponade. Chest X-ray may show mediastinal widening, but anteroposterior (AP) projection films overestimate the width of the mediastinum anyway. Dissections are generally classified into type-A dissections (involving the ascending aorta), and type-B dissections (involving the descending aorta). Type-A dissections usually require surgical intervention, although type-B dissections may be managed medically. Patients should be managed on the intensive care unit with aggressive management of blood pressure. For untreated patients, the mortality rate is said to approach 85%; even treated patients still have a mortality rate approaching 20%.

A 38-year-old man presents for review. His only previous history of note has been recurrent shoulder subluxation. His main complaints are tiredness and increasing dyspnoea on exertion. The nursing clerking on admission notes that he seems very tall and thin, his height is described as 1.93 m (6ft 4 inches). On examination his blood pressure is 165/70 mmHg, he has left ventricular hypertrophy, a low-pitched apical diastolic murmur and an early systolic apical ejection murmur. What diagnosis fits best with this clinical picture? Mitral stenosis Aortic regurgitation Mitral valve prolapse Aortic stenosis Infective endocarditis

of 179

6/30/2011 4:56 PM

A 38-year-old man presents for review. His only previous history of note has been recurrent shoulder subluxation. His main complaints are tiredness and increasing dyspnoea on exertion. The nursing clerking on admission notes that he seems very

www.apnastudent.blogspot.com tall and thin, his height is described as 1.93 m (6ft 4 inches). On examination his blood pressure is 165/70 mmHg, he has left ventricular hypertrophy, a low-pitched apical diastolic murmur and an early systolic apical ejection murmur. What diagnosis fits best with this clinical picture? Mitral stenosis Aortic regurgitation

Your answer

Mitral valve prolapse Aortic stenosis Infective endocarditis

This man has a marfanoid habitus and is at risk of suffering aortic regurgitation. Aetiological factors involved in aortic regurgitation include infective endocarditis, rheumatic heart disease, trauma with valvular rupture, congenital bicuspid aortic valve, myxomatous degeneration, syphilitic aortitis, systemic lupus erythematosus (SLE), aortic dissection and the use of amphetamine slimming products. Symptoms of aortic regurgitation include dyspnoea on exertion, syncope, chest pain and congestive heart failure. Cardiac auscultation characteristically reveals displacement of the cardiac impulse downwards and to the left, prominent S3 heard over the apex, a low-pitched apical diastolic rumble (Austin–Flint murmur) and an early systolic apical ejection murmur. Chest X-ray may reveal left ventricular hypertrophy and aortic dilatation. Echocardiography reveals the coarse diastolic fluttering of the anterior mitral valve leaflet. Surgical valve replacement is indicated in symptomatic patients with chronic aortic regurgitation who have symptoms despite optimal medical management, and in acute aortic regurgitation where there is evidence of left ventricular failure. Ideally, surgery should be considered before the ejection fraction falls to below 55%.

An 18-year-old student is admitted from a night club in a state of collapse. On admission to A&E his blood pressure is 90/45 mmHg, and he has a pulse of 190 per minute. ECG reveals a narrow complex tachycardia, which is terminated with adenosine. ECG after termination of the tachycardia reveals a PR interval of approximately 100 ms, and a slurred QRS complex with delta wave. What diagnosis fits best with this clinical picture? Amphetamine overdose Cocaine overdose Hypokalaemia-induced arrhythmia Wolff–Parkinson–White syndrome Lown–Ganong–Levine syndrome An 18-year-old student is admitted from a night club in a state of collapse. On admission to A&E his blood pressure is 90/45 mmHg, and he has a pulse of 190 per minute. ECG reveals a narrow complex tachycardia, which is terminated with adenosine. ECG after termination of the tachycardia reveals a PR interval of approximately 100 ms, and a slurred QRS complex with delta wave. What diagnosis fits best with this clinical picture? Amphetamine overdose Cocaine overdose Hypokalaemia-induced arrhythmia Wolff–Parkinson–White syndrome

Your answer

Lown–Ganong–Levine syndrome

Wolff–Parkinson–White (WPW) syndrome (due to accessory cardiac conduction pathway) presents with paroxysmal tachycardias in 10% of patients aged 20–40 years, and 35% of sufferers aged over 60 years. Common types of arrhythmia at presentation include reciprocating tachycardia at 150–250 bpm (80%), atrial fibrillation (15%) and atrial flutter (5%). Thankfully, presentation with ventricular tachycardia is rare. Prevalence in

of 179

6/30/2011 4:56 PM

the UK population is around 0.15%, being more frequent in males. Most WPW patients have a normal heart structure,

www.apnastudent.blogspot.com but there may be associated mitral valve prolapse, cardiomyopathy or Ebstein’s anomaly in certain patients. ECG abnormalities are characterised by the presence of a PR interval < 120 ms and a QRS complex >120 ms with slurred, slowly rising onset (delta wave). The Lown–Ganong–Levine syndrome is characterised by a short PR interval and normal QRS complex on ECG. Narrow complex tachycardias may be terminated acutely with adenosine or verapamil or cardioversion. Digoxin should not be used as it may accelerate tachycardias. In the non-acute stage, radiofrequency ablation of the accessory pathway may be attempted.

A 54-year-old man presents with an irregular tachycardia with a ventricular rate of around 130 bpm. He played in a cricket match the previous day and consumed 28 units of alcohol on the evening of the match. On examination his blood pressure is 95/50 mmHg. What is the most likely diagnosis? Ventricular tachycardia Sick-sinus syndrome Paroxysmal atrial fibrillation Atrial flutter Sinus tachycardia A 54-year-old man presents with an irregular tachycardia with a ventricular rate of around 130 bpm. He played in a cricket match the previous day and consumed 28 units of alcohol on the evening of the match. On examination his blood pressure is 95/50 mmHg. What is the most likely diagnosis? Ventricular tachycardia Sick-sinus syndrome Paroxysmal atrial fibrillation

Correct answer

Atrial flutter

Your answer

Sinus tachycardia

This man has paroxysmal atrial fibrillation as evidenced by his irregular fast tachycardia. Episodes of tachycardia in this condition may occasionally be precipitated by an excess intake of alcohol or caffeine. Other causes may be acute myocardial infarction, atrial septal defect, or pre-excitation syndromes such as Wolff–Parkinson–White. Atrial flutter is associated with an absolutely regular rhythm of 150–220 bpm. Standard therapy for atrial fibrillation of recent onset is electrical cardioversion, providing there are no contraindications. Intravenous flecainide may be considered for chemical cardioversion in the absence of a history of ischaemic heart disease; amiodarone is an acceptable alternative. Long-term prophylaxis with agents such as sotalol may be required.

A 34-year-old man is brought in as an emergency by his wife. He has been unwell for a few days with severe pharyngitis that is thought to be related to Epstein–Barr virus infection. On arrival in emergency he is visibly dyspnoeic at rest, there is evidence of ankle swelling and bilateral crackles on auscultation of his chest, his blood pressure is 100/55 mmHg and his pulse is 105 bpm. ECG reveals sinus tachycardia with non-specific ST–T wave changes. Troponin-T is elevated. What diagnosis fits best with this clinical picture? Acute myocardial infarction Pericarditis Cardiomyopathy Myocarditis Subacute bacterial endocarditis A 34-year-old man is brought in as an emergency by his wife. He has been unwell for a few days with severe pharyngitis that is thought to be related to Epstein–Barr virus infection. On arrival in emergency he is visibly dyspnoeic at rest, there is

of 179

6/30/2011 4:56 PM

evidence of ankle swelling and bilateral crackles on auscultation of his chest, his blood pressure is 100/55 mmHg and his pulse is 105 bpm. ECG reveals sinus tachycardia with non-specific ST–T wave changes. Troponin-T is elevated.

www.apnastudent.blogspot.com What diagnosis fits best with this clinical picture? Acute myocardial infarction Pericarditis Cardiomyopathy Myocarditis

Your answer

Subacute bacterial endocarditis

Causes of myocarditis may be viral (eg coxsackievirus B, echovirus, poliovirus, adenovirus, mumps, human immunodeficiency virus (HIV) or Epstein–Barr virus (EBV)); bacterial (eg Staphylococcus aureus, Clostridium perfringens or Corynebacterium diphtheriae); mycoplasma; fungal (eg Candida or Aspergillus spp); parasitic (eg Trypanosoma cruzi, Trichinella, Echinococcus, amoeba or Toxoplasma spp); rickettsial, or spirochaetal (eg Lyme carditis). Other possible causes are rheumatic fever; drugs (eg cocaine, doxorubicin, sulphonamides, tetracycline, amphotericin B and 5-fluorouracil (5-FU)); toxins such as carbon monoxide, lead or arsenic; systemic lupus erythematosus (SLE); sarcoidosis; or radiation. The medical history may point to a possible cause of the carditis; laboratory testing may reveal raised troponin-T levels, increased creatine kinase (CK), increased erythrocyte sedimentation rate (ESR) or increased white blood cell count. Viral titres, cold agglutinins and Lyme disease titres may also help in determining aetiology. ECG often reveals sinus tachycardia with non-specific ST changes. Echocardiography reveals dilated and hypokinetic chambers with segmental wall motion abnormalities. Prognosis is dependent on aetiology, but the 5-year mortality rate may be as high as 50% in some cases.

A 72-year-old woman presents with two syncopal episodes, and is brought to the Emergency department by her daughter. The second episode has occurred on a particularly hot day after a family walk. She has a past history of hypertension and takes bendrofluazide. On admission to Emergency her blood pressure is 160/125 mmHg and there is an ejection systolic murmur on auscultation of her chest that radiates to the carotids. What diagnosis best fits with this clinical picture? Mitral regurgitation Hypertrophic cardiomyopathy Aortic stenosis Acute arrhythmia Dehydration due to diuretic use A 72-year-old woman presents with two syncopal episodes, and is brought to the Emergency department by her daughter. The second episode has occurred on a particularly hot day after a family walk. She has a past history of hypertension and takes bendrofluazide. On admission to Emergency her blood pressure is 160/125 mmHg and there is an ejection systolic murmur on auscultation of her chest that radiates to the carotids. What diagnosis best fits with this clinical picture? Mitral regurgitation Hypertrophic cardiomyopathy Aortic stenosis

Your answer

Acute arrhythmia Dehydration due to diuretic use

Aortic stenosis causes left ventricular outflow obstruction, which is manifest by a rough ejection systolic murmur, best heard at the base of the heart, and transmitted to the carotids. As aortic stenosis becomes more severe, the sound of

of 179

6/30/2011 4:56 PM

aortic valve closure begins to diminish in intensity. There is associated left ventricular hypertrophy, with narrowing of the pulse pressure in the later stages of aortic stenosis. Symptoms commonly appear when the valve orifice decreases to less than 1 cm squared (normal orifice is 3 cm squared). Stenosis is considered severe when the orifice is less than 0.5 cm squared or the pressure gradient across the valve is 50 mmHg or greater.

www.apnastudent.blogspot.com Symptoms of aortic stenosis include angina, syncope (particularly exertional) and congestive heart failure. GI bleeding may occur, as there is an association between aortic stenosis and haemorrhagic telangiectasia. Investigations of choice are chest x-ray and Echocardiography, with cardiac catheterisation in symptomatic patients to assess the gradient across the valve. Surgical valve replacement is the treatment of choice in appropriate patients.

An 82-year-old man was admitted to the Emergency department from a local church service. He fainted and another parishioner, who is a trained first-aider, reported that he was pulseless for a few seconds after the attack. On examination his blood pressure was 165/95 mmHg (past history of hypertension), he had no murmurs on auscultation of the chest and carotid auscultation was also normal. Outpatient 7-day ambulatory cardiac rhythm monitoring was arranged, which is now reported as normal. What diagnosis fits best with this presentation? Transient ischaemic attack Transient bradycardia Paroxysmal atrial fibrillation Simple syncope Carotid sinus syndrome An 82-year-old man was admitted to the Emergency department from a local church service. He fainted and another parishioner, who is a trained first-aider, reported that he was pulseless for a few seconds after the attack. On examination his blood pressure was 165/95 mmHg (past history of hypertension), he had no murmurs on auscultation of the chest and carotid auscultation was also normal. Outpatient 7-day ambulatory cardiac rhythm monitoring was arranged, which is now reported as normal. What diagnosis fits best with this presentation? Transient ischaemic attack Transient bradycardia Paroxysmal atrial fibrillation Simple syncope Carotid sinus syndrome

Your answer

The incidence of carotid sinus syndrome is said to be around 10% in the adult population. This incidence increases with age, and men are affected twice as often as women. Presentation is rare below the age of 50 years. The incidence also increases in patients with hypertension, but often a definitive cause may not be identified. Predisposing factors may include head and neck tumours, neck surgery, significant lymphadenopathy or carotid body tumours. Physical examination in this case suggests that there is no significant cardiac pathology, and the 7-day Holter monitor result makes arrhythmia less likely as a cause. Supine carotid sinus massage with blood pressure and ECG monitoring is the diagnostic procedure. I t should not be performed in patients with a history of cerebrovascular disease or carotid bruits, and should only be applied to one artery at a time. The response may be cardioinhibitory with asystole for at least 3 s, or vasopressor with a drop of more than 30 mmHg (in the presence of symptoms) or more than 50 mmHg without symptoms. A mixed picture of a cardioinhibitory and vasopressor response may occur. Pacemaker insertion is recommended for patients with cardioinhibitory carotid sinus syndrome.

A 62-year-old woman with a past history of rheumatic fever presents for review. In recent years she has been well, but underwent a dental extraction some 8 weeks ago. During the past 4 weeks she has suffered intermittent fevers, chills and night sweats. On examination there is a pansystolic murmur, loudest at the apex. Blood tests reveal a normochromic, normocytic anaemia and raised ESR. What organism is most likely to be responsible for this clinical picture? Staphylococcus epidermidis Viridans streptococci Staphlococcus

of 179

6/30/2011 4:56 PM

aureus albicans

Candida

www.apnastudent.blogspot.com Enterococci A 62-year-old woman with a past history of rheumatic fever presents for review. In recent years she has been well, but underwent a dental extraction some 8 weeks ago. During the past 4 weeks she has suffered intermittent fevers, chills and night sweats. On examination there is a pansystolic murmur, loudest at the apex. Blood tests reveal a normochromic, normocytic anaemia and raised ESR. What organism is most likely to be responsible for this clinical picture? Staphylococcus epidermidis Viridans streptococci

Your answer

Staphlococcus aureus Candida albicans Enterococci

This patient has previous rheumatic fever (and would previously have received antibiotic cover for dental extraction, however there is no robust evidence that antibiotic prophylaxis reduces endocarditis risk). From her presentation it is most likely that she now has sub-cute bacterial endocarditis. Organisms associated with subacute bacterial endocarditis include viridans streptococci, Streptococcus bovis, enterococci and Staphylococcus aureus. Patients may present with fever, chills or fatigue (said to occur in 25–80% of patients). Heart murmurs may be absent in right-sided endocarditis. Embolic phenomena with peripheral manifestations may be found in up to 50% of patients. Other manifestations include finger clubbing, petechiae, Osler’s nodes, splinter haemorrhages and Janeway lesions. Splenomegaly may also occur in association with subacute bacterial endocarditis. I t is crucial to collect at least three sets of blood samples for culture during the first 24 h, which should, if possible, be taken before antibiotics are started. Normochromic, normocytic anaemia may also occur. The erythrocyte sedimentation rate (ESR) is elevated and a false-positive VDRL (syphilis test) may occur. Transthoracic echocardiography, with or without additional transoesophageal echo is indicated to confirm diagnosis.

A 34-year-old professional footballer is evaluated for symptoms of ‘dizziness’ during exercise. Physical examination reveals a laterally displaced apical impulse. On auscultation, there is a 2/6 mid-systolic murmur in the aortic area that increases on sudden standing. The ECG shows LVH and Q waves in the V 2–V 5 leads. What is the most likely diagnosis? Young-onset hypertension Acute MI Aortic stenosis Hypertrophic cardiomyopathy Atrial septal defect

A 34-year-old professional footballer is evaluated for symptoms of ‘dizziness’ during exercise. Physical examination reveals a laterally displaced apical impulse. On auscultation, there is a 2/6 mid-systolic murmur in the aortic area that increases on sudden standing. The ECG shows LVH and Q waves in the V 2–V 5 leads. What is the most likely diagnosis? Young-onset hypertension Acute MI Aortic stenosis Hypertrophic cardiomyopathy

Your answer

Atrial septal defect

Hypertrophic cardiomyopathy is the single, most common cause of sudden death in young athletes. The majority of

of 179

6/30/2011 4:56 PM

patients are asymptomatic or only mildly symptomatic. Syncope and sudden death are associated with severe exertion and competitive sports, which should be avoided in patients with hypertrophic cardiomyopathy. The most common symptom is dyspnoea. Typical examination findings include left ventricular hypertrophy and a loud S 4. Forceful atrial

www.apnastudent.blogspot.com systole causes a double apical impulse. A triple apical impulse can also occur due to a late systolic bulge. The carotid pulse also demonstrates a late systolic pulse causing the characteristic jerky feature. In contrast to aortic stenosis, the systolic murmur of hypertrophic cardiomyopathy does not radiate to the carotids. Moreover, it decreases on squatting and passive leg elevation and increases with the Valsalva manoeuvre. The most common changes seen on the ECG are ST–T wave abnormalities followed by LVH. Q waves may also occur in leads I I, II I, aV F or V 2–V 6. The exact cause of the Q waves remains unclear.

A 70-year-old-man reverts to atrial fibrillation after several attempts at cardioversion, but remains symptomatic despite rate control with digoxin and metoprolol. He developed pulmonary fibrosis with amiodarone. Which of the following will be the next step in the management of this patient? Switch metoprolol to amlodipine Double the dose of digoxin Radiofrequency pulmonary vein isolation with ablation Make another attempt at cardioversion Implant a cardiovertor defibrillator A 70-year-old-man reverts to atrial fibrillation after several attempts at cardioversion, but remains symptomatic despite rate control with digoxin and metoprolol. He developed pulmonary fibrosis with amiodarone. Which of the following will be the next step in the management of this patient? Switch metoprolol to amlodipine Double the dose of digoxin Radiofrequency pulmonary vein isolation with ablation

Correct answer

Make another attempt at cardioversion Implant a cardiovertor defibrillator

Your answer

It is not always possible to restore and maintain sinus rhythm in patients with AF. I f sinus rhythm cannot be maintained, treatment should be directed towards controlling the heart rate with digoxin,

β-blockers, rate-limiting

calcium-channel blockers (verapamil or diltiazem) or amiodarone. β-Blockers and calcium-channel blockers are often more effective than digoxin at controlling the heart rate during exercise. For patients who remain poorly controlled despite medical therapy, radiofrequency pulmonary vein isolation with ablation is now seen as the treatment of choice for atrial fibrillation cessation.

A 72-year-old man was discharged following successful prosthetic aortic valve replacement. Apart from a small Venflon abscess, which healed with appropriate dressings and cannula removal, his progress had been unremarkable. Now, some 6 weeks later, he is brought to A&E by his wife, suffering from malaise, fever and night sweats. On examination you can hear the murmur of his prosthetic heart valve. Blood testing reveals mild anaemia and raised ESR. Transoesophageal echocardiography suggests the possibility of vegetations. Which of the following regimens is the most appropriate initial choice of antibiotic therapy? Intravenous penicillin therapy Intravenous gentamicin therapy Intravenous penicillin and gentamicin Intravenous vancomycin, rifampicin and gentamicin Intravenous gentamicin and vancomycin A 72-year-old man was discharged following successful prosthetic aortic valve replacement. Apart from a small Venflon abscess, which healed with appropriate dressings and cannula removal, his progress had been unremarkable. Now, some 6 weeks later, he is brought to A&E by his wife, suffering from malaise, fever and night sweats. On examination you can hear the murmur of his prosthetic heart valve. Blood testing reveals mild anaemia and raised ESR. Transoesophageal echocardiography suggests the possibility of vegetations.

of 179

6/30/2011 4:56 PM

Which of the following regimens is the most appropriate initial choice of antibiotic therapy? Intravenous penicillin therapy

www.apnastudent.blogspot.com Intravenous gentamicin therapy Intravenous penicillin and gentamicin Intravenous vancomycin, rifampicin and gentamicin

Your answer

Intravenous gentamicin and vancomycin

The choice of antibiotics to treat endocarditis should be guided by local policy, but in the case of possible prosthetic valve endocarditis, regime D is the most appropriate. Regime C is the most appropriate initial therapy for non-prosthetic valve endocarditis. Early prosthetic valve endocarditis is usually caused by Staphylococcus epidermidis, occurring during the first 2 months postvalve replacement, other causative organisms include Staphylococcus aureus, Gram-negative bacilli, diphtheroids and Candida species. I n patients with prosthetic valves, transthoracic echocardiography is less sensitive than transoesophageal echo for detecting valve abnormalities. Unfortunately medical therapy is rarely successful in prosthetic valve endocarditis, and surgical valve replacement under antibiotic cover is usually required.

A 22-year-old student is admitted by ambulance from a local night club. He has no previous medical history of note and is adopted so is unaware of his family history. Bystanders who have accompanied him say that he suffered sudden collapse while dancing. Bouncers at the club claim that they couldn’t feel a strong pulse during his period of unconsciousness. On admission his blood pressure is 120/60 mmHg, and pulse is 80 bpm and regular. ECG looks normal, corrected QT interval is 0.6 s. What diagnosis fits best with his clinical picture? Simple syncope Long QT syndrome – mutation uncharacterised Ecstasy overdose Carotid sinus syndrome Jervell–Lange-Nielsen (JLN) syndrome A 22-year-old student is admitted by ambulance from a local night club. He has no previous medical history of note and is adopted so is unaware of his family history. Bystanders who have accompanied him say that he suffered sudden collapse while dancing. Bouncers at the club claim that they couldn’t feel a strong pulse during his period of unconsciousness. On admission his blood pressure is 120/60 mmHg, and pulse is 80 bpm and regular. ECG looks normal, corrected QT interval is 0.6 s. What diagnosis fits best with his clinical picture? Simple syncope Long QT syndrome – mutation uncharacterised

Your answer

Ecstasy overdose Carotid sinus syndrome Jervell–Lange-Nielsen (JLN) syndrome

This man’s QT interval is prolonged. JLN syndrome is also associated with long QT, but patients have deafness in addition to the cardiac rhythm abnormality. Episodes of severe QT prolongation and torsades de pointes ventricular tachycardia in congenital long QT syndrome may be precipitated by increased adrenergic drive (such as that from dancing in a night club). This patient is adopted, so that it may be possible that there is an unknown family history of sudden death. The molecular biology of long QT syndromes is heterogeneous, and a number of different mutations coding for potassium or sodium channels may be responsible. Where specific mutations are identified, antiarrhythmic therapy may be specifically targeted to provide optimum therapy. In patients who respond poorly to medical treatment, implantable defibrillator may be considered.

You review a 61-year-old man with paroxysmal atrial fibrillation. You consider a class Ic antiarrhythmic agent as the most appropriate choice to maintain him in sinus rhythm.

of 179

6/30/2011 4:56 PM

What statement best describes the effect that class Ic agents have on electrical activity in the heart? Lengthen the cardiac action potential Shorten the cardiac action potential Widen the duration of the action potential Have minimal effect on the duration of the action potential Predominantly affect the AV node You review a 61-year-old man with paroxysmal atrial fibrillation. You consider a class Ic antiarrhythmic agent as the most appropriate choice to maintain him in sinus rhythm. What statement best describes the effect that class Ic agents have on electrical activity in the heart? Lengthen the cardiac action potential Shorten the cardiac action potential Widen the duration of the action potential Have minimal effect on the duration of the action potential

Your answer

Predominantly affect the AV node

Class Ic antiarrhythmic agents such as flecainide or propafenone have no significant effect on the cardiac action potential. They are commonly used for the treatment and prophylaxis of atrial arrhythmias such as paroxysmal atrial fibrillation and atrial tachycardia.Class I a agents such as quinidine lengthen the action potential, Ib agents such as lidocaine shorten the action potential, and class II I agents widen the duration of the action potential. Beta-blocking agents predominantly affect the sinus node, whereas calcium-channel blocking agents predominantly affect the atrioventricular (AV) node. It is important to note that class I agents should be avoided in patients with significant coronary artery disease; flecainide, in particular, was shown to be associated with increased mortality in a postmyocardial infarction study.

A 26-year-old woman attends her GP for an insurance medical. Her previous medical history is unremarkable. On examination her BMI is 21, blood pressure is 105/62 mmHg, and auscultation of the heart reveals a mid systolic click, and a late systolic murmur (these findings being accentuated in the standing position). What diagnosis fits best with this clinical picture? Atrial septal defect Mitral regurgitation

Mitral

stenosis Mitral valve prolapse Constrictive pericarditis A 26-year-old woman attends her GP for an insurance medical. Her previous medical history is unremarkable. On examination her BMI is 21, blood pressure is 105/62 mmHg, and auscultation of the heart reveals a mid systolic click, and a late systolic murmur (these findings being accentuated in the standing position). What diagnosis fits best with this clinical picture? Atrial septal defect Mitral regurgitation

Mitral

stenosis Mitral valve prolapse Constrictive pericarditis

Your answer

www.apnastudent.blogspot.com

Mitral valve prolapse is the posterior bulging of leaflets of the mitral valve in systole. I t is thought to be present in around 4% of the population, with a higher incidence in females. Increased incidence is associated with autoimmune

of 179

6/30/2011 4:56 PM

thyroid disease, Ehlers–Danlos syndrome, Marfan’s syndrome, pseudoxanthoma elasticum and pectus excavatum. Clinically, patients are often young females with a narrow anteroposterior (AP) chest diameter, low body weight and low/normal blood pressure. Cardiac auscultation reveals a mid to late systolic click, best heard at the apex, and a mid to late systolic murmur. Associated embolic phenomena (stroke or transient ischaemic attack (TIA)) are rare. Echocardiography reveals bulging of the anterior and posterior mitral valve leaflets in systole. The incidence of complications of mitral valve prolapse is thought to be less than 1% per year, and treatment is often not required.

A 54-year-old man is 48-h postmyocardial infarction. You are asked to review him as he is suffering worsening cardiac failure. On examination he has a pansystolic murmur, loudest at the apex. What complication of his myocardial infarction is most likely to have occurred? Ventricular septal defect Atrial septal defect Acute mitral regurgitation Acute pulmonary regurgitation Ventricular rupture A 54-year-old man is 48-h postmyocardial infarction. You are asked to review him as he is suffering worsening cardiac failure. On examination he has a pansystolic murmur, loudest at the apex. What complication of his myocardial infarction is most likely to have occurred? Ventricular septal defect Atrial septal defect Acute mitral regurgitation

Your answer

Acute pulmonary regurgitation Ventricular rupture

Acute mitral regurgitation associated with myocardial infarction may occur due to ruptured chordae tendineae. Other causes of mitral regurgitation include papillary muscle dysfunction, infective endocarditis, rheumatic heart disease, idiopathic myxomatous valve degeneration, left atrial myxoma, systemic lupus erythematosus (SLE) and drugs (fenfluramine and dexfenfluramine). The investigation of choice is echocardiography, which may identify left atrial and left ventricular dilatation and confirm the diagnosis of chordae tendineae rupture. Mitral regurgitation associated with chordal rupture in MI may be catastrophic and require emergency surgery for valve replacement. Acute medical management involves treatment with angiotensin-converting enzyme (ACE) inhibition, diuretic therapy and possible anticoagulation. The prognosis for patients with mitral regurgitation is generally good, except in the post-MI situation.

A 70-year-old woman had a history of dyspnoea and palpitations for six months. An electrocardiogram (ECG) at that time showed atrial fibrillation. She was given digoxin, diuretics, and aspirin. She now presents with two short-lived episodes of altered sensation in the left face, left arm, and leg. There is poor coordination of the left hand but she tells you that this began around 6 months earlier. The echocardiogram (ECHO) was normal, as was a computed tomography (CT) head scan. What is the most appropriate next step in management? Anticoagulation Carotid endarterectomy Clopidogrel Corticosteroid treatment No action

A 70-year-old woman had a history of dyspnoea and palpitations for six months. An electrocardiogram (ECG) at that time

www.apnastudent.blogspot.com showed atrial fibrillation. She was given digoxin, diuretics, and aspirin. She now presents with two short-lived episodes of altered sensation in the left face, left arm, and leg. There is poor coordination of the left hand but she tells you that this began around 6 months earlier. The echocardiogram (ECHO) was normal, as was a computed tomography (CT) head scan.

of 179

6/30/2011 4:56 PM

What is the most appropriate next step in management? Anticoagulation

Your answer

Carotid endarterectomy Clopidogrel Corticosteroid treatment No action

Guidelines state that anticoagulation is indicated in patients with any one of prosthetic heart valve, prior history of rheumatic heart valve disease, prior history of stroke or transient ischaemic attack, age older than 75 years, hypertension or coronary artery disease with poor left ventricle ( LV) function. Other risk factors that occur concurrently with atrial fibrillation and suggest a need for possible anticoagulation include diabetes mellitus, age 65–75 years, and coronary artery disease with normal LV function (clinicians look for two of these ‘moderate’ risk factors). Last year though a Cochrane analysis has suggested that most patients with atrial fibrillation should be considered for anticoagulation unless there are specific reasons not to. In the case of this woman she has suffered at least two transient ischaemic attacks (TIAs), with some residual poor damage to coordination in the left arm, so that she now fits the criteria for anticoagulation. Carotid endarterectomy is indicated where there is symptomatic carotid stenosis. Clopidogrel would be indicated in TI A without atrial fibrillation. Corticosteroids may be considered in cases of cerebral oedema where there is significant mass effect.

A 21-year-old woman has a history of palpitations and light-headedness. The electrocardiogram (ECG) shows a short PR interval and inferior Q waves. Her symptoms improve with atenolol 25 mg/day, but she has had two short episodes of similar symptoms in the previous 24 hours. What is the long-term management of choice? Anticoagulation Oral amiodarone Oral digoxin Increase the dose of atenolol Radiofrequency ablation A 21-year-old woman has a history of palpitations and light-headedness. The electrocardiogram (ECG) shows a short PR interval and inferior Q waves. Her symptoms improve with atenolol 25 mg/day, but she has had two short episodes of similar symptoms in the previous 24 hours. What is the long-term management of choice? Anticoagulation Oral amiodarone Oral digoxin Increase the dose of atenolol Radiofrequency ablation

Your answer

The short PR interval without delta wave suggests Long–Ganong–Levine (LGL) syndrome rather than Wolf–Parkinson– White (WPW) syndrome. It is likely that the patient is suffering from short periods of supraventricular tachycardia, which result in her palpitations and light-headedness. The management of WPW and LGL syndrome is similar, radiofrequency ablation is recommended for these patients. Digoxin is not recommended as it may result in an increased ventricular rate and worsen any circulatory compromise during attacks of tachycardia. Long-term oral amiodarone therapy is not recommended in view of the age of this patient. Atenolol may be useful to manage ventricular rate during periods of tachycardia, but again is a sub-optimal choice for this patient in the long term.

A 35-year-old woman presented with a history of intermittent light-headedness. Clinical examination and 12-lead

www.apnastudent.blogspot.com electrocardiogram (ECG) were normal. Which of the following, if present on a 24 hour Holter ECG tracing, would be the most clinically important?

of 179

6/30/2011 4:56 PM

Atrial premature beats Profound sleep-associated bradycardia Supraventricular tachycardia Transient Mobitz type-1 atrioventricular block Ventricular premature beats A 35-year-old woman presented with a history of intermittent light-headedness. Clinical examination and 12-lead electrocardiogram (ECG) were normal. Which of the following, if present on a 24 hour Holter ECG tracing, would be the most clinically important? Atrial premature beats Profound sleep-associated bradycardia Supraventricular tachycardia

Correct answer

Transient Mobitz type-1 atrioventricular block

Your answer

Ventricular premature beats

Both atrial and ventricular premature beats are normal variants when seen on a 24 hour Holter electrocardiogram (ECG) tracing. Profound bradycardia may also occur during sleep and is a normal finding. Mobitz type-1 atrioventricular block carries less clinical significance than Mobitz type-2 because the risk of progression to complete heart block is much lower. Thus supraventricular tachycardia (SVT) carries the most clinical significance. Diagnosis of the underlying cause is based on the presence or absence of P-waves and P-wave morphology. Patients can be taught carotid sinus massage to avert SVTs at home, or adenosine can be used in non-asthmatic patients for acute cardioversion to sinus rhythm. Class III anti-arrythmics, such as sotalol, may be considered for prophylaxis.

A 72-year-old man presented with an episode of collapse. He had experienced two similar episodes recently, each lasting about one minute. Four years previously he suffered an anterior myocardial infarction. On examination he was orientated and symptom-free with a regular pulse rate of 80 beats per minute (bpm), blood pressure 140/80 mmHg, and the apex beat was displaced to the left. There was an apical systolic murmur. There were no signs of trauma. The electrocardiogram (ECG) showed sinus rhythm, Q waves, and ST segment elevation anteriorly without reciprocal depression. What is the diagnosis? Acute anterior myocardial infarction Cerebrovascular accident Epileptic seizure Pulmonary embolism Ventricular tachycardia A 72-year-old man presented with an episode of collapse. He had experienced two similar episodes recently, each lasting about one minute. Four years previously he suffered an anterior myocardial infarction. On examination he was orientated and symptom-free with a regular pulse rate of 80 beats per minute (bpm), blood pressure 140/80 mmHg, and the apex beat was displaced to the left. There was an apical systolic murmur. There were no signs of trauma. The electrocardiogram (ECG) showed sinus rhythm, Q waves, and ST segment elevation anteriorly without reciprocal depression. What is the diagnosis? Acute anterior myocardial infarction Cerebrovascular accident

Your answer

Epileptic seizure Pulmonary embolism Ventricular tachycardia

Correct answer

www.apnastudent.blogspot.com It is likely that this man has suffered a transient episode of ventricular tachycardia, which results in a period of circulatory compromise that leads to the collapse. The persistent ST segment elevation in this case would not indicate acute myocardial infarction, but it is likely to represent left ventricular aneurysm, a recognised complication of acute

of 179

6/30/2011 4:56 PM

anterior myocardial infarction. Holter monitoring of his electrocardiogram (ECG) would be the investigation of choice –seven-day cardiac monitors are now available and would indicate the best chance of capturing a period of ventricular tachycardia. Anti-arrythmic of choice would be amiodarone, although this patient should probably be considered for an implantable defibrillator once the diagnosis is confirmed.

A 58-year-old male patient has suffered from a recent acute myocardial infarction 3 days ago. He becomes acutely unwell with a hypotensive episode. There is a pansystolic murmur which is accentuated by inspiration, along the lower left sternal border. A Swan-Ganz catheter was inserted and the following was noted: right atrial pressure was 12 (very high); calculated left atrial pressure was 2 (low normal). What is the likely cause? Right heart failure Left heart failure Mitral regurgitation Tricuspid regurgitation Aortic regurgitation A 58-year-old male patient has suffered from a recent acute myocardial infarction 3 days ago. He becomes acutely unwell with a hypotensive episode. There is a pansystolic murmur which is accentuated by inspiration, along the lower left sternal border. A Swan-Ganz catheter was inserted and the following was noted: right atrial pressure was 12 (very high); calculated left atrial pressure was 2 (low normal). What is the likely cause? Right heart failure Left heart failure Mitral regurgitation Tricuspid regurgitation

Your answer

Aortic regurgitation

Tricuspid regurgitation may occur in post-myocardial infarction, in association with cor pulmonale, rheumatic heart disease, infective endocarditis, carcinoid syndrome, Ebstein’s anomaly, and other congenital abnormalities of the atrioventricular valves. Regurgitation gives rise to high right atrial pressures (as seen here). Physical signs include a large jugular venous cardiovascular wave and a pulsatile liver that pulsates in systole. A right ventricular impulse may be felt at the left sternal edge and there is a blowing pansystolic murmur. Severe tricuspid regurgitation may require valve repair, or rarely replacement. Another consideration with this type of presentation post-myocardial infarction is pulmonary embolus, a high proportion of those patients who die post-myocardial infarction, do so because of thromboembolic disease.

A 35-year-old-woman of African origin presents with a 4-month history of increasing swelling over her feet and abdominal distension. She has no history of cough, orthopnoea or breathlessness on exertion. Her heart rate is 98 beats/minute: irregularly irregular. Her JVP is markedly raised and she has pitting lower limb oedema. The heart sounds are soft, and there are no audible murmurs. Abdominal examination reveals hepatomegaly along with ascites. Chest X-ray reveals a normal cardiac size and clear lung fields. A lateral X-ray shows calcification around the heart border. Urinalysis is normal. Her ECG shows a low QRS voltage and lateral T-wave changes. What is the likely diagnosis? Dilated cardiomyopathy Cirrhosis of the liver Constrictive pericarditis Restrictive cardiomyopathy Hypertrophic cardiomyopathy

A 35-year-old-woman of African origin presents with a 4-month history of increasing swelling over her feet and abdominal

www.apnastudent.blogspot.com distension. She has no history of cough, orthopnoea or breathlessness on exertion. Her heart rate is 98 beats/minute: irregularly irregular. Her JVP is markedly raised and she has pitting lower limb oedema. The heart sounds are soft, and there are no audible murmurs. Abdominal examination reveals hepatomegaly along with ascites. Chest X-ray reveals a normal cardiac size and clear lung fields. A lateral X-ray shows calcification around the heart border. Urinalysis is normal. Her ECG

of 179

6/30/2011 4:56 PM

shows a low QRS voltage and lateral T-wave changes. What is the likely diagnosis? Dilated cardiomyopathy Cirrhosis of the liver Constrictive pericarditis

Your answer

Restrictive cardiomyopathy Hypertrophic cardiomyopathy

This patient has signs of severe right heart failure but the chest X-ray reveals a normal heart size. The possibilities are constrictive pericarditis and restrictive cardiomyopathy. The presence of calcification around the heart favours constrictive pericarditis. Causes of restrictive cardiomyopathy include cardiac amyloidosis, haemachromatosis, endomyocardial fibrosis, systemic sclerosis, carcinoid syndrome and malignancy. Cardiac amyloidosis is usually associated with myeloma. It is more common in males in their sixth or seventh decades.

A 32-year-old man is recently diagnosed with ankylosing spondylitis. Echocardiogram showed a valvular abnormality. What is the most likely diagnosis? Mitral regurgitation Aortic regurgitation Mitral stenosis Aortic stenosis Tricuspid stenosis A 32-year-old man is recently diagnosed with ankylosing spondylitis. Echocardiogram showed a valvular abnormality. What is the most likely diagnosis? Mitral regurgitation Aortic regurgitation

Your answer

Mitral stenosis Aortic stenosis Tricuspid stenosis

Causes and associations of chronic aortic regurgitation include rheumatic heart disease, syphilis, sero-negative arthritides including ankylosing spondylitis, severe hypertension, congenital bicuspid aortic valve, aortic endocarditis, Marfan’s syndrome and osteogenesis imperfecta. Signs include a bounding or collapsing pulse and the number of eponymous signs that accompany that (Quincke’s, De Musset’s and Duroziez’s signs). Investigations include chest X-ray, electrocardiograph (ECG), echocardiogram and cardiac catheterisation to assess the degree of regurgitation. Valve replacement is indicated before the appearance of significant left ventricular failure, as valve replacement before the onset of symptoms is associated with a much more favourable prognosis.

A patient presents with shortness of breath and ankle swelling. An echocardiogram has been ordered to determine the left ventricular ejection fraction. Which echocardiography mode is the most appropriate?

www.apnastudent.blogspot.com M-mode A-mode Modern transthoracic Continuous wave

of 179

6/30/2011 4:56 PM

Power wave A patient presents with shortness of breath and ankle swelling. An echocardiogram has been ordered to determine the left ventricular ejection fraction. Which echocardiography mode is the most appropriate? M-mode A-mode Modern transthoracic

Correct answer

Continuous wave

Your answer

Power wave

Modern transthoracic echocardiography combines real-time two-dimensional imaging of the myocardium and valves with information about velocity and direction of blood flow obtained by Doppler and colour flow mapping. It is non-invasive and a complete examination can be performed in most patients in less than 30 min. M-mode echocardiography has preceded modern two-dimensional imaging. Unlike two-dimensional imaging, which uses a series of sweeps across the heart, M-mode uses a single static beam of very frequent ultrasound pulses. The narrow beam is analogous to a vertical mineshaft passing through various layers of rock. Displayed in real time, this results in reflections from cardiac structures being displayed as horizontal lines with superficial structures at the top of the screen and the deeper structures at the bottom. This data are interpretable when one knows which structure each line represents, and the technique has excellent spatial resolution.

A 28-year-old man presents with a 2-year history of increasing dyspnoea with strenuous exertion. Hypertrophic cardiomyopathy is diagnosed. Which is the most appropriate screening method for his brother? Computed tomography (CT) scan Exercise tolerance test Ventilation–perfusion scan Echocardiography Genetic screening

A 28-year-old man presents with a 2-year history of increasing dyspnoea with strenuous exertion. Hypertrophic cardiomyopathy is diagnosed. Which is the most appropriate screening method for his brother? Computed tomography (CT) scan Exercise tolerance test Ventilation–perfusion scan Echocardiography

Your answer

Genetic screening

Hypertrophic cardiomyopathy is usually familial with autosomal dominant transmission. The diagnosis of hypertrophic cardiomyopathy is based upon the demonstration of unexplained myocardial hypertrophy, which is best done using two-dimensional echocardiography. The diagnosis requires that measurements of wall thickness exceed two standard deviations for sex-, age-, and size-matched populations. In practice, in an adult of normal size, the presence of a left ventricular myocardial segment of 1.5 cm or greater in thickness, in the absence of a recognised cause, is usually considered to be diagnostic. Less stringent criteria should be applied to first-degree relatives of an affected individual, where the probability of carrying the disease gene increases from 1:500 to 1:2.

www.apnastudent.blogspot.com A patient with left ventricular failure undergoes echocardiography. Which is the correct formula for calculating the ejection fraction (EF)?

of 179

6/30/2011 4:56 PM

EF = [end diastolic volume (EDV) – end-systolic volume (ESV)]/EDV EF = [end diastolic volume (EDV) – end-systolic volume (ESV)]/heart rate (HR) EF = [heart rate (HR) × end

diastolic volume (EDV)]/end-systolic volume

(ESV)

EF = [heart rate (HR) × end-systolic volume (ESV)]/end diastolic volume

(EDV)

EF = [end-systolic volume (ESV) – end diastolic volume (EDV)]/EDV

A patient with left ventricular failure undergoes echocardiography. Which is the correct formula for calculating the ejection fraction (EF)? EF = [end diastolic volume (EDV) – end-systolic volume (ESV)]/EDV

Your answer

EF = [end diastolic volume (EDV) – end-systolic volume (ESV)]/heart rate (HR) EF = [heart rate (HR) × end

diastolic volume (EDV)]/end-systolic volume

(ESV)

EF = [heart rate (HR) × end-systolic volume (ESV)]/end diastolic volume

(EDV)

EF = [end-systolic volume (ESV) – end diastolic volume (EDV)]/EDV

Ejection fraction (EF) is calculated using the following equation: EF = [end diastolic volume (EDV) - end-systolic volume (ESV)] / EDV

A 69-year-old man has been admitted to the emergency department with syncope. He felt hot, complained of nausea and then fainted. His electrocardiogram (ECG) was normal. His brother suffers from adult onset epilepsy. What is the most appropriate investigation? Electroencephalogram (EEG) 24-h ECG Computed tomography (CT) of the brain Echocardiography Tilt test A 69-year-old man has been admitted to the emergency department with syncope. He felt hot, complained of nausea and then fainted. His electrocardiogram (ECG) was normal. His brother suffers from adult onset epilepsy. What is the most appropriate investigation? Electroencephalogram (EEG) 24-h ECG

Your answer

Computed tomography (CT) of the brain Echocardiography Tilt test

Correct answer

The development of tilt testing has allowed the study of the pathophysiology of neurocardiogenic syncope. The patient is strapped to a tilt-table and is tilted, head upright, usually at 70 degrees for up to 45 min. Protocols that use additional provocation with isoprenaline or nitrates are also commonly used. Blood pressure and cardiac rhythm are monitored throughout the tilt test. In neurocardiogenic syncope, the patient classically maintains normal blood pressure initially, until the sudden onset of syncope is associated with severe hypotension and bradycardia, often preceded by tachycardia. These features resolve with return to the supine posture. Some patients have a mainly vasodepressor response, with hypotension and little change in heart rate, while others have a marked cardioinhibitory response, with severe bradycardia or asystole of several seconds’ duration. However, most patients exhibit a mixed response, and those

www.apnastudent.blogspot.com patients with marked cardioinhibition also have a preceding vasodepressor response. This is an important observation when treatment is considered, since permanent pacing to maintain cardiac rhythm may not cure all symptoms, because falls in blood pressure may still occur even when bradycardia is prevented. I n this case the factor of the family history of adult-onset epilepsy is irrelevant given the clinical picture of syncope.

of 179

6/30/2011 4:56 PM

/

A 62-year-old patient presents with atrial fibrillation of unknown duration. Which drug may slow his ventricular rate over a prolonged period but is unlikely to result in cardioversion? Adenosine Amlodipine Digoxin Flecanide Amiodarone A 62-year-old patient presents with atrial fibrillation of unknown duration. Which drug may slow his ventricular rate over a prolonged period but is unlikely to result in cardioversion? Adenosine Amlodipine Digoxin

Your answer

Flecanide Amiodarone

+

+

Digoxin has inotropic actions based on inhibition of cardiac Na K ATPase; the antiarrhythmic activity appears to be mediated predominantly through vagal stimulation. Digoxin is used to slow ventricular rate in atrial fibrillation. Adenosine will reveal underlying atrial tachycardia but is unlikely to result in cardioversion versus flecanide.

Which of the following statements are most indicative of myocardial ischaemia? Associated shortness of breath Claudication Dizziness Radiation to jaw Relief by glyceryl trinitrate Which of the following statements are most indicative of myocardial ischaemia? Associated shortness of breath

Your answer

Claudication Dizziness Radiation to jaw

Correct answer

Relief by glyceryl trinitrate

The pain of angina usually radiates out from the chest, the commonest sites of radiation include the following. The neck and throat, causing a feeling of choking, strangulation or suffocation. The jaw, and maybe interpreted as toothache or problems with dentures. Down one or both arms: this is usually felt down the inside, under the axilla to the inner two fingers. By contrast muscular pain usually runs over the shoulder and down the outside of the arm. Other sites include the abdomen, the back, and areas of previous injury. Angina is often misinterpreted as indigestion.

Which is the best clinical marker of the severity of aortic stenosis?

www.apnastudent.blogspot.com Character of apex beat Character of carotid pulse

of 179

6/30/2011 4:56 PM

Character of S2 Intensity of murmur Pulse rate Which is the best clinical marker of the severity of aortic stenosis? Character of apex beat Character of carotid pulse Character of S2

Your answer

Intensity of murmur Pulse rate

Physical findings of aortic stenosis may include a narrow pulse pressure, especially when stroke volume decreases, and a slow-rising, small-volume carotid pulse. However, the poorly compliant arterial wall may mask these abnormalities, so that the carotid pulse appears relatively normal. The cardiac apex impulse is forceful and sustained, but this finding may be masked by kyphosis (in which the anteroposterior diameter of the chest is increased). The first heart sound is soft. The aortic component of the second heart sound is also soft; it may be inaudible when stenosis is severe and the valve is heavily calcified. Reverse splitting of the second heart sound may occur in patients with left ventricular failure. A fourth heart sound is common but disappears in one-quarter of elderly patients who develop atrial fibrillation. Ejection sounds are rare because the valve cusps are immobile.

A 55-year-old man who has sustained an acute MI subsequently presents with heart failure. As well as other treatments the cardiologist has recommended that abeta-blocker be commenced. According to currently available evidence which of the following beta-blockers would be most appropriate? Celiprolol Labetalol Bisoprolol Propranolol Sotalol A 55-year-old man who has sustained an acute MI subsequently presents with heart failure. As well as other treatments the cardiologist has recommended that abeta-blocker be commenced. According to currently available evidence which of the following beta-blockers would be most appropriate? Celiprolol Labetalol Bisoprolol

Your answer

Propranolol Sotalol

Beta-blockers may produce benefit in heart failure by blocking sympathetic activity. Bisoprolol and carvedilol reduce mortality in any grade of stable heart failure. Treatment should be initiated by those experienced in the management of heart failure. According to currently available evidence (see NICE guidelines on cardiac failure, 2003), bisoprolol, metoprolol sustained release and carvedilol have shown the most useful effects. At present metoprolol is not licensed in the UK for this indication and so carvedilol or bisoprolol are the preferred choices.

A 67-year-old lady is found to have a small pericardial effusion located posteriorly on routine echocardiography. There is no

www.apnastudent.blogspot.com haemodynamic compromise. What is the next most appropriate step in her management?

of 179

6/30/2011 4:56 PM

Diagnostic tap Mammography Tuberculosis screen Reassure Right heart catheter examination A 67-year-old lady is found to have a small pericardial effusion located posteriorly on routine echocardiography. There is no haemodynamic compromise. What is the next most appropriate step in her management? Diagnostic tap Mammography Tuberculosis screen Reassure

Your answer

Right heart catheter examination

Once the diagnosis of pericardial effusion has been made, it is important to determine whether the effusion is creating significant haemodynamic compromise. Asymptomatic patients without haemodynamic compromise, even with large pericardial effusions, do not need to be treated with pericardiocentesis unless there is a need for fluid analysis for diagnostic purposes (eg, in acute bacterial pericarditis, tuberculosis, and neoplasias).

Which of the following is the commonest cardiovascular abnormality seen in an adult patient with Marfan’s syndrome? Aortic regurgitation Aortic root dilatation Mitral regurgitation Mitral annular calcification Aortic dissection Which of the following is the commonest cardiovascular abnormality seen in an adult patient with Marfan’s syndrome? Aortic regurgitation Aortic root dilatation

Your answer

Mitral regurgitation Mitral annular calcification Aortic dissection

Marfan’s syndrome is a connective tissue disorder that is inherited as an autosomal-dominant trait. There is considerable variation in its clinical manifestations. The ocular (dislocation of the lens), skeletal (arachnodactyly, joint hypermobility, scoliosis, chest deformity and high arched palate) and cardiovascular systems (aortic root dilatation (70%) and mitral valve prolapse (60%)) are characteristically involved. Weakening of the aortic media leads to a fusiform ascending aortic aneurysm, which may be complicated by aortic regurgitation and aortic dissection. Mitral regurgitation can result from mitral valve prolapse, dilatation of a mitral valve annulus or mitral annular calcification. Pregnancy is particularly hazardous. Treatment with risk of rupture.

β-blockers reduces the

rate of aortic dilatation and the

A 39-year-old female is admitted with pulmonary oedema, blood pressure 195/134 mmHg and fundoscopy showing retinal

www.apnastudent.blogspot.com haemorrhages but not papilloedema. She has systemic sclerosis and asthma. Which of the following agents would be the most appropriate immediate management?

of 179

6/30/2011 4:56 PM

Intravenous labetalol Intravenous sodium nitroprusside Atenolol Nifedipine oral Nifedipine sublingual A 39-year-old female is admitted with pulmonary oedema, blood pressure 195/134 mmHg and fundoscopy showing retinal haemorrhages but not papilloedema. She has systemic sclerosis and asthma. Which of the following agents would be the most appropriate immediate management? Intravenous labetalol Intravenous sodium nitroprusside Atenolol Nifedipine oral

Your answer

Nifedipine sublingual

The malignant phase of hypertension is a rare condition characterised by very high blood pressures, with bilateral retinal haemorrhages and/or exudates or cotton wool spots, without the added requirement for papilloedema. All patients with malignant hypertension should be admitted for assessment, investigation, and commencement of therapy under supervision. The initial aim of treatment is to lower the diastolic pressure to about 100–105 mmHg over a period of 2–3 days, with oral therapy and dose escalation at daily intervals if necessary. The maximum initial fall in blood pressure should not exceed 25% of the presenting value. Blood pressure should be measured 4-hourly. The first-line oral antihypertensive agent is either a short-acting calcium antagonist (such as nifedipine) or a b-blocker (such as atenolol). Beta-blockers are contraindicated in asthma. An appropriate dose of nifedipine is 10–20 mg of the tablet formulation, which can be repeated or increased as necessary to bring about gradual reduction in blood pressure. Nifedipine is not absorbed from the oral mucosa, and there have been reports of complications including visual loss, cerebral infarction, and myocardial infarction with nifedipine therapy using the short-acting sublingual capsules. Sublingual nifedipine produces unpredictable falls in blood pressure and should never be used. ACE inhibitors and Angiotensin II receptor antagonist are beneficial in the treatment of renal failure in patients with scleroderma.

A 67-year-old lady during pre-operative assessment is found to have a small pericardial effusion located posteriorly on routine Echocardiography. Electrocardiogram (ECG) is entirely normal. What is the next most appropriate step in her management? Cardiac catheterisation Reassure Pericardiocentesis Diuretics Computed tomography (CT) of the heart A 67-year-old lady during pre-operative assessment is found to have a small pericardial effusion located posteriorly on routine Echocardiography. Electrocardiogram (ECG) is entirely normal. What is the next most appropriate step in her management? Cardiac catheterisation Reassure Pericardiocentesis Diuretics Computed tomography (CT) of the heart

Your answer

www.apnastudent.blogspot.com Once the diagnosis of pericardial effusion has been made, it is important to determine whether the effusion is creating significant haemodynamic compromise. Asymptomatic patients without haemodynamic compromise, even with large

of 179

6/30/2011 4:56 PM

pericardial effusions, do not need to be treated with pericardiocentesis unless there is a need for fluid analysis for diagnostic purposes (eg in acute bacterial pericarditis, tuberculosis, and neoplasias).

An elderly gentleman is admitted with syncope. He also complains of shortness of breath and is diagnosed as having aortic stenosis. Which of the following conditions when associated with aortic stenosis would indicate a poor prognosis? Aortic regurgitation Left ventricular failure Electrocardiography (ECG) changes Endocarditis Valvular calcification

An elderly gentleman is admitted with syncope. He also complains of shortness of breath and is diagnosed as having aortic stenosis. Which of the following conditions when associated with aortic stenosis would indicate a poor prognosis? Aortic regurgitation Left ventricular failure

Your answer

Electrocardiography (ECG) changes Endocarditis Valvular calcification

The prognosis of symptomatic aortic stenosis is poor with a 50% survival of only 1 to 2 years. Approximately half the deaths are due to relentless haemodynamic deterioration, and the remainder are sudden and unexpected. The prognosis of asymptomatic but haemodynamically severe aortic stenosis is somewhat better. However, older patients with a peak velocity of 4 m/s or more across the aortic valve are likely to become symptomatic in a period of 2 years or less.

A 46-year-old man is admitted with shortness of breath, headache, blood pressure 190/110 mmHg and fundoscopy showing retinal haemorrhages and papilloedema. ECG revealed left ventricular hypertrophy, chest x-ray did not reveal any evidence of left ventricular failure. Which of the following agents would be the most appropriate management? Intravenous labetalol Intravenous sodium nitroprusside Atenolol Doxasosin MR oral Nifedipine sublingual A 46-year-old man is admitted with shortness of breath, headache, blood pressure 190/110 mmHg and fundoscopy showing retinal haemorrhages and papilloedema. ECG revealed left ventricular hypertrophy, chest x-ray did not reveal any evidence of left ventricular failure. Which of the following agents would be the most appropriate management? Intravenous labetalol Intravenous sodium nitroprusside Atenolol Doxasosin MR oral Nifedipine sublingual

Your answer

www.apnastudent.blogspot.com

The malignant phase of hypertens ion is a rare condition characterised by very high blood pressures, with bilateral

of 179

6/30/2011 4:56 PM

retinal haemorrhages and/or exudates or cotton wool spots, without the added requirement for papilloedema. All patients with malignant hypertension should be admitted for assessment, investigation, and commencement of therapy under supervision. The initial aim of treatment is to lower the diastolic pressure to about 100–105 mmHg over a period of 2 to 3 days, with oral therapy and dose escalation at daily intervals if necessary. The maximum initial fall in blood pressure should not exceed 25% of the presenting value. Blood pressure should be measured 4-hourly. The first-line oral antihypertensive agent is either a calcium antagonist (such as nifedipine) or a b-blocker (such as atenolol). Standard guidance is to use modified release nifedipine or a long-acting compound like amlodipine although some clinicians suggest using a standard oral formulation to achieve dose titration. Nifedipine is not absorbed from the oral mucosa, and there have been reports of complications including visual loss, cerebral infarction, and myocardial infarction with nifedipine therapy using the short-acting sublingual capsules. Sublingual nifedipine produces unpredictable falls in blood pressure and should never be used.

A patient with underlying ischaemic heart disease had two transient episodes of loss of consciousness but feels fine at present. Both episodes were preceded by a feeling of dizziness, "vision going black" and witnesses report that the subject went very pale and then collapsed, lying motionless for a few seconds before making a rapid recovery. No abnormal movements were seen during the period of unconsciousness. What investigation will you order next? Echocardiography Computed tomography (CT) of the head 24-hour electrocardiogram (ECG) Cardiac catheterisation Treadmill test A patient with underlying ischaemic heart disease had two transient episodes of loss of consciousness but feels fine at present. Both episodes were preceded by a feeling of dizziness, "vision going black" and witnesses report that the subject went very pale and then collapsed, lying motionless for a few seconds before making a rapid recovery. No abnormal movements were seen during the period of unconsciousness. What investigation will you order next? Echocardiography Computed tomography (CT) of the head 24-hour electrocardiogram (ECG)

Your answer

Cardiac catheterisation Treadmill test

The key in assessing any episode of loss of consciousness is a detailed history including eye-witness descriptions. This is necessary to try to clinically distinguish between the many different possible aetiologies of such an occurrence. In this case the pre-syncopal symptoms, as well as the brief nature of the attack, pallor, lack of convulsions and prior cardiac history are all in favour of a cardiac cause. Loss of consciousness of cardiac origin may result from abnormalities of heart rhythm, due to extremes of rate, either fast or slow, or from some major disturbance of cardiovascular function, with resultant reduced cerebral perfusion. The importance in establishing the diagnosis of cardiac syncope is the associated adverse prognosis, which may be improved with appropriate treatment. The probability of cardiac syncope is increased in the presence of structural cardiovascular disease identified from the history, clinical examination, or investigation. Syncope is defined as a transient loss of consciousness with the loss of postural tone, and is most commonly due to cardiovascular mechanisms resulting in reduced cerebral perfusion. It is a common presentation, resulting in 1–2% of emergency department visits and up to 6% of hospital admissions. The cause is often initially uncertain, and assessment must first differentiate syncope from other causes of loss of consciousness, in particular epileptic seizures. The next priority is to identify high-risk patients. Documentation of cardiac rhythm during syncope is desirable, but is difficult to obtain because of the intermittent and usually infrequent nature of the symptom. Holter monitoring is unlikely to record the rhythm during an episode, but may provide evidence of lesser degrees of abnormality, which may support a diagnosis such as sinoatrial dysfunction.

A 65-year-old male patient with stable angina complains of shortness of breath after walking two flights of stairs. He has normal left ventricular function on the echocardiogram and a positive exercise tolerance test (3 mm ST depression at stage II I). What is the most appropriate therapy?

www.apnastudent.blogspot.com Atenolol Simvastatin Isosorbide mononitrate Angiotensin-converting enzyme (ACE) inhibitor Nicardipine A 65-year-old male patient with stable angina complains of shortness of breath after walking two flights of stairs. He has normal left ventricular function on the echocardiogram and a positive exercise tolerance test (3 mm ST depression at stage II I). What is the most appropriate therapy? Atenolol

Your answer

Simvastatin Isosorbide mononitrate Angiotensin-converting enzyme (ACE) inhibitor Nicardipine

Beta-blocking agents are the cornerstone of the pharmacological management of chronic angina pectoris. They are well tolerated and reduce the frequency and duration of anginal episodes and improve exercise tolerance. They are also effective antihypertensive agents and prevent some arrhythmias. They act by competitively inhibiting catecholamine effects on the β-adrenergic receptor. This reduces heart rate and improves coronary perfusion (by prolonging diastole), thereby reducing an exercise-induced rise in blood pressure and contractility.

A 65-year-old female patient with severe heart failure presents with increasing shortness of breath. Her current pharmacological treatment consists of an angiotensin-converting enzyme (ACE) inhibitor, loop diuretic and is the most appropriate management?

β-blocker.

What

Add digoxin Add spironolactone Stop b-blocker Stop ACE inhibitor Add simvastatin

A 65-year-old female patient with severe heart failure presents with increasing shortness of breath. Her current pharmacological treatment consists of an angiotensin-converting enzyme (ACE) inhibitor, loop diuretic and is the most appropriate management?

β-blocker.

What

Add digoxin Add spironolactone

Your answer

Stop b-blocker Stop ACE inhibitor Add simvastatin

The positive inotropic effects of cardiac glycosides can be useful in reducing symptoms (mainly breathlessness) in patients already taking diuretics and angiotensin-converting enzyme (ACE) inhibitors, although they have no effect on mortality and must be used with caution in patients such as this who are prone to disturbances of potassium. Low dose spironolactone has however been shown to have positive effects on cardiovascular mortality in patients with end stage heart failure. Large outcome studies with carvedilol and bisprolol have also shown a mortality benefit in heart failure.

of 179

6/30/2011 4:56 PM

A 22-year-old cocaine addict presents with an acute myocardial infarction. His blood pressure is 180/110 mmHg. Which is the most appropriate treatment?

www.apnastudent.blogspot.com Thrombolysis Heparin Percutaneous coronary intervention Naloxone Glycoprotein 2b/3a inhibitors A 22-year-old cocaine addict presents with an acute myocardial infarction. His blood pressure is 180/110 mmHg. Which is the most appropriate treatment? Thrombolysis Heparin Percutaneous coronary intervention

Your answer

Naloxone Glycoprotein 2b/3a inhibitors

Cocaine use has recently been implicated as a cause of unstable angina. Three possible mechanisms by which cocaine induces myocardial ischemia are: (1) increased myocardial oxygen demand, (2) decreased myocardial oxygen supply secondary to vasospasm or coronary thrombosis, and (3) direct myocardial toxicity. Documented cocaine use should not be considered to rule out underlying significant coronary artery disease (CAD), since the drug may precipitate coronary vasospasm or acute myocardial infarction in the patient with atherosclerotic CAD. Where urgent angioplasty is available, this is preferable to thrombolysis as outcome studies show it to be superior.

A patient presents to the emergency department with severe chest pain, what are the indications for thrombolysis? Q waves in any two leads 1 mm ST depression in 1 chest lead 1 mm ST depression in 2 limb leads Ebstein’s anomaly 1 mm ST elevation in 2 limb leads A patient presents to the emergency department with severe chest pain, what are the indications for thrombolysis? Q waves in any two leads 1 mm ST depression in 1 chest lead 1 mm ST depression in 2 limb leads Ebstein’s anomaly 1 mm ST elevation in 2 limb leads

Your answer

Acute injury ST elevation of 1 mm in two or more limb leads, or 2 mm or more in two precordial leads, not suggestive of early repolarisation, pericarditis or repolarisation abnormality from left ventricular hypertrophy or bundle branch block, require immediate reperfusion therapy. Fast-track systems in hospitals have been developed to minimise the delay of thrombolysis; these are facilitated by specifically trained medical and nursing staff, with the aim of ensuring clinical assessment and electrocardiography within 15 min of arrival and the institution of thrombolytic therapy within 30 min. Audit programmes and continuous training are necessary for centres to achieve this 30 min median door-to-needle time. Prior to the advent of fast-track systems, door-to-needle times of between 60 and 90 min were frequently recorded in clinical trials and in observational studies.

of 179

6/30/2011 4:56 PM

A 16-year-old girl presents to the Emergency Department with a collapse and palpitations after attending her end-of-term school disco. Only medication history of note includes a recent antibiotic prescription for an infected toe. Past medical history includes allergy to penicillin. Family history reveals that her mother died suddenly at the age of 34 when the daughter was 3 years old. One aunt and one uncle have also passed away suddenly. Electrocardiogram (ECG) reveals sinus

www.apnastudent.blogspot.com rhythm in the Emergency Department but the QT interval is prolonged at 550 ms (corrected). Which of the following conditions is most likely to be related to her collapse? Wolff–Parkinson–White type A Wolff–Parkinson–White type B Congenital long QT syndrome Lown–Ganong–Levine syndrome Ebstein's anomaly A 16-year-old girl presents to the Emergency Department with a collapse and palpitations after attending her end-of-term school disco. Only medication history of note includes a recent antibiotic prescription for an infected toe. Past medical history includes allergy to penicillin. Family history reveals that her mother died suddenly at the age of 34 when the daughter was 3 years old. One aunt and one uncle have also passed away suddenly. Electrocardiogram (ECG) reveals sinus rhythm in the Emergency Department but the QT interval is prolonged at 550 ms (corrected). Which of the following conditions is most likely to be related to her collapse? Wolff–Parkinson–White type A Wolff–Parkinson–White type B Congenital long QT syndrome

Your answer

Lown–Ganong–Levine syndrome Ebstein's anomaly

The QT prolongation and history of sudden death in the family suggests the possibility of congenital long QT syndrome. Her presentation may have occurred because QT interval can be prolonged in association with a number of medications that include erythromycin (prescribed for her foot), ketoconazole, antihistamines, anti-arrhythmics and a number of other agents. The condition is associated with torsades de pointe ventricular tachycardia. Lange–Nielsen syndrome is one syndrome of QT prolongation, which has been described in association with congenital deafness; Romano–Ward syndrome is associated with normal hearing. The Lown-Ganong-Levine syndrome (LGL) is usually considered in a class of preexcitation syndromes that includes the Wolff-Parkinson-White syndrome (WPW), LGL, and Mahaim-type preexcitation. Theories proposed to explain LGL have centered around the possible existence of intranodal or paranodal fibers that bypass all or part of the atrioventricular (AV) node. Criteria for LGL include PR interval less than or equal to 0.12 second (120 ms), normal QRS complex duration, and occurrence of supraventricular tachycardia but not atrial fibrillation or atrial flutter.

An elderly man is admitted to the ICU and put on intermittent positive-pressure ventilation. Which of the following statements is true when compared to spontaneous ventilation? Lung volumes are decreased Pulmonary vascular resistance is decreased Systemic blood pressure rises Venous return and cardiac output fall Intrathoracic pressure is decreased An elderly man is admitted to the ICU and put on intermittent positive-pressure ventilation. Which of the following statements is true when compared to spontaneous ventilation? Lung volumes are decreased Pulmonary vascular resistance is decreased Systemic blood pressure rises Venous return and cardiac output fall

Your answer

Intrathoracic pressure is decreased

of 179

6/30/2011 4:56 PM

During intermittent positive-pressure ventilation (IPPV), lung volumes are significantly increased when compared to spontaneous ventilation. A large tidal volume causes a rise in pulmonary vascular resistance, which may lead to pulmonary hypertension and right ventricular compromise. The over-inflated alveoli cause compression of the alveolar blood vessels. Moreover, the resultant increase in RV volume may impede LV filling (ventricular interdependence). Hyperinflation also releases prostaglandins, which decrease blood pressure. The intrathoracic pressure is increased at all points in the respiratory cycle. Inspiration during IPPV increases intrathoracic pressure and so increases right atrial pressure relative to atmospheric pressure, therefore leading to decreased venous return. The increased intrathoracic pressure also decreases the gradient across the LV that it has to work against, which results in a decreased afterload. Both these effects reduce intrathoracic blood volume.

You review a 68-year-old woman who presents with a sudden episode of collapse while taking communion in church. This has been her third syncopal episode. Past medical history of note includes recently diagnosed severe hypertension, for which her GP has commenced enalapril therapy. On examination her blood pressure is 160/130 mmHg, she has left ventricular hypertrophy on clinical examination and a loud ejection systolic murmur. Auscultation of the chest reveals bibasilar crackles consistent with mild heart failure. Which of the following is the definitive investigation of choice for this patient? Chest X-ray Electrocardiogram (ECG) Echocardiogram Cardiac catheterisation 24 h holter monitor You review a 68-year-old woman who presents with a sudden episode of collapse while taking communion in church. This has been her third syncopal episode. Past medical history of note includes recently diagnosed severe hypertension, for which her GP has commenced enalapril therapy. On examination her blood pressure is 160/130 mmHg, she has left ventricular hypertrophy on clinical examination and a loud ejection systolic murmur. Auscultation of the chest reveals bibasilar crackles consistent with mild heart failure. Which of the following is the definitive investigation of choice for this patient? Chest X-ray Electrocardiogram (ECG) Echocardiogram

Your answer

Cardiac catheterisation

Correct answer

24 h holter monitor

This patient is suffering from symptomatic aortic stenosis as evidenced by the history of syncope, hypertension, left ventricular hypertrophy and harsh ejection systolic murmur. While echocardiography will aid in diagnosis, gradient across the aortic valve may be underestimated because of the possibility of multiple echo signals and co-existent left ventricular dysfunction. As such cardiac catheterisation is the definitive investigation as it allows for more accurate estimation of valve gradient and characterisation of co-existent coronary artery disease, which may require intervention at the same time.

You are called urgently to review a 54-year-old man who has developed acute onset pulmonary oedema some 36 h after his myocardial infarction. On arrival you note that his blood pressure is 95/50 mmHg with a pulse of 100/min regular and a pan-systolic murmur is noted. There are crackles on auscultation of the chest consistent with heart failure. Which of the following represents the next investigation of choice in this man? Troponin I Troponin

www.apnastudent.blogspot.com T Urgent chest X-ray

of 179

6/30/2011 4:56 PM

Referral for angiography Urgent echocardiogram You are called urgently to review a 54-year-old man who has developed acute onset pulmonary oedema some 36 h after his myocardial infarction. On arrival you note that his blood pressure is 95/50 mmHg with a pulse of 100/min regular and a pan-systolic murmur is noted. There are crackles on auscultation of the chest consistent with heart failure. Which of the following represents the next investigation of choice in this man? Troponin I Troponin T Urgent chest X-ray Referral for angiography Urgent echocardiogram

Your answer

The timing of this man’s deterioration coupled with a murmur of mitral regurgitation and acute pulmonary oedema suggests the onset of papillary muscle dysfunction or even rupture. Echocardiogram is the investigation of choice to demonstrate the mitral regurgitation. Management involves the use of vasodilators such as sodium nitroprusside and the use of angiotensin-converting enzyme (ACE) inhibition if tolerated. Inotropic support with drugs such as dopamine or dobutamine may also be required. The case should be discussed with cardiothoracic surgical colleagues to assess suitability for surgical repair, although this should be postponed until after haemodynamic stabilisation if possible due to the high risks of peri-infarct surgical intervention.

A 32-year-old woman who is known to be 17 weeks’ pregnant presents for review. She has periods of paroxysmal supraventricular tachycardia (SVT) and on this occasion has a ventricular rate of 165/min and a blood pressure of 90/50 mmHg, feeling faint and unwell. Which of the following anti-arrhythmics would be the most appropriate prophylaxis for her? Metoprolol Amiodarone Digoxin Phenytoin Propafenone A 32-year-old woman who is known to be 17 weeks’ pregnant presents for review. She has periods of paroxysmal supraventricular tachycardia (SVT) and on this occasion has a ventricular rate of 165/min and a blood pressure of 90/50 mmHg, feeling faint and unwell. Which of the following anti-arrhythmics would be the most appropriate prophylaxis for her? Metoprolol

Correct answer

Amiodarone

Your answer

Digoxin Phenytoin Propafenone

This patient has paroxysmal supraventricular tachycardia (SVT). While digoxin slows the ventricular rate in patients with chronic atrial fibrillation, it does not maintain sinus rhythm in patients with paroxysmal tachycardia. Amiodarone is known to be teratogenic and is contraindicated in pregnancy. Guidelines suggest that metoprolol is potentially the most

www.apnastudent.blogspot.com appropiate option for SVT in pregnancy.

You are asked to review a 19-year-old woman who presents with increasing shortness of breath on exercise. She is from a travelling family and has rarely encountered medical care. On examination she appears of short stature with extra skin folds around her neck, and appears to have failure of secondary sexual development. Her blood pressure is raised at 165/100 mmHg. She reports that her legs feel tired all the time and she has occasional chest pain on exercising.

of 179

6/30/2011 4:56 PM

Which of the following cardiac diagnoses fits best with her clinical condition? Pulmonary stenosis Mitral regurgitation Coarctation of the aorta Aortic regurgitation Hypertrophic obstructive cardiomyopathy You are asked to review a 19-year-old woman who presents with increasing shortness of breath on exercise. She is from a travelling family and has rarely encountered medical care. On examination she appears of short stature with extra skin folds around her neck, and appears to have failure of secondary sexual development. Her blood pressure is raised at 165/100 mmHg. She reports that her legs feel tired all the time and she has occasional chest pain on exercising. Which of the following cardiac diagnoses fits best with her clinical condition? Pulmonary stenosis Mitral regurgitation Coarctation of the aorta

Your answer

Aortic regurgitation Hypertrophic obstructive cardiomyopathy

Given this patient’s clinical picture it appears that she has Turner’s syndrome. I t is likely that, because she is from a travelling family, she has only had irregular access to medical services. Turner’s syndrome is associated with coarctation of the aorta, and bicuspid aortic stenosis may also co-exist. Symptoms include vigorous pulsation in the neck or throat, hypertension, tired legs or intermittent claudication on running, left ventricular failure, angina pectoris, and aortic rupture. Physical signs to note include radiofemoral pulse delay, development of collateral vessels, left ventricular failure, and ejection systolic murmur. Electrocardiogram (ECG) shows left ventricular hypertrophy and right bundle branch block is common.

A 41-year-old man with a family history of sudden death presents to casualty with a second episode of collapse. On this occasion he is referred to the Cardiology Department for review. Echocardiography reveals asymmetrical septal hypertrophy, abnormal systolic motion of the anterior mitral valve leaflet and narrowing of the left ventricular outflow tract. The 24-h electrocardiogram (ECG) monitoring as an outpatient reveals several periods of non-sustained ventricular tachycardia. Which of the following would be most appropriate for the management of his arrhythmia? Oral flecainide 100 mg daily Oral amiodarone 200 mg tds Oral amiodarone 200 mg daily Implantable cardioverter defibrillator Phenytoin 100 mg po daily A 41-year-old man with a family history of sudden death presents to casualty with a second episode of collapse. On this occasion he is referred to the Cardiology Department for review. Echocardiography reveals asymmetrical septal hypertrophy, abnormal systolic motion of the anterior mitral valve leaflet and narrowing of the left ventricular outflow tract. The 24-h electrocardiogram (ECG) monitoring as an outpatient reveals several periods of non-sustained ventricular tachycardia. Which of the following would be most appropriate for the management of his arrhythmia? Oral flecainide 100 mg daily Oral amiodarone 200 mg tds Oral amiodarone 200 mg daily Implantable cardioverter defibrillator Phenytoin 100 mg po daily

Your answer

www.apnastudent.blogspot.com

Given this clinical history, this patient has a diagnosis of hypertrophic obstructive cardiomyopathy. He has

of 179

6/30/2011 4:56 PM

non-sustained ventricular tachycardias and is at significant risk of sudden death. While myomectomy will impact on symptoms, it won’t affect the risk of arrhythmia. Management in the past would have been medical therapy with amiodarone, but recent studies have demonstrated superior efficacy for implantable cardioverter defibrillators (I CDs). Given that long-term use of amiodarone is associated with significant morbidity, ICDs are taking over as management of choice.

A 72-year-old man presents for an extraction of three teeth under local anaesthesia. He has a past history of rheumatic heart disease. Mitral stenosis has been identified but the rotten teeth are being removed before valve replacement. He is allergic to penicillin. Which of the following would be the most appropriate antibiotic regime for him? Amoxicillin 3 g po 1 hour before procedure No prophylaxis necessary Vancomycin 1 g po 1 hour before procedure Ciprofloxacin 1 g po 1 hour before procedure Augmentin 1 g po before procedure A 72-year-old man presents for an extraction of three teeth under local anaesthesia. He has a past history of rheumatic heart disease. Mitral stenosis has been identified but the rotten teeth are being removed before valve replacement. He is allergic to penicillin. Which of the following would be the most appropriate antibiotic regime for him? Amoxicillin 3 g po 1 hour before procedure No prophylaxis necessary

Your answer

Vancomycin 1 g po 1 hour before procedure Ciprofloxacin 1 g po 1 hour before procedure Augmentin 1 g po before procedure

After an extensive evidence review, the most recent NICE guidelines do not recommend antibiotic prophylaxis for a number of routine procedures including dental extraction. Their conclusion is based on the fact that they were unable to determine any evidence which suggested that antibiotic prophylaxis prevented the development of endocarditis.

A 45-year-old man attends for review. He has been suffering increasing shortness of breath over the past few years. He is a non-smoker who drinks 20 units per week of alcohol and has no significant past cardiovascular history. Now he presents with what seems to have been a transient ischaemic attack (TI A), with weakness and co-ordination problems affecting his left side, which have resolved over the past 24 hours. On examination blood pressure is 142/95 mmHg and he is in sinus rhythm. There is no opening snap, but a diastolic murmur is heard which changes in character according to posture. Bloods are unremarkable, including C-reactive protein (CRP), which is in the normal range. Which of the following diagnoses fit best with this clinical picture? Right atrial myxoma Left atrial myxoma Aortic stenosis Mitral stenosis Mitral regurgitation A 45-year-old man attends for review. He has been suffering increasing shortness of breath over the past few years. He is a non-smoker who drinks 20 units per week of alcohol and has no significant past cardiovascular history. Now he presents with what seems to have been a transient ischaemic attack (TI A), with weakness and co-ordination problems affecting his left side, which have resolved over the past 24 hours. On examination blood pressure is 142/95 mmHg and he is in sinus rhythm. There is no opening snap, but a diastolic murmur is heard which changes in character according to posture. Bloods are unremarkable, including C-reactive protein (CRP), which is in the normal range.

www.apnastudent.blogspot.com Which of the following diagnoses fit best with this clinical picture? Right atrial myxoma

of 179

6/30/2011 4:56 PM

Left atrial myxoma

Your answer

Aortic stenosis Mitral stenosis Mitral regurgitation

This patient has suffered a TIA, most likely due to embolus from an intra-cardiac cause. One possible clinical explanation could be mitral stenosis, left atrial enlargement and atrial fibrillation, leading to clot formation within the atrium, but this patient is in sinus rhythm, there is no opening snap on auscultation, and the murmur changes in character with posture. This suggests the possibility of another cause, and left atrial myxoma would fit the bill. Myxoma can occur in any cardiac chamber, but occurs most commonly in the left atrium. It is a gelatinous, friable tumour, which leads to transient signs of mitral stenosis that only occur if the tumour approaches the mitral valve orifice. There is no opening snap. There may be an early diastolic plop as the tumour prolapses through the mitral valve. X-ray may show calcification within the tumour if it is long standing. Definitive treatment involves surgical excision; recurrence rate is extremely low, but follow up is recommended for a period of 5 years. Right atrial myxomas are more rare and difficult to identify clinically; there may be evidence of multiple pulmonary infarcts due to formation of emboli.

A 75-year-old man is referred for total hip replacement. He has a history of hypertension and angina and has suffered a myocardial infarction some 8 years earlier. Current medication includes atenolol 50 mg daily, ramipril 10 mg daily, aspirin 75 mg daily and isosorbide dinitrate 60 mg. Blood pressure at the preoperative assessment was 160/80 mmHg but he maintains that his readings with the general practitioner have been normal. He last had an exercise test some 3 years earlier and managed 8 min with no significant electrocardiogram (ECG) changes. Which one of the following investigations in addition to standard assessment would be most appropriate for the preoperative assessment of this patient? Repeat exercise ECG test Routine echocardiogram 99

Tc

m

MIBI SPECT scan

Stress ECG Magnetic resonance angiography A 75-year-old man is referred for total hip replacement. He has a history of hypertension and angina and has suffered a myocardial infarction some 8 years earlier. Current medication includes atenolol 50 mg daily, ramipril 10 mg daily, aspirin 75 mg daily and isosorbide dinitrate 60 mg. Blood pressure at the preoperative assessment was 160/80 mmHg but he maintains that his readings with the general practitioner have been normal. He last had an exercise test some 3 years earlier and managed 8 min with no significant electrocardiogram (ECG) changes. Which one of the following investigations in addition to standard assessment would be most appropriate for the preoperative assessment of this patient? Repeat exercise ECG test 99

Tc

m

Routine echocardiogram Your answer MIBI SPECT scan

Correct answer

Stress ECG Magnetic resonance angiography

From the history given it appears that this man has relatively stable angina and there seems little value to be gained from further imaging of his coronary arteries. Routine echocardiogram would add information about left ventricular function and point out any valvular disease. Given possible haemodynamic changes during total hip replacement, significant left venticule (LV) impairment or valvular disease may impact significantly on operative prognosis. His blood pressure does appear labile and it would be worthwhile to document that his normal blood pressure is within acceptable limits. If he has significant white coat hypertension, when he attends for surgery he may well be cancelled if his blood pressure is too high. For this reason, 24-h outpatient blood pressure recording would seem sensible.

www.apnastudent.blogspot.com A 64-year-old woman suffers from frequent and painful urinary tract infections. After her third course of antibiotics in the past 6 months she is advised by the GP to take cranberry juice supplements. Significant past medical history of note

of 179

6/30/2011 4:56 PM

includes hypertension for which she takes ramipril and bendroflumethiazide and hypercholesterolaemia for which she takes simvastatin. There is also a history of paroxysmal atrial fibrillation for which she takes warfarin and amiodarone. Which of her medications is most likely to interact with the cranberry juice? Simvastatin Amiodarone Bendroflumethiazide Warfarin Ramipril

A 64-year-old woman suffers from frequent and painful urinary tract infections. After her third course of antibiotics in the past 6 months she is advised by the GP to take cranberry juice supplements. Significant past medical history of note includes hypertension for which she takes ramipril and bendroflumethiazide and hypercholesterolaemia for which she takes simvastatin. There is also a history of paroxysmal atrial fibrillation for which she takes warfarin and amiodarone. Which of her medications is most likely to interact with the cranberry juice? Simvastatin Amiodarone Bendroflumethiazide Warfarin

Your answer

Ramipril

Cranberry juice contains a number of bioflavinoids, some of which are thought to cause inhibition of the cytochrome p450 2C9 isoenzyme which is responsible for warfarin metabolism. Metabolism of simvastatin is inhibited by grapefruit juice. The warfarin interaction was given as a Committee for Safety of Medicines (CSM) warning in 2003 and features prominently in the MRCP examination.

A 63-year-old smoker is admitted with nausea, sweating and central crushing chest pain. A 12 lead ECG reveals ST elevation in leads II , III and aVF. Which coronary artery is most likely to have been affected in this case? Circumflex artery Left anterior descending artery Right coronary artery Obtuse marginal artery Posterolateral artery A 63-year-old smoker is admitted with nausea, sweating and central crushing chest pain. A 12 lead ECG reveals ST elevation in leads II , III and aVF. Which coronary artery is most likely to have been affected in this case? Circumflex artery

Your answer

Left anterior descending artery Right coronary artery

Correct answer

Obtuse marginal artery Posterolateral artery

The right coronary artery generally supplies the right ventricle, the posterior third of the interventricular septum, the inferior wall of the left ventricle and a portion of the posterior wall of the left ventricle. Occasionally the posterior

www.apnastudent.blogspot.com interventricular septum may be supplied by a branch of the of the left circumflex artery, a so called left dominant circulation. The left anterior descending and left circumflex coronary arteries arise at the left main coronary artery bifurcation and supply the anterior left ventricle, the bulk of the interventricular septum (anterior two thirds), the apex,

of 179

6/30/2011 4:56 PM

and the lateral and posterior left ventricular walls.

A 20-year-old-man attends A&E with palpitations described as ‘regular rapid beating of the heart’. An ECG shows a regular rhythm with a rate of 200 beats/minute and a QRS duration of 80 ms. The tachycardia spontaneously resolves. An ECG in sinus rhythm reveals a PR interval of 60 ms and a QRS duration of 120 ms with a positive delta in V 1. Which of the following statements is true regarding this man’s tachycardia? Carotid sinus massage will be ineffective Intravenous adenosine is of no use Digoxin should be used as a prophylactic agent Verapamil is contraindicated Atrial fibrillation is well tolerated in such patients A 20-year-old-man attends A&E with palpitations described as ‘regular rapid beating of the heart’. An ECG shows a regular rhythm with a rate of 200 beats/minute and a QRS duration of 80 ms. The tachycardia spontaneously resolves. An ECG in sinus rhythm reveals a PR interval of 60 ms and a QRS duration of 120 ms with a positive delta in V 1. Which of the following statements is true regarding this man’s tachycardia? Carotid sinus massage will be ineffective Intravenous adenosine is of no use Digoxin should be used as a prophylactic agent Verapamil is contraindicated

Your answer

Atrial fibrillation is well tolerated in such patients

This man has type A Wolff–Parkinson–White (WPW) syndrome, in which an abnormal band of atrial tissue connects the atria to the ventricle to bypass the atrioventricular (AV) node. As the AV node and bypass tract have different conduction speeds and refractory periods, a re-entry circuit can be formed that results in tachycardia. Carotid sinus massage or intravenous adenosine will often terminate an episode of tachycardia. Prophylactic drug therapy is indicated for symptomatic patients. Agents such as flecainide, disopyramide or amiodarone are used in an attempt to slow the conduction rate and prolong the refractory period of the bypass tract. Digoxin and verapamil are contraindicated as they increase conduction in the bypass tract. Atrial fibrillation is poorly tolerated and is a life-threatening arrhythmia in such patients: the bypass tract lacks the rate-limiting properties of the normal AV node and ventricular fibrillation ensues. This is treated as a medical emergency with DC cardioversion.

A 25-year-old man was found by his family at home having suffered a cardiac arrest. He was previously well, apart from well controlled Type 1 diabetes controlled with a basal bolus insulin regime. His family followed the ambulance and ask if they can be in the resuscitation room. After 20 mins of repeated resuscitation cycles he has remained in asystole. Blood gases; pH

7.01

PO2

8.4 kPa

PCO2

3.9 kPa

Bicarb

10 mmol/l

Which person is the most appropriate person to make the decision to discontinue resuscitation? A&E consultant On call medical consultant Parents of the patient Patient’s fianceé Resuscitation team leader

www.apnastudent.blogspot.com A 25-year-old man was found by his family at home having suffered a cardiac arrest. He was previously well, apart from

of 179

6/30/2011 4:56 PM

well controlled Type 1 diabetes controlled with a basal bolus insulin regime. His family followed the ambulance and ask if they can be in the resuscitation room. After 20 mins of repeated resuscitation cycles he has remained in asystole. Blood gases; pH

7.01

PO2

8.4 kPa

PCO2

3.9 kPa

Bicarb

10 mmol/l

Which person is the most appropriate person to make the decision to discontinue resuscitation? A&E consultant On call medical consultant Parents of the patient

Your answer

Patient’s fianceé Resuscitation team leader

Correct answer

Clear, appropriate communication is a key component of resuscitation. Whilst it may of course be appropriate to allow the presence of relatives within the resuscitation room, they do not have the authority to continue or discontinue resuscitation; of course relatives do however need to be informed of progress. Whilst the experience of the A&E or on-call medical consultants may be useful in gaining advice, the resuscitation team leader is usually a senior anaesthetist or physician in their own right, and qualified to decide on discontinuing resuscitation, if there is any doubt they can consider discussing with the consultant on call.

A 70-year-old lady with a history of asthma presents with shortness of breath for some days. She is also treated with ramipril 10mg daily. On examination her blood pressure is 135/85 mmHg, pulse is 100/min (atrial fibrillation). She is not in cardiac failure. Examination of the respiratory system reveals wheeze consistent with asthma. Results; Hb

13.2 g/dl

WCC

9 6.1 x10 / l

PLT

9 240 x10 / l

Na+

138 mmol/l

+

4.7 mmol/l

K

Creatinine

125 µmol/l

CXR

Cardiomegaly consistent with longstanding hypertensive heart disease

Which of the following is the most appropriate treatment for her atrial fibrillation? Diltiazem Digoxin Amiodarone Atenolol Dysopyramide A 70-year-old lady with a history of asthma presents with shortness of breath for some days. She is also treated with ramipril 10mg daily. On examination her blood pressure is 135/85 mmHg, pulse is 100/min (atrial fibrillation). She is not in cardiac failure. Examination of the respiratory system reveals wheeze consistent with asthma. Results; Hb

13.2 g/dl

www.apnastudent.blogspot.com 9

WCC

6.1 x10 /l

PLT

240 x10 /l

Na+

138 mmol/l

+

4.7 mmol/l

K

9

Creatinine

125 µmol/l

CXR

Cardiomegaly consistent with longstanding hypertensive heart disease

Which of the following is the most appropriate treatment for her atrial fibrillation? Diltiazem Digoxin

Your answer

Amiodarone Atenolol Dysopyramide

This lady has been in atrial fibrillation for an unknown period of time and her pressing need at this point is rate control. Options for rate control include diltiazem, atenolol and digoxin. Atenolol may lead to worsening of her asthma and should not be used. Diltiazem is negatively inotropic and may precipitate cardiac failure (presence of cardiomegaly on CXR); as such digoxin is an effective option for rate control here.

A 71-year-old lady with a history of one previous myocardial infarction presents to the Emergency department. She has sudden onset shortness of breath and palpitations which happened after her dinner a couple of hours earlier. A previous ECG from clinic a month earlier shows sinus rhythm. Medication includes ramipril 10mg daily, amlodipine 10mg daily and aspirin 75mg. On examination her blood pressure is 100/60 mmHg, pulse is 140/min irregular and she has evidence of LVF. Bloods

Hb

14.0 g/dl

WCC

9 6.7 x10 / l

PLT

9 190 x10 / l

Na+

140 mmol/l

+

5.0 mmol/l

K

Creatinine

130 µmol/l

ECG

Fast atrial fibrillation, lateral ST depression

Which of the following is the most appropriate medication to control her AF? Digoxin Amiodarone Flecainide Sotalol Verapamil

A 71-year-old lady with a history of one previous myocardial infarction presents to the Emergency department. She has sudden onset shortness of breath and palpitations which happened after her dinner a couple of hours earlier. A previous ECG from clinic a month earlier shows sinus rhythm. Medication includes ramipril 10mg daily, amlodipine 10mg daily and aspirin 75mg. On examination her blood pressure is 100/60 mmHg, pulse is 140/min irregular and she has evidence of LVF. Bloods

Hb

of 179

14.0 g/dl

6/30/2011 4:56 PM

9

WCC

6.7 x10 /l

PLT

190 x10 /l

Na+

140 mmol/l

+

5.0 mmol/l

K

9

Creatinine

130 µmol/l

ECG

Fast atrial fibrillation, lateral ST depression

Which of the following is the most appropriate medication to control her AF? Digoxin Amiodarone Flecainide

Correct answer

Your answer

Sotalol Verapamil

Flecainide, whilst effective at cardioverting atrial fibrillation is contra-indicated in patients with a history of ischaemic heart disease since the CAST post infarct trial demonstrated increased mortality in patients treated with flecainide. Sotalol and verapamil are negatively inotropic and likely to worsen LVF. Digoxin would be effective at slowing the ventricular rate, but given that she was in sinus rhythm 1 month earlier, it would be a reasonable objective to attain sinus rhythm again. As such IV loading with amiodarone would appear to be the most appropriate option in this patient.

A 78-year-old lady is admitted from home by ambulance. She was found lying on the floor by her home help after suffering o

a fall. She has a history of hypertension managed with ramipril 10mg PO daily. On examination her temperature is 30.0 C, her BP is 100/50 mmHg, with a pulse of 52/min. She has a fractured left neck of femur. Bloods; Hb

14.5 g/dl

WCC

4.5 x10 /l

PLT

192 x10 9 /l

Na+

143 mmol/l

+

5.3 mmol/l

K

9

Creatinine

195 µmol/l

Which of the following ECG features is most characteristic of moderate to severe hypothermia? Long QT interval Short PR interval 2nd degree heart block Complete heart block J waves A 78-year-old lady is admitted from home by ambulance. She was found lying on the floor by her home help after suffering o

a fall. She has a history of hypertension managed with ramipril 10mg PO daily. On examination her temperature is 30.0 C, her BP is 100/50 mmHg, with a pulse of 52/min. She has a fractured left neck of femur. Bloods; Hb

14.5 g/dl

WCC

9 4.5 x10 / l

www.apnastudent.blogspot.com PLT

of 179

9

192 x10 / l

6/30/2011 4:56 PM

Na K

+

143 mmol/l

+

5.3 mmol/l

Creatinine

195 µmol/l

Which of the following ECG features is most characteristic of moderate to severe hypothermia? Long QT interval

Your answer

Short PR interval 2nd degree heart block Complete heart block J waves

Correct answer

Whilst varying degrees of heart block may be seen in association with hypothermia, J waves are said to be most characteristic of moderate to severe hypothermia. J waves are best seen in the left chest leads and are described as a dome or hump in the terminal portion of the QRS complex. The size of the J wave is correlated with the degree of hypothermia. Slow atrial fibrillation may also be seen as core temperature falls. Death from ventricular arrhythmias is common in moderate to severe hypothermia.

A 70-year-old woman presents to the pre-operative orthopaedic clinic prior to hip replacement. She has suffered a myocardial infarction 4 years earlier and is managed with aspirin 75mg daily, ramipril 10mg daily and atorvastatin 40mg daily. There is no history of angina but she is only able to walk around 50 yards. On examination she looks well, her BP is 145/80 mmHg with a pulse of 75/minute. Bloods; Hb

14.0 g/dl

WCC

9 5.9 x10 / l

PLT

9 180 x10 / l

Na+

140 mmol/l

+

5.0 mmol/l

K

Creatinine

130 µmol/l

Which of the following is the most appropriate investigation to assess her suitability for surgery from the point of view of her cardiovascular status? 12-lead ECG Treadmill stress test Echocardiogram Dobutamine stress echo Cardiac angiography A 70-year-old woman presents to the pre-operative orthopaedic clinic prior to hip replacement. She has suffered a myocardial infarction 4 years earlier and is managed with aspirin 75mg daily, ramipril 10mg daily and atorvastatin 40mg daily. There is no history of angina but she is only able to walk around 50 yards. On examination she looks well, her BP is 145/80 mmHg with a pulse of 75/minute. Bloods; Hb

14.0 g/dl

WCC

9 5.9 x10 / l

PLT

9 180 x10 / l

Na+

140 mmol/l

www.apnastudent.blogspot.com K

of 179

+

5.0 mmol/l

6/30/2011 4:56 PM

Creatinine

130 µmol/l

Which of the following is the most appropriate investigation to assess her suitability for surgery from the point of view of her cardiovascular status? 12-lead ECG Treadmill stress test Echocardiogram

Your answer

Dobutamine stress echo

Correct answer

Cardiac angiography

Dobutamine stress echo simulates the effect of exercise on the heart in patients who are unable to undertake a stress test. Dobutamine is given via IV infusion, and ECG monitoring with Echocardiography is undertaken both at rest and at the point of maximal stimulation. Patients are recommended to discontinue beta-blockade if possible for around 3 days prior to the procedure, as the negatively inotropic and chronotropic effects of beta blockade can blunt the effects of the dobutamine infusion.

A 45-year-old man was diagnosed with new onset AF after visiting his GP complaining of palpitations. An ECG confirmed atrial fibrillation with a ventricular rate of 85/minute, and an ECHO did not reveal any significant structural heart disease. On advice of the hospital he was given low molecular weight heparin and stabilised on warfarin, with an INR of 2.5. You arrange for him to be cardioverted a few weeks later and the procedure is successful. For how long is it recommended to continue his warfarin therapy according to current guidelines? For life For 1 week For 72 hours For four weeks For 6 months A 45-year-old man was diagnosed with new onset AF after visiting his GP complaining of palpitations. An ECG confirmed atrial fibrillation with a ventricular rate of 85/minute, and an ECHO did not reveal any significant structural heart disease. On advice of the hospital he was given low molecular weight heparin and stabilised on warfarin, with an INR of 2.5. You arrange for him to be cardioverted a few weeks later and the procedure is successful. For how long is it recommended to continue his warfarin therapy according to current guidelines? For life For 1 week For 72 hours For four weeks

Your answer

For 6 months

Guidelines published in 2006 recommend warfarinisation for at least 3 weeks pre and for 4 weeks post cardioversion, aiming for a target INR of 2.5, although this can be allowed to drift up to an INR of 3 a few days prior to the procedure to minimise any risks of cancellation due to inadequate anti-coagulation. The period of 4 weeks post procedure is recommended because there is a high relapse rate in the first few weeks.

You review a 26-year-old woman who attends the cardiology clinic with her husband. They wish to start a family, but they have been referred by the GP as he is worried that she has a history of heart disease. Which of the following cardiovascular conditions is an absolute contra-indication to pregnancy?

www.apnastudent.blogspot.com Mitral valve prolapse Previous repaired patent ductus arteriosus Atrial septal defect Primary pulmonary hypertension Bicuspid aortic valve You review a 26-year-old woman who attends the cardiology clinic with her husband. They wish to start a family, but they have been referred by the GP as he is worried that she has a history of heart disease. Which of the following cardiovascular conditions is an absolute contra-indication to pregnancy? Mitral valve prolapse Previous repaired patent ductus arteriosus Atrial septal defect Primary pulmonary hypertension

Your answer

Bicuspid aortic valve

Whilst increased monitoring is recommended in patients with mitral valve prolapse or congenital bicuspid aortic valve, neither are absolute contraindications to pregnancy. Small atrial septal defects often remain undetected for many years, and previously repaired patent ductus arteriosis should not impair pregnancy. Primary pulmonary hypertension rapidly worsens in pregnancy however, and patients are advised not to get pregnant.

A 58-year-old man with multiple dental problems presents to the Emergency department. Apart from an abscess on his toe for which he has been receiving flucloxacillin he has been relatively well. On examination he has splinter haemorrhages and looks anaemic. You detect an aortic systolic murmur. Echocardiogram is suggestive of aortic valve endocarditis and blood cultures confirm Streptococcus viridans. In addition to IV benzylpenicillin which antibiotic would you prescribe? Ceftriaxone Gentamicin Azithromycin Vancomycin Ciprofloxacin A 58-year-old man with multiple dental problems presents to the Emergency department. Apart from an abscess on his toe for which he has been receiving flucloxacillin he has been relatively well. On examination he has splinter haemorrhages and looks anaemic. You detect an aortic systolic murmur. Echocardiogram is suggestive of aortic valve endocarditis and blood cultures confirm Streptococcus viridans. In addition to IV benzylpenicillin which antibiotic would you prescribe? Ceftriaxone Gentamicin

Your answer

Azithromycin Vancomycin Ciprofloxacin

The standard regime for suspected viridans endocarditis would be benzylpenicillin IV together with gentamycin 1mg/kg/day. Ceftriaxone is an alternative in stable patients and has the advantage of being given once per day. In

of 179

6/30/2011 4:56 PM

patients who are penicillin allergic vancomycin is a viable alternative. The most likely route for his infection is via dental infection. As long as S viridans endocarditis is appropriately managed, cure rates approach 98%.

A 23-year-old woman presents to the GP complaining of palpitations. She says these are rapid and when she gets them she

www.apnastudent.blogspot.com feels light headed and sick. They tend to come on without warning, but have occurred when she has been out dancing with friends, and after a game of squash. On examination she looks well; her BMI is 21, pulse 70/min regular, BP 122/70 mmHg. Bloods; Hb

13.1 g/dl

WCC

9 5.4 x10 / l

PLT

9 251 x10 / l

Na+

139 mmol/l

+

4.0 mmol/l

K

Creatinine

75 µmol/l

Which of the following investigations is most likely to help with the diagnosis? Tilt table test Continuous loop recorder 24hr holter monitor 3 day holter monitor 12 lead ECG A 23-year-old woman presents to the GP complaining of palpitations. She says these are rapid and when she gets them she feels light headed and sick. They tend to come on without warning, but have occurred when she has been out dancing with friends, and after a game of squash. On examination she looks well; her BMI is 21, pulse 70/min regular, BP 122/70 mmHg. Bloods; Hb

13.1 g/dl

WCC

5.4 x10 /l

PLT

251 x10 9 /l

Na K

9

+

139 mmol/l

+

4.0 mmol/l

Creatinine

75 µmol/l

Which of the following investigations is most likely to help with the diagnosis? Tilt table test

Your answer

Continuous loop recorder

Correct answer

24hr holter monitor 3 day holter monitor 12 lead ECG

Whilst a 12 lead ECG may reveal evidence of structural heart disease leading to changes in the resting ECG, or an aberrant pathway such as that found in Wolf-Parkinson-White, more often than not it will be normal. A holter monitor may also be in place during a period where no palpitations occur. In contrast, a continuous loop recorder can be activated by the patient during symptoms and therefore carries the greatest chance of recording the arrhythmia.

You review a 28-year-old woman with palpitations. On examination you suspect that there is splitting of the first heart sound. Her BP is 123/80 mmHg, P 70/min regular, and her chest is clear, there are no other cardiovascular findings. You arrange a 12 lead ECG Which part of the ECG is most closely associated with the first heart sound?

of 179

6/30/2011 4:56 PM

P Wave

www.apnastudent.blogspot.com T Wave S Wave R Wave U wave You review a 28-year-old woman with palpitations. On examination you suspect that there is splitting of the first heart sound. Her BP is 123/80 mmHg, P 70/min regular, and her chest is clear, there are no other cardiovascular findings. You arrange a 12 lead ECG Which part of the ECG sound? P Wave

is most closely associated with the first heart

Your answer

T Wave S Wave R Wave

Correct answer

U wave

Splitting of the first heart sound may occur in atrioventricular septal defects or in conditions such as Ebstein’s anomaly. The first heart sound occurs most closely in association with the R wave and in patients with AVSD there is usually left axis deviation, prominent P waves and prolongation of the P-R interval. Echocardiography and cardiac catheterisation follow to determine the extent of the defect.

A 22-year-old-woman presents to A&E with a 4-day history of chest pain. She has been unwell with an influenza-like illness for the last week. The ECG shows widespread ST elevation in the inferior, anterior and lateral leads. What ECG changes would you expect to see in the next week or two? Development of deep Q waves in all leads ST depression in inferior and lateral leads T-wave inversion in all leads Tall and peaked T waves in all leads Loss of R waves in all leads A 22-year-old-woman presents to A&E with a 4-day history of chest pain. She has been unwell with an influenza-like illness for the last week. The ECG shows widespread ST elevation in the inferior, anterior and lateral leads. What ECG changes would you expect to see in the next week or two? Development of deep Q waves in all leads ST depression in inferior and lateral leads T-wave inversion in all leads

Your answer

Tall and peaked T waves in all leads Loss of R waves in all leads

The ECG changes in acute pericarditis consist of ST elevation with concavity upward in all leads facing the epicardial surface, ie anterior, inferior and lateral. Only ‘cavity’ leads avr, V 1 and, rarely, V 2 show ST depression. This is followed by the return of ST segments to baseline and flat T waves. T waves then become inverted without the loss of R waves or development of Q waves. As the illness improves, T waves become normal but may occasionally persist in patients with chronic pericarditis. Rhythm and conduction abnormalities are not typical in pericarditis unless the myocardium is

of 179

6/30/2011 4:56 PM

involved.Causes of pericarditis: • Acute idiopathic pericarditis • Infections:

viral infections (eg coxsackievirus B) tuberculosis

www.apnastudent.blogspot.com other bacteria fungi • Inflammatory; post-MI /cardiotomy: • Others:

autoimmune rheumatic disorder neoplastic uraemia trauma aortic dissection hypothyroidism irradiation drugs, eg hydralazine

A 58-year-old man with a history of hypertension managed with ramipril 10mg daily and 40 pack years of cigarette smoking presents to the Emergency department after a collapse at work. Neurological examination reveals a left sided hemiplegia. Investigations; Hb

13.8 g/dl

WCC

9 5.4 x10 / l

PLT

9 192 x10 / l

Na+

139 mmol/l

+

4.9 mmol/l

K

Creatinine

149 µmo l/l

CT head

No intra-cerebral haemorrhage identified

What is the time limit after presentation up to which thrombolysis should be administered? 1hr 3hrs 4 1/2 hrs 6hrs 12hrs A 58-year-old man with a history of hypertension managed with ramipril 10mg daily and 40 pack years of cigarette smoking presents to the Emergency department after a collapse at work. Neurological examination reveals a left sided hemiplegia. Investigations; Hb

13.8 g/dl

WCC

5.4 x10 /l

PLT

192 x10 9/l

Na+

139 mmol/l

+

4.9 mmol/l

K

9

Creatinine

149 µmo l/l

CT head

No intra-cerebral haemorrhage identified

What is the time limit after presentation up to which thrombolysis should be administered? 1hr 3hrs 4 1/2 hrs

Correct answer

6hrs

of 179

6/30/2011 4:56 PM

12hrs

Your answer

www.apnastudent.blogspot.com

Although 3 hours was the initial time limit recommended in 2004 guidelines from the RCP, the newer SIGN guidelines (Dec 2008) have superceded these with the proposed limit of 4 and a half hour. This takes into account all currently available data on the risk-benefit ratio of treatment within this timeframe. Despite an increase in haemorrhagic stroke, the group receiving thrombolysis in the NINDs study had a lower rate of death or severe disability. Both the SI GN and RCP guidelines recommend that thrombolysis for stroke should only be undertaken in a specialist stroke thrombolysis unit. http://www.sign.ac.uk/pdf/sign108.pdf

A 62-year-old man with two previous myocardial infarctions and a history of LVF controlled with ramipril and furosemide presents to his GP with palpitations. On examination his BP is 100/72 mmHg, pulse 85/min AF, with bibasal crackles consistent with heart failure. Investigations; Hb

12.1 g/dl

WCC

5.4 x10 /l

PLT

234 x10 /l

Na+

140 mmol/l

+

5.0 mmol/l

K

9

9

Creatinine

130 µmol/l

ECHO - Dilated left atrium and left ventricle Which of the following would be the most appropriate agent to control his AF? Diltiazem Sotalol Amiodarone Digoxin Verapamil A 62-year-old man with two previous myocardial infarctions and a history of LVF controlled with ramipril and furosemide presents to his GP with palpitations. On examination his BP is 100/72 mmHg, pulse 85/min AF, with bibasal crackles consistent with heart failure. Investigations; 12.1 g/dl

Hb WCC

5.4 x10 9/l

PLT

234 x10 /l

Na K

9

+

140 mmol/l

+

5.0 mmol/l

Creatinine

130 µmol/l

ECHO - Dilated left atrium and left ventricle Which of the following would be the most appropriate agent to control his AF? Diltiazem Sotalol Amiodarone

Your answer

Digoxin

Correct answer

Verapamil

of 179

6/30/2011 4:56 PM

Verapamil, diltiazem and sotalol are all to a greater or lesser extent negatively inotropic and may worsen cardiac failure. Given that his systolic blood pressure is only 100, any further reduction in cardiac output is likely to further worsen his BP. Amiodarone is useful for chemical cardioversion and as such is not the best choice here. Digoxin is less useful for rate control in AF than calcium antagonists or beta-blockers, but is the most appropriate choice here as it does improve symptoms in patients with cardiac failure, and given the enlarged left atrium, successful cardioversion is unlikely.

A 54-year-old woman with a history of atrial fibrillation presents with left face and arm weakness consistent with a stroke. On examination her BP is 162/82 mmHg, with a pulse of 85/min, irregular. Investigations; Hb

12.1 g/dl

WCC

5.4 x10 9/l

PLT

9 175 x10 / l

Na+

140 mmol/l

+

5.0 mmol/l

K

Creatinine

105 µmol/l

CT head – No evidence of intracerebral haemorrhage 6hrs post stroke you are considering anti-coagulation or anti-platelet therapy, which of the following would be most appropriate? Full IV heparinisation LMW heparin and commence warfarin treatment Alteplase Aspirin Streptokinase

A 54-year-old woman with a history of atrial fibrillation presents with left face and arm weakness consistent with a stroke. On examination her BP is 162/82 mmHg, with a pulse of 85/min, irregular. Investigations; Hb

12.1 g/dl

WCC

5.4 x10 /l

PLT

9 175 x10 / l

Na+

140 mmol/l

+

5.0 mmol/l

K

Creatinine

9

105 µmol/l

CT head – No evidence of intracerebral haemorrhage 6hrs post stroke you are considering anti-coagulation or anti-platelet therapy, which of the following would be most appropriate? Full IV heparinisation LMW heparin and commence warfarin treatment Alteplase Aspirin Streptokinase

Your answer

www.apnastudent.blogspot.com

This patient is outside the window of the NI NDS study within which thrombolysis is recommended. Meta-analysis suggested strong evidence of benefit up to 3hrs, and guidelines now support use of thrombolysis up to 4hrs after the onset of symptoms. Additionally, early anti-coagulation with heparin has been shown to increase the risk of intracerebral haemorrhage, without having a significant impact on the risk of long-term disability or death. As such, commencement of aspirin is the most appropriate option, with anti-coagulation at a later stage. A (somewhat arbitrary) delay of 2 weeks after acute stroke is recommended before starting warfarin for AF, to minimise the risk of haemorrhagic complications. See also SIGN guidelines for further details; http://www.sign.ac.uk/pdf/sign108.pdf

A 30-year-old man is being investigated for hypertension. A combination of BPs estimated by colour flow Doppler and measured values are listed below. Observed BPs LV Ascending aorta

200/10 mmHg 200/70 mmHg

Right arm

190/70 mmHg

Right femoral artery

110/70 mmHg

Which of the following is the most likely diagnosis? Coarctation of aorta Left subclavian artery stenosis Aortic regurgitation Aortic stenosis HOCM

A 30-year-old man is being investigated for hypertension. A combination of BPs estimated by colour flow Doppler and measured values are listed below. Observed BPs LV Ascending aorta

200/10 mmHg 200/70 mmHg

Right arm

190/70 mmHg

Right femoral artery 110/70 mmHg Which of the following is the most likely diagnosis? Coarctation of aorta

Your answer

Left subclavian artery stenosis Aortic regurgitation Aortic stenosis HOCM

This picture would be typical of coarctation distal to the origin of the right subclavian artery. Cardiac catheterisation is the next most appropriate step in further characterising the pressure gradient across the coarctation, +/- MRI scanning. Prognosis after coarctation is dependent on speed of diagnosis. In those diagnosed after the age of 35 years, survival to age 50 is only 20%, whereas diagnosis and treatment as a child is associated with a survival above 90%.

of 179

6/30/2011 4:56 PM

A 53-year-old patient who has had chemotherapy for metastatic breast cancer 6 months earlier comes to the clinic complaining of shortness of breath on exertion. Her BP is 125/78 mmHg, her pulse is 94/min and her apex beat is displaced

www.apnastudent.blogspot.com to the anterior axillary line. Investigations Hb

11.9 g/dl

WCC

9 5.0 x10 / l

PLT

9 190 x10 / l

Na+

140 mmol/l

K+

4.5 mmol/l

Creatinine

160 µmol/l

CXR – Cardiomegaly, increased shadowing consistent with mild pulmonary oedema at both bases Which chemotherapeutic agent is most likely to be responsible for this patient's symptoms? Doxorubicin Docetaxel Cisplatin Bleomycin Carbiplatin

A 53-year-old patient who has had chemotherapy for metastatic breast cancer 6 months earlier comes to the clinic complaining of shortness of breath on exertion. Her BP is 125/78 mmHg, her pulse is 94/min and her apex beat is displaced to the anterior axillary line. Investigations Hb

11.9 g/dl

WCC

9 5.0 x10 / l

PLT

9 190 x10 / l

Na+

140 mmol/l

+

4.5 mmol/l

K

Creatinine

160 µmol/l

CXR – Cardiomegaly, increased shadowing consistent with mild pulmonary oedema at both bases Which chemotherapeutic agent is most likely to be responsible for this patient's symptoms? Doxorubicin

Your answer

Docetaxel Cisplatin Bleomycin Carbiplatin

Doxorubicin can be associated with cardiac failure, reduced left ventricular ejection fraction and tachyarrhythmias. Docetaxel may also be associated with arrhythmias and cardiac failure, although the likelihood is less than with doxorubicin. Platinum based chemotherapies are associated with nerve damage, and bleomycin with interstitial pneumonitis.

A 24-year-old man from a travelling family who has shunned regular medical follow up comes to the clinic complaining of shortness of breath and chest pain. You review his catheterisation results.

of 179

6/30/2011 4:56 PM

Pressure RV

110/0 mmHg

www.apnastudent.blogspot.com Pressure LV

90/0 mmHg

LV oxygen saturation

88%

Given the likely clinical diagnosis, which of the following is the most likely finding on clinical examination? A diastolic murmur Persistent hypoxia despite maximal oxygen therapy Tapping apex beat Broad P waves on ECG Decreased pulmonary vasculature on CXR

A 24-year-old man from a travelling family who has shunned regular medical follow up comes to the clinic complaining of shortness of breath and chest pain. You review his catheterisation results. Pressure RV

110/0 mmHg

Pressure LV

90/0 mmHg

LV oxygen saturation

88%

Given the likely clinical diagnosis, which of the following is the most likely finding on clinical examination? A diastolic murmur Persistent hypoxia despite maximal oxygen therapy

Correct answer

Tapping apex beat Broad P waves on ECG

Your answer

Decreased pulmonary vasculature on CXR

This man has a right to left shunt, with right ventricular pressure greater than left. There is clear mixing of deoxygenated and oxygenated blood within the left ventricle as evidenced by the LV oxygen saturation of 88%. The most likely diagnosis is a long-standing VSD. The typical murmur seen is a holosystolic murmur, and the apex beat is usually displaced. P waves are tall on the ECG, a sign typical of that seen with right atrial overload due to pulmonary hypertension. Of course, because of pulmonary hypertension, increased pulmonary vasculature markings are normally seen on the CXR.

A 62-year-old woman is admitted having collapsed at her local supermarket complaining of palpitations. On examination she is very unwell with a BP of 90/50 mmHg and very rapid palpitations. Investigations; ECG – Ventricular tachycardia with moving axis – torsade de pointes Which of the following drugs is not associated with this arrhythmia? Sotalol Verapamil Flecainide Digoxin Risperidone A 62-year-old woman is admitted having collapsed at her local supermarket complaining of palpitations. On examination she is very unwell with a BP of 90/50 mmHg and very rapid palpitations. Investigations; ECG – Ventricular tachycardia with moving axis – torsade de pointes Which of the following drugs is not associated with this arrhythmia? Sotalol

of 179

Your answer

6/30/2011 4:56 PM

Verapamil

Correct answer

www.apnastudent.blogspot.com Flecainide Digoxin Risperidone

Verapamil leads to a reduction in the risk of torsade de pointes (also called torsades de pointes) by reducing left ventricular transmural dispersion of repolarisation, and suppresses after depolarisations. Risperidone, sotalol and flecainide may all lead to QT prolongation and precipitate development of torsade de pointes. Digoxin in toxic levels may also be associated with ventricular arrhythmias. Acute treatment of torsade involves correction of electrolyte abnormalities such as hypokalaemia and hypomagnesemia, and removal of QT prolonging agents. Short acting beta-blockade is the mainstay of pharmacological intervention.

A 60-year-old woman with a long history of manic depressive psychosis managed with lithium therapy is sent to see you for review. She has a BP of 152/93 mmHg, and the GP is keen to commence anti-hypertensive therapy. Investigations Hb

12.3 g/dl

WCC

5.4 x10 /l

PLT

195 x10 9 /l

Na+

143 mmol/l

+

4.0 mmol/l

K

9

µmol/l

Creatinine

145

Total cholesterol

5.9 mmol/l

HDL

0.8 mmol/l

Which antihypertensive would be most appropriate for her to start without risk of significant lithium toxicity? Ramipril Valsartan Indapamide Amlodipine Atenolol A 60-year-old woman with a long history of manic depressive psychosis managed with lithium therapy is sent to see you for review. She has a BP of 152/93 mmHg, and the GP is keen to commence anti-hypertensive therapy. Investigations Hb

12.3 g/dl

WCC

5.4 x10 9/l

PLT

195 x10 9 /l

Na K

+

143 mmol/l

+

4.0 mmol/l

µmol/l

Creatinine

145

Total cholesterol

5.9 mmol/l

HDL

0.8 mmol/l

Which antihypertensive would be most appropriate for her to start without risk of significant lithium toxicity? Ramipril

of 179

6/30/2011 4:56 PM

Valsartan

www.apnastudent.blogspot.com Indapamide Amlodipine Atenolol

Your answer

ACE inhibitors, angiotensin II receptor blockers both lead to increases in lithium concentration and can lead to lithium toxicity. A paradoxical anti-diuretic effect exists when thiazides such as indapamide are combined with lithium, leading to salt and water retention and increased lithium concentration. Calcium antagonists may lead to elevated levels of lithium and hence toxicity. This leaves atenolol as the available option which we are left with as, whilst it is not first choice in hypertension guidelines, it represents the safest option here.

A 42-year-old patient who has a history of paroxysmal AF has been treated with warfarin. The AF has now resolved after successful DC cardioversion. Investigations; Hb

13.1 g/dl

WCC

9 4.9 x10 / l

PLT

9 294 x10 / l

Na+

139 mmol/l

+

4.8 mmol/l

K

Creatinine

100 µmol/l

TSH

2.1 U/l

ECHO – Normal sized left atrium, no significant valvular disease For how long should the warfarin be continued? 4 weeks 6 months 1 year 3 years Stop with immediate effect A 42-year-old patient who has a history of paroxysmal AF has been treated with warfarin. The AF has now resolved after successful DC cardioversion. Investigations; Hb

13.1 g/dl

WCC

9 4.9 x10 / l

PLT

9 294 x10 / l

Na+

139 mmol/l

+

4.8 mmol/l

K

Creatinine

100 µmol/l

TSH

2.1 U/l

ECHO – Normal sized left atrium, no significant valvular disease For how long should the warfarin be continued? 4 weeks

of 179

Your answer

6/30/2011 4:56 PM

6 months 1 year

www.apnastudent.blogspot.com 3 years Stop with immediate effect

This patient has benign findings on echocardiography, with normal left atrial size; it is therefore likely that he has a reasonable chance of remaining in sinus rhythm. NICE guidelines on the management of atrial fibrillation, most recently re-issued in 2006, recommend continuing warfarin therapy for a minimum of 4 weeks. Where the risk of recurrence is high, or there are multiple failed cardioversions, then long-term warfarin therapy is advised. http://www.nice.org.uk/Guidance/CG36/QuickRefGuide/pdf/English

A 32-year-old man presents to the clinic with shortness of breath, which is particularly bad when he goes jogging. He has recently increased his exercise to try and reduce his weight. On a couple of occasions he has also noticed some chest discomfort which has caused him to stop exercising. On examination his BP is 150/88 mmHg, and he has a double apical impulse. On auscultation there is a harsh mid systolic murmur which is loudest between the apex and the left sternal border. Investigations; Hb

13.0 g/dl

WCC

4.8 x10 /l

PLT

199 x10 9 /l

Na+

140 mmol/l

+

5.0 mmol/l

K

Creatinine

9

100 µmol/l

ECG LVH and widespread Q waves Which of the following is most directly correlated with increased risk of sudden death? Increased left ventricular outflow tract gradient Presence of mitral regurgitation Degree of left ventricular hypertrophy Asymmetrical septal hypertrophy Systolic anterior motion A 32-year-old man presents to the clinic with shortness of breath, which is particularly bad when he goes jogging. He has recently increased his exercise to try and reduce his weight. On a couple of occasions he has also noticed some chest discomfort which has caused him to stop exercising. On examination his BP is 150/88 mmHg, and he has a double apical impulse. On auscultation there is a harsh mid systolic murmur which is loudest between the apex and the left sternal border. Investigations; Hb

13.0 g/dl

WCC

4.8 x10 9/l

PLT

9 199 x10 / l

Na+

140 mmol/l

+

5.0 mmol/l

K

Creatinine

100 µmol/l

ECG LVH and widespread Q waves Which of the following is most directly correlated with increased risk of sudden death? Increased left ventricular outflow tract gradient

of 179

6/30/2011 4:56 PM

Presence of mitral regurgitation Degree of left ventricular hypertrophy

Your answer

Asymmetrical septal hypertrophy Systolic anterior motion

A number of studies have attempted to examine potential correlations between clinical features of HOCM and increased risk of sudden cardiac death. Of the options given, degree of left ventricular hypertrophy appears to be most strongly linked to increased risk of sudden death. I ncreased outflow tract gradient appears to be related to symptoms like SOB and angina, and myomectomy taking tissue from the interventricular septum appears to improve these symptoms. Patients with HOCM usually die from arrhythmias, and previous VT is thus strongly predictive of the risk of sudden death. J Am Coll Cardiol, 2003; 41:994-996, doi:10.1016/S0735-1097(02)03003-6 is a good review of the relevant data.

Which of the following statements is true of raised cardiac troponin levels in the blood? Are commonly seen after DC cardioversion Remain elevated for up to two days after myocardial damage Are seen in patients with NSTEMI Can be used to distinguish Non-Q from Q MI Can be found in patients with hypertrophic cardiomyopathy

Which of the following statements is true of raised cardiac troponin levels in the blood? Are commonly seen after DC cardioversion Remain elevated for up to two days after myocardial damage Are seen in patients with NSTEMI

Your answer

Can be used to distinguish Non-Q from Q MI Can be found in patients with hypertrophic cardiomyopathy

The most sensitive markers of myocardial cell damage are the cardiac troponins T and I. Troponins are regulatory elements of the contractile apparatus in muscle. They are released within 4–6 hours and remain elevated for up to two weeks. Unstable angina and ST segment myocardial infarction are different ends of the spectrum of myocardial damage, with greater rises in troponin seen towards the more severe end of the spectrum. Angina associated with a troponin rise is essentially an NSTEMI. Cardioversion can give rise to raised creatine kinase (CK) levels due to skeletal muscle damage. Non-Q and Q MI are electrocardiographic diagnoses. Raised troponin levels indicate heart muscle damage: the commonest cause being ischaemic damage. Myocarditis or myocardial contusion can also cause raised troponin levels.

A 42-year-old man with the features of congenital myotonic dystrophy comes to see you for review. He has suffered from mild intellectual impairment, frontal balding typical of the disease and increasing muscle weakness with increased muscle tone over the past few years. Most recently he has suffered from a number of episodes of syncope. On examination his BP is 129/70 mmHg, his pulse 55 BPM, there are no other significant findings on cardiovascular examination. What ECG findings might you most commonly expect to see in this case? Short PR interval PR prolongation

www.apnastudent.blogspot.com Long QT syndrome Bifasicular block Left bundle branch block A 42-year-old man with the features of congenital myotonic dystrophy comes to see you for review. He has suffered from

of 179

6/30/2011 4:56 PM

www.apnastudent.blogspot.com mild intellectual impairment, frontal balding typical of the disease and increasing muscle weakness with increased muscle tone over the past few years. Most recently he has suffered from a number of episodes of syncope. On examination his BP is 129/70 mmHg, his pulse 55 BPM, there are no other significant findings on cardiovascular examination. What ECG findings might you most commonly expect to see in this case? Short PR interval PR prolongation

Your answer

Long QT syndrome Bifasicular block Left bundle branch block

PR prolongation is the commonest feature seen in association with congenital myotonic dystrophy. Varying conduction defects including right bundle branch block, left bundle branch block and bifasicular block may be seen. The syncopes seen here may be related to periods of complete heart block, and a 72hr holter monitor would be the next most logical investigation in an attempt to capture these. The other possibility would be short runs of VT, as myotonic dystrophy also increases the risk of paroxysms of VT, again, it would be hoped that these would be captured on a holter monitor.

A 50-year-old man with no previous cardiovascular history comes to the Emergency room after referral from his GP. He attended the GP surgery with palpitations, which were extremely rapid and irregular. I t is now 9am and he tells you the palpitations began the previous morning after a heavy drinking session with a friend from work. There is no history of smoking, cardiovascular disease or previous myocardial infarction, he plays squash twice per week and cycles to work. On examination his BP is 125/77 mmHg, his pulse is 140/min, irregular. He is not in cardiac failure. Investigations; Hb

13.1 g/dl

WCC

9 4.9 x10 / l

PLT

9 210 x10 / l

Na+

139 mmol/l

+

4.7 mmol/l

K

µmol/l

Creatinine

120

CXR

No cardiomegaly, no LVF

ECG

Fast atrial fibrillation, no Q waves

Which of the following is the most appropriate therapy to chemically cardiovert him? Adenosine Bisoprolol Digoxin Flecainide Verapamil A 50-year-old man with no previous cardiovascular history comes to the Emergency room after referral from his GP. He attended the GP surgery with palpitations, which were extremely rapid and irregular. I t is now 9am and he tells you the palpitations began the previous morning after a heavy drinking session with a friend from work. There is no history of smoking, cardiovascular disease or previous myocardial infarction, he plays squash twice per week and cycles to work. On examination his BP is 125/77 mmHg, his pulse is 140/min, irregular. He is not in cardiac failure. Investigations; Hb

13.1 g/dl

WCC

9 4.9 x10 / l

PLT

9 210 x10 / l

www.apnastudent.blogspot.com Na K

+

139 mmol/l

+

4.7 mmol/l

µmol/l

Creatinine

120

CXR

No cardiomegaly, no LVF

ECG

Fast atrial fibrillation, no Q waves

Which of the following is the most appropriate therapy to chemically cardiovert him? Adenosine Bisoprolol Digoxin Flecainide

Your answer

Verapamil

In the UK the commonest drugs used for cardioversion of atrial fibrillation are flecainide and amiodarone. Successful cardioversion is reported in up to 90% of patients given IV flecainide. It should be avoided in patients with a previous ischaemic cardiovascular history as the CAST trial suggested that mortality was increased in patients given flecanide post myocardial infarction. Digoxin does not cardiovert, and bisoprolol and verapamil are more usually employed to maintain sinus rhythm after successful electrical cardioversion. As we have a very clear history of duration of AF and no evidence of structural heart disease, successful cardioversion is likely to be possible.

An 18-year-old man comes to the Emergency room because he has suffered a severe syncopal attack whilst playing a game of squash. His opponent tells you that he collapsed and took a few minutes to recover. Apparently this was the second episode, the first having occurred after a strenuous period of exercise at the swimming pool. Of note is the fact that his father died of a cardiac arrest at the age of 32. On examination he looks fit, his BP is 132/78 mmHg, his pulse is 70/min, sinus rhythm. Investigations; Hb

12.8 g/dl

WCC

5.0 x10 /l

PLT

182 x10 9/l

Na+

139 mmol/l

K+

4.8 mmol/l

Creatinine

120

ECG

Sinus rhythm but QT interval 0.51s

24hr tape

paroxysmal AF on 2 occasions

9

µmol/l

Which of the following agents should be given for rhythm control in this case? Adenosine Flecainide Verapamil Amiodarone Metoprolol An 18-year-old man comes to the Emergency room because he has suffered a severe syncopal attack whilst playing a game of squash. His opponent tells you that he collapsed and took a few minutes to recover. Apparently this was the second episode, the first having occurred after a strenuous period of exercise at the swimming pool. Of note is the fact that his father died of a cardiac arrest at the age of 32. On examination he looks fit, his BP is 132/78 mmHg, his pulse is 70/min, sinus rhythm. Investigations;

of 179

(FACEBOOK) https/c 6/30/2011 4:56 PM

www.apnastudent.blogspot.com Hb

12.8 g/dl

WCC

5.0 x10 9/l

PLT

182 x10 /l

Na

9

+

139 mmol/l

K+

4.8 mmol/l

Creatinine

120

ECG

Sinus rhythm but QT interval 0.51s

24hr tape

paroxysmal AF on 2 occasions

µmol/l

Which of the following agents should be given for rhythm control in this case? Adenosine Flecainide Verapamil

Your answer

Amiodarone Metoprolol

Correct answer

Beta blocking agents are the drugs of choice for rhythm control in long QT syndrome. They decrease conduction through the AV node and have negatively inotropic and chronotropic effects. I f patients continue to have rhythm disturbances on beta blockade then cervical sympathectomy is one treatment option, or they may be referred for Implantable cardioverter defibrillator. Lifestyle changes such as avoiding competitive or particularly intensive sporting activity may also be recommended. Other agents, particularly flecainide or amiodarone lengthen the cardiac action potential and may increase the risk of torsades de pointes VT.

An 18-year-old student is admitted to the Emergency room after a collapse in a night club. He has no recollection of the incident, was assisted by his friends and had begun to regain consciousness by the time the ambulance had arrived. On direct questioning in the Emergency room he admits to 2 previous syncopal episodes. He denies elicit drug use. On examination his BP is 123/72 mmHg, his pulse is 72 regular. Investigations; Hb

13.2 g/dl

WCC

5.3 x109/l

PLT

199 x109/l

Na

142 mmol/l

K

4.6 mmol/l

Creatinine

90

ECG

Sinus rhythm, QT interval 0.52s

µmol/l

A defect in which ion channel is the most likely cause of his symptoms? Magnesium Sodium Potassium Chloride Calcium An 18-year-old student is admitted to the Emergency room after a collapse in a night club. He has no recollection of the incident, was assisted by his friends and had begun to regain consciousness by the time the ambulance had arrived. On direct questioning in the Emergency room he admits to 2 previous syncopal episodes. He denies elicit drug use. On examination his BP is 123/72 mmHg, his pulse is 72 regular.

www.apnastudent.blogspot.com Investigations;

(FACEBOOK) https/c of 179

6/30/2011 4:56 PM

www.apnastudent.blogspot.com

Hb

13.2 g/dl

WCC

5.3 x109/l

PLT

199 x109/l

Na

142 mmol/l

K

4.6 mmol/l

Creatinine

90

ECG

Sinus rhythm, QT interval 0.52s

µmol/l

A defect in which ion channel is the most likely cause of his symptoms? Magnesium Sodium Potassium

Correct answer

Chloride Calcium

Your answer

LQT1, 2 and 3 mutations account for 45%,45% and 7% of cases of long QT syndrome respectively. Both LQT1 and 2 mutations are associated with defective potassium transport, leading to a decrease in potassium outflow and more prolonged depolarisation. LQT8 is associated with defective calcium channel transport; often patients also have associated congenital heart disease and behavioural disorders. LQT3 mutation is associated with a gain of function mutation in sodium channels.

A 72-year-old man who visited his GP suffering from an infection 2 days earlier is admitted to the Emergency room after suffering a collapse at the supermarket. His wife tells you that he suffered from a myocardial infarction some 6 years ago, but has otherwise been relatively well, taking aspirin, ramipril and atorvastatin as regular medications. She doesn’t know the type of antibiotics he has been taking. On examination his BP is 120/71 mmHg and he is drowsy. His pulse is 70/min and regular. Investigations; Hb

13.1 g/dl

WCC

5.1 x10 /l

PLT

232 x10 /l

Na+

140 mmol/l

+

4.2 mmol/l

K

9

9

Creatinine

123

µmol/l

Short runs of torsades seen on the monitor Which of the following is the most likely causative antibiotic? Oxytetracycline Metformin Co-amoxyclav Cephalexin Clarithromycin A 72-year-old man who visited his GP suffering from an infection 2 days earlier is admitted to the Emergency room after suffering a collapse at the supermarket. His wife tells you that he suffered from a myocardial infarction some 6 years ago, but has otherwise been relatively well, taking aspirin, ramipril and atorvastatin as regular medications. She doesn’t know the type of antibiotics he has been taking. On examination his BP is 120/71 mmHg and he is drowsy. His pulse is 70/min and regular.

www.apnastudent.blogspot.com Investigations; Hb

13.1 g/dl

WCC

5.1 x10 /l

PLT

232 x10 /l

Na+

140 mmol/l

+

4.2 mmol/l

K

9

9

123

Creatinine

µmol/l

Short runs of torsades seen on the monitor Which of the following is the most likely causative antibiotic? Oxytetracycline

Your answer

Metformin Co-amoxyclav Cephalexin Clarithromycin

Correct answer

Both macrolides such as erythromycin, and clarithromycin, and quinolones such as ciprofloxacin and olfloxacin may lead to QT prolongation. The problem may be exacerbated by co-administration with CYP-P450 inhibitors such as ketoconazole. Initial management involves withdrawal of the potential offending agent and electrolyte assay to exclude potential exacerbating factors such as hypomagnesaemia.

A 55 year-old man with a history of mitral regurgitation and atrial fibrillation is warfarinised. His INR is therapeutic at 2.0. He needs to undergo pre-planned tooth extraction under local anaesthesia. How would you manage him prior to the procedure? Stop warfarin for 2 days Stop warfarin, start LMWH Stop warfarin, start unfractionated heparin Stop warfarin start aspirin Maintain warfarin at the therapeutic dose A 55 year-old man with a history of mitral regurgitation and atrial fibrillation is warfarinised. His INR is therapeutic at 2.0. He needs to undergo pre-planned tooth extraction under local anaesthesia. How would you manage him prior to the procedure? Stop warfarin for 2 days Stop warfarin, start LMWH Stop warfarin, start unfractionated heparin Stop warfarin start aspirin Maintain warfarin at the therapeutic dose

Your answer

For patients on short term warfarin treatment it is recommended that patients wait for treatment until after they have discontinued their period of warfarin therapy. For those on long-term warfarin therapy, British Haematological Society guidelines suggest that as long as the INR is not above 2, the procedure may take place in the standard way. UK

(FACEBOOK) https/c

of 179

6/30/2011 4:56 PM

www.apnastudent.blogspot.com Medicines Information (UKMI ) recommendations are more relaxed, suggesting that dental procedures may take place as long as the INR is less than 4.0.

www.apnastudent.blogspot.com A 67-year-old man attends the cardiology clinic. He has been suffering some angina-type chest pain on going out in the cold air and is worried that he might have coronary artery disease. There is a past medical history of smoking 20 cigarettes per day, and hypertension which is managed with ramipril 10mg daily. His GP has sent an ECG which appears to show that he is in left bundle branch block. What would you expect to hear on auscultation? Loud first heart sound, reversed splitting of the 2nd heart sound Soft first heart sound, fixed splitting of the 2nd heart sound Soft first heart sound, reversed splitting of the 2nd heart sound Soft first heart sound, normal 2nd heart sound Loud first heart sound, normal 2nd heart sound A 67-year-old man attends the cardiology clinic. He has been suffering some angina-type chest pain on going out in the cold air and is worried that he might have coronary artery disease. There is a past medical history of smoking 20 cigarettes per day, and hypertension which is managed with ramipril 10mg daily. His GP has sent an ECG which appears to show that he is in left bundle branch block. What would you expect to hear on auscultation? Loud first heart sound, reversed splitting of the 2nd heart sound

Your answer

Soft first heart sound, fixed splitting of the 2nd heart sound Soft first heart sound, reversed splitting of the 2nd heart sound

Correct answer

Soft first heart sound, normal 2nd heart sound Loud first heart sound, normal 2nd heart sound

Left bundle branch block results in the left ventricle depolarising from cell to cell conduction via the right ventricle, rather than via the normal pathway. This results in reversed splitting of the second heart sound, in other words split in expiration and single in inspiration. Additionally the first heart sound tends to be softer than usual. Left bundle branch block in this age group is most likely to be ischaemic in origin, hence his chest pain almost certainly warrants further investigation, especially given his history of smoking and hypertension.

You are looking at drawing up guidelines for appropriate use of IIb 3a inhibitors within your hospital. Looking at available evidence, what is the most appropriate indication for using these therapies? In a patient presenting with chest pain without ECG changes A patient with chest pain, a positive troponin and awaiting angiography In conjunction with thrombolysis in an MI with ST segment elevation In a thrombolysed patient suffering continuing chest pain In a patient with T wave inversion who is pain free You are looking at drawing up guidelines for appropriate use of IIb 3a inhibitors within your hospital. Looking at available evidence, what is the most appropriate indication for using these therapies? In a patient presenting with chest pain without ECG changes A patient with chest pain, a positive troponin and awaiting angiography

Correct answer

In conjunction with thrombolysis in an MI with ST segment elevation In a thrombolysed patient suffering continuing chest pain

Your answer

In a patient with T wave inversion who is pain free

This patient population has been examined in clinical studies with and without background clopidogrel therapy. Even on top of clopidogrel therapy a 35% reduction was seen in a combined endpoint of death, target vessel revascularisation

(FACEBOOK) https/c

of 179

6/30/2011 4:56 PM

www.apnastudent.blogspot.com and MI at 30 days. Other studies involving patients not undergoing PCI were not positive, and in ST segment MI alternative anti-thrombotic regimens, for instance using fondaparinux may be more appropriate.

www.apnastudent.blogspot.com You are working in the chemical pathology lab and receive a sample request for analysis of B-type natriuretic peptide (BNP). You don’t have any clinical details on the form apart from “chest pain”. You plan to ring the SHO who requested the test for further details. In which of the following situations is BNP most likely to be normal? Unstable angina Pericarditis Pulmonary embolus Acute myocardial infarction Acute mitral valve rupture You are working in the chemical pathology lab and receive a sample request for analysis of B-type natriuretic peptide (BNP). You don’t have any clinical details on the form apart from “chest pain”. You plan to ring the SHO who requested the test for further details. In which of the following situations is BNP most likely to be normal? Unstable angina

Correct answer

Pericarditis

Your answer

Pulmonary embolus Acute myocardial infarction Acute mitral valve rupture

BNP is secreted in response to raised intra-cardiac pressures, primarily due to volume distension, and leads to increased sodium excretion and decreased systemic vascular resistance. Both acute myocardial infarction and acute mitral valve rupture may result in volume distension, leading to elevated levels of BNP. Constrictive pericarditis may also lead to raised intracavity pressures and hence elevated BNP. Large pulmonary embolus produces raised right sided cardiac pressures and thus again may lead to elevated BNP. Thus the only other remaining option is unstable angina.

A 62-year-old woman suddenly deteriorates 2 days after receiving TPA for an acute myocardial infarction. She complained of severe shortness of breath during the course of the afternoon and when the nurses examined her, her O2 saturation was only 91% on oxygen delivered via a non-re-breather. On examination her BP is 105/70 mmHg, with a pulse of 105/min regular. She has an apical systolic murmur and marked left ventricular failure. Which of the following is the most likely cause? Acute VSD Acute ASD Pericardial tamponade LV wall rupture Papillary muscle rupture A 62-year-old woman suddenly deteriorates 2 days after receiving TPA for an acute myocardial infarction. She complained of severe shortness of breath during the course of the afternoon and when the nurses examined her, her O2 saturation was only 91% on oxygen delivered via a non-re-breather. On examination her BP is 105/70 mmHg, with a pulse of 105/min regular. She has an apical systolic murmur and marked left ventricular failure. Which of the following is the most likely cause? Acute VSD

Your answer

Acute ASD Pericardial tamponade LV wall rupture

(FACEBOOK) https/c of 179

6/30/2011 4:56 PM

www.apnastudent.blogspot.com Papillary muscle rupture

Correct answer

www.apnastudent.blogspot.com

The posteromedial papillary muscle is twice as likely to rupture as the anterolateral one because the posteromedial papillary muscle is supplied by the right coronary artery only, whereas the anterolateral papillary muscle receives supply from both the left anterior descending and left circumflex arteries. The murmur is consistent with mitral regurgitation which in turn leads to acute left ventricular failure. Management centres on decreasing afterload in an attempt to stabilise patients before undergoing valvular surgery; sodium nitroprusside is the usual therapy of choice where blood pressure allows.

A 75-year-old-man presents to A&E with a history of sudden collapse. This occurred unexpectedly while he was walking his dog. There have been no similar episodes in the past. On examination there were no positive findings. An ECG performed with carotid sinus massage revealed a 5-second pause. Which of the following statements is true? Carotid sinus hypersensitivity is due to atherosclerosis Carotid sinus massage is contraindicated in patients with carotid vascular disease A permanent pacemaker has no role in the management of these patients Carotid sinus hypersensitivity is related to vertebrobasilar ischaemia Carotid sinus massage is contraindicated in patients taking

β-blockers

A 75-year-old-man presents to A&E with a history of sudden collapse. This occurred unexpectedly while he was walking his dog. There have been no similar episodes in the past. On examination there were no positive findings. An ECG performed with carotid sinus massage revealed a 5-second pause. Which of the following statements is true? Carotid sinus hypersensitivity is due to atherosclerosis Carotid sinus massage is contraindicated in patients with carotid vascular disease

Your answer

A permanent pacemaker has no role in the management of these patients Carotid sinus hypersensitivity is related to vertebrobasilar ischaemia Carotid sinus massage is contraindicated in patients taking

β-blockers

Carotid sinus baroreceptors consist of sensory nerve endings located in the internal carotid artery just above the bifurcation of the common carotid artery. Cardioinhibitory carotid sinus hypersensitivity is defined as cardiac asystole of > 3 s. The pure vasodepressor type is defined as a systolic blood pressure drop of > 50 mmHg (in the absence of significant bradycardia). A mixed type consists of a combination of cardioinhibitory and vasodepressor responses. As AV block can occur during the periods of hypersensitive carotid reflex, some form of ventricular pacing, with or without atrial pacing, is generally required. The mechanism responsible for carotid sinus hypersensitivity is unknown, but possibilities include a high level of resting vagal tone, hyperresponsiveness to acetylcholine or an excessive release of acetylcholine.

A 70-year-old man presents with severe tearing back and chest pain which came on very suddenly. He has a past medical history of hypertension for which he takes ramipril 10mg daily, amlodipine 5mg, and he smokes 30 cigarettes per day. On examination he is in severe pain, his BP is 155/85 mmHg, he has bilateral upgoing plantars and 4/5 weakness affecting left ankle dorsiflexion. He appears to have a pericardial rub. Which of the following features is most suggestive of dissecting aortic aneurysm? The pattern of pain described Hypertension Bilateral upgoing plantars Left lower limb signs Pericardial rub

(FACEBOOK) https/c of 179

6/30/2011 4:56 PM

www.apnastudent.blogspot.com A 70-year-old man presents with severe tearing back and chest pain which came on very suddenly. He has a past medical history of hypertension for which he takes ramipril 10mg daily, amlodipine 5mg, and he smokes 30 cigarettes per day. On

www.apnastudent.blogspot.com examination he is in severe pain, his BP is 155/85 mmHg, he has bilateral upgoing plantars and 4/5 weakness affecting left ankle dorsiflexion. He appears to have a pericardial rub. Which of the following features is most suggestive of dissecting aortic aneurysm? The pattern of pain described

Your answer

Hypertension Bilateral upgoing plantars Left lower limb signs Pericardial rub

The acute onset of severe tearing back and chest pain is very typical of dissecting aortic aneurysm. It is impossible to tell whether his neurological signs seen are new or old, and a pericardial rub or hypertension are more likely to be associated with other causes of chest pain. Upper limb neurological signs are more likely to be associated with thoracic aortic dissection, lower limb signs may be commoner in anterior spinal artery dissection or thrombosis. Painless aortic dissection only occurs in around 10% of patients, and is more common in patients who have connective tissue disorders such as Marfan’s.

A 72-year-old man was admitted with an acute anterior myocardial infarction. He has chronic renal impairment, with a recent creatinine recorded at 148 µmol/l. Medication included ramipril, atorvastatin and indapamide for the treatment of hypertension. He was taken straight to the angiography suite where he received stenting of a left main stem stenosis. You are asked to see him about 30hrs after as the nurses feel he is deteriorating. On examination his BP is 149/84 mmHg, his pulse is 75/min and regular. His legs look dusky in colour, particularly his right big toe which looks blue in colour. He has splinter haemorrhages affecting toenails on both feet. There is a loud left femoral bruit. Investigations; Hb

13.2 g/dl

WCC

5.0 x10 9 /l

PLT

190 x10 9 /l

Na K

+

141 mmol/l

+

5.9 mmol/l

Creatinine

630 µmol/l

Urine

blood ++, protein +

Which of the following is the most likely diagnosis? Renal vein thrombosis Acute tubular necrosis Renal artery stenosis Cholesterol embolism Femoral artery embolism

A 72-year-old man was admitted with an acute anterior myocardial infarction. He has chronic renal impairment, with a recent creatinine recorded at 148 µmol/l. Medication included ramipril, atorvastatin and indapamide for the treatment of hypertension. He was taken straight to the angiography suite where he received stenting of a left main stem stenosis. You are asked to see him about 30hrs after as the nurses feel he is deteriorating. On examination his BP is 149/84 mmHg, his pulse is 75/min and regular. His legs look dusky in colour, particularly his right big toe which looks blue in colour. He has splinter haemorrhages affecting toenails on both feet. There is a loud left femoral bruit. Investigations; Hb

13.2 g/dl

WCC

9 5.0 x10 / l

(FACEBOOK) https/c of 179

6/30/2011 4:56 PM

www.apnastudent.blogspot.com PLT

9

190 x10 / l

www.apnastudent.blogspot.com Na K

+

141 mmol/l

+

5.9 mmol/l

Creatinine

630 µmol/l

Urine

blood ++, protein +

Which of the following is the most likely diagnosis? Renal vein thrombosis Acute tubular necrosis Renal artery stenosis Cholesterol embolism

Your answer

Femoral artery embolism

Risk factors for cholesterol embolism after coronary artery instrumentation include increased age (>60 years), hypertension, cerebral vascular disease and aorto-iliac arterial disease. Further vascular procedures, anti-coagulant and thrombolytic therapies are not of value in the management of the condition. Patients should be dialysed during the acute period as they may recover a limited amount of renal function. Unfortunately the prognosis of cholesterol embolism is very poor: where multiple organs are involved mortality may approach 90% at 3 months.

A 21-year-old woman presents to the clinic with symptoms of increased shortness of breath and decreased exercise tolerance. She used to be a keen hockey player when at school but is now virtually unable to even walk to the bus stop without becoming short of breath. On examination she looks tired and slightly short of breath at rest. Her BP is elevated at 145/92 mmHg. Investigations; Echocardiogram – increased right atrial size, elevated right arterial pressure by Doppler Cardiac catheterization; O2 O2 O2 O2 O2

saturation SVC 74% saturation RA 82% saturation RV 82% saturation LA 91% saturation LV 91%

Which of the following is the most likely diagnosis? Ostium primum atrial septal defect Secundum atrial septal defect Patent ductus arteriosus Pulmonary stenosis Tricuspid regurgitation A 21-year-old woman presents to the clinic with symptoms of increased shortness of breath and decreased exercise tolerance. She used to be a keen hockey player when at school but is now virtually unable to even walk to the bus stop without becoming short of breath. On examination she looks tired and slightly short of breath at rest. Her BP is elevated at 145/92 mmHg. Investigations; Echocardiogram – increased right atrial size, elevated right arterial pressure by Doppler Cardiac catheterization; O2 saturation SVC 74% O2 saturation RA 82%

(FACEBOOK) https/c of 179

6/30/2011 4:56 PM

www.apnastudent.blogspot.com O2 saturation RV 82% O2 saturation LA 91%

www.apnastudent.blogspot.com O2 saturation LV 91% Which of the following is the most likely diagnosis? Ostium primum atrial septal defect

Your answer

Secundum atrial septal defect

Correct answer

Patent ductus arteriosus Pulmonary stenosis Tricuspid regurgitation

Secundum defects may often be diagnosed in patients entering early adulthood. The difference in saturations between the SVC and the right ventricle indicates that there is a left to right shunt of oxygenated blood. Patients with secundum defects tend to be slim and not to suffer from cyanosis. Therapy of choice is via delivery of a catheter device to close the defect, or surgical closure if the defect is particularly large.

A 52-year-old man is admitted to the intensive therapy unit with left ventricular failure post- myocardial infarction. Despite prompt activity including angioplasty within a few minutes of the onset of chest pain, his systolic BP on admission to the unit was only 80 mmHg, with a pulse of 105/min. Auscultation of the chest revealed crackles up to the mid zones on both sides consistent with cardiac failure. The team decide to insert an intra-aortic balloon pump timed to coincide with the dicrotic notch. What does the dicrotic notch refer to? Aortic valve opening Aortic valve closure Mitral valve opening Mitral valve closure Pulmonary valve closure A 52-year-old man is admitted to the intensive therapy unit with left ventricular failure post- myocardial infarction. Despite prompt activity including angioplasty within a few minutes of the onset of chest pain, his systolic BP on admission to the unit was only 80 mmHg, with a pulse of 105/min. Auscultation of the chest revealed crackles up to the mid zones on both sides consistent with cardiac failure. The team decide to insert an intra-aortic balloon pump timed to coincide with the dicrotic notch. What does the dicrotic notch refer to? Aortic valve opening

Your answer

Aortic valve closure

Correct answer

Mitral valve opening Mitral valve closure Pulmonary valve closure

The dicrotic notch refers to a secondary upstroke in the downward part of the pulse wave which corresponds with closure of the aortic valve. I ntra-aortic balloon pumps deflate during systole which then increases forward blood flow because of the reduction in afterload. The pump then re-inflates during diastole, increasing the blood flow to coronary arteries.

A 72-year-old woman is admitted for assessment after two episodes of collapse over the past few months. She has been managed by her GP for many years for hypertension and is currently treated with indapamide and amlodipine. On examination she looks a little thin. Her BP is 175/125 mmHg. Auscultation of the chest reveals an ejection systolic murmur. She has minor crackles at both lung bases.

(FACEBOOK) https/c of 179

6/30/2011 4:56 PM

www.apnastudent.blogspot.com Investigations; Hb

13.0 g/dl

www.apnastudent.blogspot.com 9

WCC

5.3 x10 /l

PLT

194 x10 /l

Na+

140 mmol/l

+

4.2 mmol/l

K

9

µmol/l

Creatinine

145

Cardiac catheterisation

gradient of 50mmHg across the valve

Which of the following would most influence your decision to refer this patient for valve replacement? Gradient of 50 mmHg Presence of left ventricular hypertrophy Presence of symptoms Presence or absence of valvular calcification Her relatively young age A 72-year-old woman is admitted for assessment after two episodes of collapse over the past few months. She has been managed by her GP for many years for hypertension and is currently treated with indapamide and amlodipine. On examination she looks a little thin. Her BP is 175/125 mmHg. Auscultation of the chest reveals an ejection systolic murmur. She has minor crackles at both lung bases. Investigations; Hb

13.0 g/dl

WCC

5.3 x10 /l

PLT

194 x10 /l

Na+

140 mmol/l

+

4.2 mmol/l

K

9

9

µmol/l

Creatinine

145

Cardiac catheterisation

gradient of 50mmHg across the valve

Which of the following would most influence your decision to refer this patient for valve replacement? Gradient of 50 mmHg Presence of left ventricular hypertrophy Presence of symptoms

Your answer

Presence or absence of valvular calcification Her relatively young age

Whilst in gradient terms her valvular stenosis is on the cusp of the severe category, elective valve replacement is generally not recommended in the absence of symptoms. If there is co-existent coronary artery disease, then elective valve replacement may be undertaken at the same time as CABG. Elevated age is not a barrier to aortic valve replacement, functional status prior to surgery is much more important. In selected patients who are unfit to undergo surgery, balloon valvuloplasty may provide symptomatic relief for a period of 6-12 months.

A 73-year-old woman is admitted for pacemaker insertion because of a number of syncopes and periods of complete heart block identified on 72hr ECG. She receives a DDDR pacemaker. What does the R stand for? Rate limiting Rate modulated Repolarising

(FACEBOOK) https/c of 179

6/30/2011 4:56 PM

www.apnastudent.blogspot.com Rate enhancing Rate reducing A 73-year-old woman is admitted for pacemaker insertion because of a number of syncopes and periods of complete heart block identified on 72hr ECG. She receives a DDDR pacemaker. What does the R stand for? Rate limiting Rate modulated

Your answer

Repolarising Rate enhancing Rate reducing

DDDR stands for dual chamber paced, dual chamber sensed, dual response, rate modulated device. I n other words, the activity of the pacemaker is varied according to the background heart rate. Dual chamber pacing devices are less likely than ventricular pacing only devices, which are associated with increased risk of AV dysyncrony.

A 32-year-old woman is admitted in an unconscious state after an overdose of a large number of amitriptyline tablets. It is thought that she took them between 7 and 8pm and was not found by her partner until he returned from a bar some 3hrs later. When you get to see her she has already been intubated by the Emergency department consultant. Her BP is 100/70 mmHg and she has a sinus tachycardia of 100 BPM. While you are watching the monitor you can see she is suffering from short unsustained runs of ventricular tachycardia. Investigations; pH

7.29

pO2

8.1 kPa

pCO2

4.9 kPa -

HCO3

13 mmol/l

Which of the following is the most appropriate way to initially manage the short runs of VT? Normal saline infusion Magnesium infusion Amiodarone infusion Adenosine bolus IV Sodium bicarbonate A 32-year-old woman is admitted in an unconscious state after an overdose of a large number of amitriptyline tablets. It is thought that she took them between 7 and 8pm and was not found by her partner until he returned from a bar some 3hrs later. When you get to see her she has already been intubated by the Emergency department consultant. Her BP is 100/70 mmHg and she has a sinus tachycardia of 100 BPM. While you are watching the monitor you can see she is suffering from short unsustained runs of ventricular tachycardia. Investigations; pH

7.29

pO2

8.1 kPa

pCO2

4.9 kPa

HCO3-

13 mmol/l

Which of the following is the most appropriate way to initially manage the short runs of VT? Normal saline infusion Magnesium infusion

www.apnastudent.blogspot.com Amiodarone infusion Adenosine bolus IV Sodium bicarbonate

Your answer

Whilst magnesium may be a useful option to control VT in cases of severe tricyclic antidepressant (TCA) overdose, IV sodium bicarbonate is the initial therapy of choice. Increasing the pH to the range of 7.45-7.55 has been shown to decrease the QRS interval, stabilise arrhythmias and increase blood pressure. Other case series state that phenytoin may be useful in correcting conduction defects in this patient population. Conventional class IA, I C, II and III anti-arrhythmic agents should however be avoided.

A 54-year-old man with a history of smoking and hypertension presents to the Emergency room with central crushing chest pain, nausea and sweating. On examination his BP is 104/70 mmHg, his pulse 85/min regular and he looks pale, grey and sweaty. There are no murmurs on auscultation but he has crackles at both lung bases consistent with heart failure. Investigations; Hb

12.8 g/dl

WCC

5.9 x10 /l

PLT

190 x10 9/l

Na+

141 mmol/l

+

5.0 mmol/l

K

9

Creatinine

110 µmol/l

ECG

ST elevation V1-V4, ST depression I I, I II and aVL

Which of the following is the most likely finding on angiography? Hypothermia promotes pulmonary vasoconstriction 70% stenosis of left anterior descending artery Total occlusion of left anterior descending artery 70% stenosis of left circumflex artery Total occlusion of left circumflex artery

A 54-year-old man with a history of smoking and hypertension presents to the Emergency room with central crushing chest pain, nausea and sweating. On examination his BP is 104/70 mmHg, his pulse 85/min regular and he looks pale, grey and sweaty. There are no murmurs on auscultation but he has crackles at both lung bases consistent with heart failure. Investigations; Hb

12.8 g/dl

WCC

9 5.9 x10 / l

PLT

9 190 x10 / l

Na+

141 mmol/l

+

5.0 mmol/l

K

Creatinine

110 µmol/l

ECG

ST elevation V1-V4, ST depression I I, I II and aVL

Which of the following is the most likely finding on angiography? Hypothermia promotes pulmonary vasoconstriction 70% stenosis of left anterior descending artery

(FACEBOOK) https/c of 179

6/30/2011 4:56 PM

www.apnastudent.blogspot.com Total occlusion of left anterior descending artery 70% stenosis of left circumflex artery

Your answer

www.apnastudent.blogspot.com Total occlusion of left circumflex artery

This man is suffering from an ST elevation anterior myocardial infarction and most likely has an occlusion of his left anterior descending artery due to plaque rupture. Management of choice is primary PCI , as early as possible after the onset of chest pain. Studies have now confirmed that primary PCI is superior to thrombolysis with rates of arterial patency in more than 90% of procedures, and lower rates of bleeding complications. Left circumflex or right coronary artery occlusions would be expected to result in either lateral or inferior ECG changes.

A 50-year-old man presents to the cardiology clinic for review. His brother died suddenly of a cardiac arrest while playing Sunday league soccer, and was found on post-mortem to have an underlying diagnosis of hypertrophic obstructive cardiomyopathy (HOCM). Which of the following is the most appropriate way to screen the patient? Exercise ECG Electrocardiography studies Trans-oesophageal ECHO Radionucleotide scanning Resting 12 lead ECG A 50-year-old man presents to the cardiology clinic for review. His brother died suddenly of a cardiac arrest while playing Sunday league soccer, and was found on post-mortem to have an underlying diagnosis of hypertrophic obstructive cardiomyopathy (HOCM). Which of the following is the most appropriate way to screen the patient? Exercise ECG Electrocardiography studies Trans-oesophageal ECHO

Your answer

Radionucleotide scanning Resting 12 lead ECG

Two dimensional echocardiography is diagnostic for HOCM, with TOE delivering the best views. Elevated flow velocity across the left ventricular outflow tract is seen, diastolic dysfunction with reduced LV compliance, systolic anterior motion of the anterior mitral valve leaflet, and asymmetrical septal hypertrophy are also seen. Radionucleotide imaging may show reversible perfusion defects, but these are not diagnostic of the condition. Cardiac catheterisation follows ECHO, to assess the degree of left ventricular outflow tract obstruction, LV anatomy and coronary artery anatomy.

A 54-year-old patient was admitted with central crushing chest pain and had a troponin rise to 3.2µg/L with anterior ST depression. He has a past history of hypertension for which he takes ramipril 10mg, and smokes 20 cigarettes per day. He was recovering on the cardiology ward after angiography and stenting when he started suffering further central chest pain 3 days later. Again his ECG showed anterior ST depression. Investigations; Hb

13.1 g/dl

WCC

9 7.8 x10 / l

PLT

9 201 x10 / l

Na+

139 mmol/l

+

4.9 mmol/l

K

Creatinine

120 µmol/l

(FACEBOOK) https/c of 179

6/30/2011 4:56 PM

www.apnastudent.blogspot.com Which of the following is the most appropriate enzyme screen to look for further myocardial damage?

www.apnastudent.blogspot.com Troponin T Troponin I CKMB LDH AST A 54-year-old patient was admitted with central crushing chest pain and had a troponin rise to 3.2µg/L with anterior ST depression. He has a past history of hypertension for which he takes ramipril 10mg, and smokes 20 cigarettes per day. He was recovering on the cardiology ward after angiography and stenting when he started suffering further central chest pain 3 days later. Again his ECG showed anterior ST depression. Investigations; Hb

13.1 g/dl

WCC

7.8 x10 /l

PLT

201 x10 9 /l

Na K

9

+

139 mmol/l

+

4.9 mmol/l

Creatinine

120 µmol/l

Which of the following is the most appropriate enzyme screen to look for further myocardial damage? Troponin T Troponin I CKMB

Correct answer

LDH

Your answer

AST

Troponin remains elevated for a few days after initial myocardial infarction, and LDH only begins to reach a peak within 3-6 days. I n contrast, as long as serial CKMB measurements had been monitored since admission then a new increase in CKMB would be a good indication of a new event. An increase in white cell count and ESR is also seen after myocardial infarction. ESR may remain elevated for a number of days after infarction. A 32-year-old-woman was cross-country skiing when she fell down a water-filled gully and became trapped beneath an ice-sheet. Frantic efforts were made to extract her, but after 40 minutes all movements ceased. Which of the following statements is true? Her pulseless state is, in all likelihood, due to ventricular fibrillation Defibrillation at the scene is likely to succeed Metabolic alkalosis will quickly set in Aspirated seawater is more likely to produce pulmonary oedema than fresh water It is important to lift her out of water in the prone position A 32-year-old-woman was cross-country skiing when she fell down a water-filled gully and became trapped beneath an ice-sheet. Frantic efforts were made to extract her, but after 40 minutes all movements ceased. Which of the following statements is true? Her pulseless state is, in all likelihood, due to ventricular fibrillation Defibrillation at the scene is likely to succeed

Your answer

Metabolic alkalosis will quickly set in Aspirated seawater is more likely to produce pulmonary oedema than fresh water It is important to lift her out of water in the prone position

Correct answer

(FACEBOOK) https/c of 179

6/30/2011 4:56 PM

www.apnastudent.blogspot.com

The management of patients who nearly drown in cold water is quite different from that for routine cardiopulmonary arrests. Re-warming such patients should be undertaken in a hospital that has extracorporeal re-warming facilities. Head-out upright immersion in water at body temperature results in a 32–66% increase in cardiac output because of the pressure of the surrounding water. Resistance to circulation is suddenly removed as the person leaves the water, which when added to venous pooling can cause circulatory collapse. This is believed to be the cause of death in many individuals. To counter this effect, patients should be lifted out of the water in the prone position. Continuous chest compression should be applied throughout transportation. This alone is as effective as chest compression with expired air resuscitation. Because hypothermia may render the carotid pulse impalpable, it is important to commence chest compression with firm evidence of cardiac arrest. Electrocardiographic monitoring should be available. Defibrillation is ineffective if the myocardium is cold. Evidence suggests that aspirated fresh water is more likely than seawater to produce pulmonary oedema. Metabolic acidosis can develop and should be corrected with adequate oxygenation and plasma expansion. http://www.bmj.com/content/327/7427/1336.full.pdf

A 67-year-old woman presents with syncope. She has suffered two or three episodes of collapse during the past 6 months, the most recent whilst attending church on a Sunday morning. She has a history of hypertension which is currently managed with ramipril and bendroflumethiazide and dyslipidaemia treated with simvastatin. On examination her pulse is 40/min, blood pressure 100/50 mmHg. Her chest is clear and heart sounds are normal. You notice irregular cannon waves on examination of the JVP. Investigations; Hb

12.1 g/dl

WCC

7.4 x10 9/l

PLT

203 x10 /l

Na K

9

+

139 mmol/l

+

4.9 mmol/l

Creatinine

149 µmol/l

Which of the following is the most likely diagnosis? Complete heart block Mobitz type 2 heart block Sinus bradycardia Junctional rhythm Ventricular bigeminy

A 67-year-old woman presents with syncope. She has suffered two or three episodes of collapse during the past 6 months, the most recent whilst attending church on a Sunday morning. She has a history of hypertension which is currently managed with ramipril and bendroflumethiazide and dyslipidaemia treated with simvastatin. On examination her pulse is 40/min, blood pressure 100/50 mmHg. Her chest is clear and heart sounds are normal. You notice irregular cannon waves on examination of the JVP. Investigations; Hb

12.1 g/dl

WCC

7.4 x10 9/l

PLT

203 x10 9 /l

Na K

+

139 mmol/l

+

4.9 mmol/l

Creatinine

149 µmol/l

Which of the following is the most likely diagnosis? Complete heart block

Your answer

www.apnastudent.blogspot.com Mobitz type 2 heart block Sinus bradycardia Junctional rhythm Ventricular bigeminy

Cannon waves occur when the right atrium contracts against a closed tricuspid valve, and these occur irregularly in complete heart block. This diagnosis is entirely in accordance with the presentation with bradycardia, relative hypotension and syncope. Management in this case would be an ECG to confirm the diagnosis and then likely referral for insertion of permanent pacemaker. Cannon waves are also seen in conjunction with ventricular tachycardia. A 46-year-old man is admitted with a tachycardia. He has no previous medical history of note, but admits to excessive use of alcohol and caffeine associated with a particularly stressful period at work during his job as a bond trader. On examination his BP is 122/80 mmHg, his pulse is 180/min. His chest is clear and there are no signs of cardiac failure. Investigations; Hb

12.1 g/dl

WCC

9 5.6 x10 / l

PLT

9 190 x10 / l

Na+

139 mmol/l

+

4.8 mmol/l

K

Creatinine

110 µmol/l

ECG

Narrow complex tachycardia, rate 180/min

You try 3 and 6mg of adenosine IV with no effect. Which of the following is the most appropriate next management step? IV 12mg adenosine IV amiodarone loading IV atenolol IV verapamil IV flecainide

A 46-year-old man is admitted with a tachycardia. He has no previous medical history of note, but admits to excessive use of alcohol and caffeine associated with a particularly stressful period at work during his job as a bond trader. On examination his BP is 122/80 mmHg, his pulse is 180/min. His chest is clear and there are no signs of cardiac failure. Investigations; Hb

12.1 g/dl

WCC

9 5.6 x10 / l

PLT

9 190 x10 / l

Na+

139 mmol/l

+

4.8 mmol/l

K

Creatinine

110 µmol/l

ECG

Narrow complex tachycardia, rate 180/min

You try 3 and 6mg of adenosine IV with no effect. Which of the following is the most appropriate next management step? IV 12mg adenosine

Correct answer

IV amiodarone loading IV atenolol IV verapamil

of 179

(FACEBOOK) https/c 6/30/2011 4:56 PM

www.apnastudent.blogspot.com IV flecainide

Your answer

It is appropriate to try 12mg IV adenosine before moving on to an alternative therapy. Verapamil or short acting beta blockers such as esmolol are both appropriate alternative agents for achieving sinus rhythm. Flecanide is useful for chemical cardioversion of paroxysmal atrial fibrillation in patients with no history of underlying ischaemic heart disease; IV amiodarone is a reasonable alternative. A 30-year-old man presents with 3hrs of central crushing chest pain. He admits to regular cocaine use, including on the evening that he presents to the Emergency department. Other history of note includes smoking 10 cigarettes per day and a family history of mixed hyperlipidaemia. On examination his BP is 220/120 mmHg, but this falls to 180/80 mmHg after diamorphine. He has a sinus tachycardia of 110 beats per minute. He has been given 300mg of aspirin by the ambulance crew. Investigations; Hb

13.8 g/dl

WCC

5.9 x10 9/l

PLT

211 x10 /l

Na+

141 mmol/l

K+

4.9 mmol/l

Creatinine

110 µmol/l

ECG

Anterior ST elevation consistent with acute myocardial infarction

9

Which of the following is the most appropriate way to manage him? Abciximab Percutaneous coronary intervention Low molecular weight heparin Streptokinase rTPA A 30-year-old man presents with 3hrs of central crushing chest pain. He admits to regular cocaine use, including on the evening that he presents to the Emergency department. Other history of note includes smoking 10 cigarettes per day and a family history of mixed hyperlipidaemia. On examination his BP is 220/120 mmHg, but this falls to 180/80 mmHg after diamorphine. He has a sinus tachycardia of 110 beats per minute. He has been given 300mg of aspirin by the ambulance crew. Investigations; Hb

13.8 g/dl

WCC

9 5.9 x10 / l

PLT

9 211 x10 / l

Na+

141 mmol/l

K+

4.9 mmol/l

Creatinine

110 µmol/l

ECG

Anterior ST elevation consistent with acute myocardial infarction

Which of the following is the most appropriate way to manage him? Abciximab Percutaneous coronary intervention Low molecular weight heparin Streptokinase rTPA

Your answer

www.apnastudent.blogspot.com

Guidelines suggest that patients with evidence of acute STEMI should be managed in the same way as those without history of cocaine abuse; as such percutaneous transluminal coronary angioplasty (PTCA)is the most appropriate option. Some notable exceptions do exist however, mainly around use of beta-blockade, which is not recommended in patients with a history of cocaine abuse, as it may worsen coronary artery vasospasm. I nstead, nitrates or calcium channel blockade are recommended as alternatives. With respect to use of low molecular weight heparin (LMWH) or IIbII Ia antagonists, the AHA guidelines endorse their use in a clinically appropriate situation despite the fact that no randomised controlled trial evidence exists. http://circ.ahajournals.org/cgi/content/full/117/14/1897

A 45-year-old man with previously diagnosed Type 2 diabetes presents to the Emergency room with severe central chest pain, nausea and sweating. He was riding his bike in an attempt to lose weight when the chest pain began. He also smokes 15 cigarettes per day and is hypertensive, managed with ramipril and amlodipine. On examination his BP is 155/95 mmHg, he is pale, sweaty and anxious. He has been given 300mg of aspirin by a passer-by. Investigations; Hb

13.2 g/dl

WCC

5.9 x10 /l

PLT

209 x10 9 /l

Na+

141 mmol/l

+

4.9 mmol/l

K

9

Creatinine

130 µmol/l

Glucose

17.1 mmol/l

ECG

Inferior ST elevation

Which of the following is the most appropriate intervention? Low molecular weight heparin Alteplase Streptokinase Percutaneous coronary intervention Abciximab

A 45-year-old man with previously diagnosed Type 2 diabetes presents to the Emergency room with severe central chest pain, nausea and sweating. He was riding his bike in an attempt to lose weight when the chest pain began. He also smokes 15 cigarettes per day and is hypertensive, managed with ramipril and amlodipine. On examination his BP is 155/95 mmHg, he is pale, sweaty and anxious. He has been given 300mg of aspirin by a passer-by. Investigations; Hb

13.2 g/dl

WCC

5.9 x10 /l

PLT

9 209 x10 / l

Na+

141 mmol/l

+

4.9 mmol/l

K

9

Creatinine

130 µmol/l

Glucose

17.1 mmol/l

ECG

Inferior ST elevation

Which of the following is the most appropriate intervention? Low molecular weight heparin Alteplase

of 179

(FACEBOOK) https/c 6/30/2011 4:56 PM

www.apnastudent.blogspot.com Streptokinase Percutaneous coronary intervention

Your answer

Abciximab

Studies have shown that in patients with acute STEMI , percutaneous coronary intervention is superior to thrombolysis. As such PCI is recommended here above both streptokinase and alterplase. Whilst some acute trusts still do not yet have access to an acute angioplasty service, development of this has been made a priority for the next few years. A 70-year-old man with a history of extensive acute myocardial infarction 4 years earlier comes to the hospital with his wife. He has suffered 4 episodes of collapse over the past 6 months, the most recent that morning, when she witnessed slurred speech, confusion and weakness of his right arm and leg. On examination he has no chest pain, his BP is 145/82 mmHg, and he is not in cardiac failure. His apex beat is displaced to the left. Investigations; Hb

12.1 g/dl

WCC

5.9 x10 /l

PLT

187 x10 9/l

Na+

142 mmol/l

+

5.1 mmol/l

K

9

Creatinine

148 µmol/l

ECG

ST elevation in the anterior leads

Which of the following is the most appropriate way to diagnose his primary underlying pathology? Monitor on the CCU Thrombolyse with TPA Refer for immediate PCI Arrange an urgent CT head Arrange a cardiac MRI A 70-year-old man with a history of extensive acute myocardial infarction 4 years earlier comes to the hospital with his wife. He has suffered 4 episodes of collapse over the past 6 months, the most recent that morning, when she witnessed slurred speech, confusion and weakness of his right arm and leg. On examination he has no chest pain, his BP is 145/82 mmHg, and he is not in cardiac failure. His apex beat is displaced to the left. Investigations; Hb

12.1 g/dl

WCC

9 5.9 x10 / l

PLT

9 187 x10 / l

Na+

142 mmol/l

+

5.1 mmol/l

K

Creatinine

148 µmol/l

ECG

ST elevation in the anterior leads

Which of the following is the most appropriate way to diagnose his primary underlying pathology? Monitor on the CCU Thrombolyse with TPA Refer for immediate PCI Arrange an urgent CT head

Your answer

Arrange a cardiac MRI

Correct answer

www.apnastudent.blogspot.com

Persistent ST elevation in the absence of chest pain in a patient who has a history of previous anterior myocardial infarction raises the possibility of left ventricular aneurysm. Cardiac MRI is an effective way to image the aneurysm non-invasively, and would be the preferred initial investigation. The presence of multiple TI As raises the possibility of thrombus formation. Hence anti-coagulation may be considered here, with possible referral for surgical excision of aneurysm. A 67-year-old man is referred to the cardiology clinic with angina, progressive heart failure and two episodes of syncope. He has a history of hypertension managed with ramipril and indapamide and suffered an inferior myocardial infarction some 4 years ago. On examination his BP is 125/105 mmHg, and he has a soft ejection systolic murmur loudest at the apex. He has evidence of LVH and there are bilateral inspiratory crackles on auscultation of the chest consistent with LVF. Investigations; Hb

12.4 g/dl

WCC

6.1 x10 /l

PLT

9 208 x10 / l

Na+

140 mmol/l

+

4.3 mmol/l

K

Creatinine

9

185 µmol/l

Which of the following is likely to be the most significant problem which is driving his symptoms? Coronary artery disease Mitral regurgitation Aortic stenosis Cardiac arrhythmias Chronic renal failure A 67-year-old man is referred to the cardiology clinic with angina, progressive heart failure and two episodes of syncope. He has a history of hypertension managed with ramipril and indapamide and suffered an inferior myocardial infarction some 4 years ago. On examination his BP is 125/105 mmHg, and he has a soft ejection systolic murmur loudest at the apex. He has evidence of LVH and there are bilateral inspiratory crackles on auscultation of the chest consistent with LVF. Investigations; Hb

12.4 g/dl

WCC

9 6.1 x10 / l

PLT

9 208 x10 / l

Na+

140 mmol/l

+

4.3 mmol/l

K

Creatinine

185 µmol/l

Which of the following is likely to be the most significant problem which is driving his symptoms? Coronary artery disease Mitral regurgitation Aortic stenosis

Your answer

Cardiac arrhythmias Chronic renal failure

The triad of angina, LVF and syncope is classical with respect to aortic stenosis. Two confounders exist: in the elderly the more high frequency components of aortic stenosis may be heard best at the apex, the so called Gallavardin

(FACEBOOK) https/c

of 179

6/30/2011 4:56 PM

www.apnastudent.blogspot.com phenomenon, and the components of the murmur may be softened in situations where cardiac output is reduced. Given

www.apnastudent.blogspot.com this man has evidence of coronary artery disease he may well have co-existent reduced cardiac output. Hence he requires assessment of both aortic valve and coronary artery status, with combined valve replacement and CABG likely to be the most appropriate way to manage him. A 52-year-old male is undergoing exercise tolerance testing for coronary artery disease screening after suffering indigestion type pain whilst playing squash with a workmate. He reaches stage I I of the Bruce protocol when his BP is 210/100 mmHg and HR 170/min. ECG changes are noted. Which of the following is the strongest indicator of underlying arterial disease for stopping the test? His BP of 210/100 mmHg His heart rate 2mm ST depression in the lateral leads Patient request Ventricular ectopics on the monitor

A 52-year-old male is undergoing exercise tolerance testing for coronary artery disease screening after suffering indigestion type pain whilst playing squash with a workmate. He reaches stage I I of the Bruce protocol when his BP is 210/100 mmHg and HR 170/min. ECG changes are noted. Which of the following is the strongest indicator of underlying arterial disease for stopping the test? His BP of 210/100 mmHg His heart rate 2mm ST depression in the lateral leads

Your answer

Patient request Ventricular ectopics on the monitor

Hypertension of greater than 250/115 mmHg is usually considered as a reason to discontinue the test. A drop of more than 10mmHg blood pressure in the presence of other evidence of ischaemia is also a reason for discontinuing the exercise test. In the presence of an achieved heart rate of 170/min, ST depression of 2mm is an entirely appropriate reason for discontinuing the test. Ventricular ectopics, rather than sustained VT are acceptable, and the test need not be stopped for these. A 62-year-old man presents to the clinic with increasing shortness of breath. He has a history of smoking 10 cigarettes per day and hypertension which he takes ramipril 10mg daily. On examination his BP is 152/87 mmHg, pulse is 75/min and there is reversed splitting of the second heart sound. There are no signs of cardiac failure. Investigations; Hb

13.2 g/dl

WCC

9 5.6 x10 / l

PLT

9 209 x10 / l

Na+

139 mmol/l

+

4.5 mmol/l

K

Creatinine

132 µmol/l

Which of the following is the most likely diagnosis? Atrial septal defect Left bundle branch block Mitral regurgitation Right bundle branch block Ventricular septal defect A 62-year-old man presents to the clinic with increasing shortness of breath. He has a history of smoking 10 cigarettes per day and hypertension which he takes ramipril 10mg daily. On examination his BP is 152/87 mmHg, pulse is 75/min and

(FACEBOOK) https/c

of 179

6/30/2011 4:56 PM

www.apnastudent.blogspot.com there is reversed splitting of the second heart sound. There are no signs of cardiac failure.

www.apnastudent.blogspot.com Investigations; Hb

13.2 g/dl

WCC

5.6 x10 9/l

PLT

209 x10 /l

Na+

139 mmol/l

+

4.5 mmol/l

K

9

Creatinine

132 µmol/l

Which of the following is the most likely diagnosis? Atrial septal defect Left bundle branch block

Your answer

Mitral regurgitation Right bundle branch block Ventricular septal defect

Reversed splitting of the second heart sound occurs when the sound from the closure of the pulmonary valve occurs prior to that from the closure of the aortic valve. Causes include aortic stenosis, left bundle branch block and hypertrophic obstructive cardiomyopathy. Fixed splitting of the second heart sound where aortic valve closure occurs before the pulmonary, and which doesn’t vary with inspiration, occurs with an ASD or VSD. A 52-year-old woman presents to the clinic with shortness of breath and angina. She has a history of hypertension and Type 2 diabetes for which she takes metformin and BD mixed insulin. On examination her BP is 155/92 mmHg. You notice reversed splitting of the second heart sound and bibasal crackles on auscultation of the chest consistent with cardiac failure. Which of the following is the most likely finding on ECG? Right bundle branch block QT prolongation P mitrale P pulmonale Left bundle branch block A 52-year-old woman presents to the clinic with shortness of breath and angina. She has a history of hypertension and Type 2 diabetes for which she takes metformin and BD mixed insulin. On examination her BP is 155/92 mmHg. You notice reversed splitting of the second heart sound and bibasal crackles on auscultation of the chest consistent with cardiac failure. Which of the following is the most likely finding on ECG? Right bundle branch block QT prolongation P mitrale P pulmonale Left bundle branch block

Your answer

Reversed splitting of the second heart sound occurs when closure of the pulmonary valve occurs before the aortic valve. Conditions associated with reversed splitting of the second heart sound include left bundle branch block, hypertrophic obstructive cardiomyopathy and aortic stenosis. Fixed splitting of the second heart sound is associated with an atrial septal or ventricular septal defect. You are designing a study for a new agent that may reduce myocardial necrosis after myocardial infarction. The agent is specifically thought to reduce early myonecrosis.

(FACEBOOK) https/c

of 179

6/30/2011 4:56 PM

www.apnastudent.blogspot.com Which of the following enzymes is most appropriate to measure early myocardial necrosis?

www.apnastudent.blogspot.com Glycogen phosphorylase isoenzyme BB (GPBB) Myoglobin Creatinine kinase Troponin Lactate dehydrogenase You are designing a study for a new agent that may reduce myocardial necrosis after myocardial infarction. The agent is specifically thought to reduce early myonecrosis. Which of the following enzymes is most appropriate to measure early myocardial necrosis? Glycogen phosphorylase isoenzyme BB (GPBB)

Correct answer

Myoglobin

Your answer

Creatinine kinase Troponin Lactate dehydrogenase

Glycogen phosphorylase exists in a number of isoforms, but GPBB exists in heart and brain tissue. During a period of ischaemia GPBB is released and is elevated 1-3 hrs after the event. Myoglobin levels become significantly elevated 2hrs after ischaemia, for CK the level may not be markedly elevated until 4hrs or more after an infarct. LDH is a late marker of myocardial infarction which remains elevated for a few days after infarct. Troponin is the current gold standard marker for myocardial infarction, although levels only become elevated after 6hrs; conventional practice is to measure levels at 6 and 12hrs after a period of chest pain. A 79 year-old-man known to have chronic congestive heart failure is readmitted with worsening heart failure. His furosemide (frusemide) dosage is increased to 200 mg/day to aid the relief of his symptoms. His other medications are bendrofluazide, ramipril and bisoprolol. Which of the following effects is most likely to be encountered? Hyperkalaemia Hypercalciuria Hypermagnesaemia Hyperuricaemia Hypoalbuminaemia

A 79 year-old-man known to have chronic congestive heart failure is readmitted with worsening heart failure. His furosemide (frusemide) dosage is increased to 200 mg/day to aid the relief of his symptoms. His other medications are bendrofluazide, ramipril and bisoprolol. Which of the following effects is most likely to be encountered? Hyperkalaemia Hypercalciuria Hypermagnesaemia Hyperuricaemia

Your answer

Hypoalbuminaemia

Diuretics can cause various electrolyte disturbances, eg hyponatraemia, hypomagnesaemia and hypokalaemia. Metabolic alkalosis results from chloride loss and extracellular fluid volume contraction. Extracellular volume depletion and raised urea concentrations can occur due to renal hypoperfusion. Typically, however, serum urea concentrations are unchanged or do not increase by more than 10–20% of baseline values. Tubular handling of uric acid is complex, with both reabsorption and secretion occurring in the proximal tubule. Diuretics

(FACEBOOK) https/c of 179

6/30/2011 4:56 PM

www.apnastudent.blogspot.com can interfere with either of these processes thereby causing hyperuricaemia. The effect is usually dose-dependent and frequently asymptomatic. Clinical gout is more likely if the patient is also extracellular fluid volume-depleted.

www.apnastudent.blogspot.com Ototoxicity is an uncommon side-effect occurring with high-dose loop diuretic therapy. It has been noted in patients with renal failure who are given high-dose (> 2 g/day) infusion therapy.

A 74-year-old man presents with acute onset palpitations, ECG showed regular tachycardia with a ventricular rate of 150. He has a history of hypertension treated with ramipril and amlodipine, but has no other significant past medical history. He smokes 5 cigars per day. On examination his BP is 110/70 mmHg; his pulse is very rapid, at least 130/min. Investigations; Hb

12.9 g/dl

WCC

6.7 x10 /l

PLT

9 291 x10 / l

Na+

141 mmol/l

K+

5.3 mmol/l

Creatinine

141 µmol/l

ECG

regular narrow complex tachycardia with ventricular rate of 150BPM; saw-tooth pattern particularly evident in leads I I, II I and aVF

9

Which of the following rhythms is it most likely to be? Idioventricular tachycardia with dissociated AV response Atrial flutter Atrial fibrillation AV nodal re-entrant tachycardia Atrioventricular re-entrant tachycardia A 74-year-old man presents with acute onset palpitations, ECG showed regular tachycardia with a ventricular rate of 150. He has a history of hypertension treated with ramipril and amlodipine, but has no other significant past medical history. He smokes 5 cigars per day. On examination his BP is 110/70 mmHg; his pulse is very rapid, at least 130/min. Investigations; Hb

12.9 g/dl

WCC

6.7 x10 /l

PLT

9 291 x10 / l

Na+

141 mmol/l

K+

5.3 mmol/l

Creatinine

141 µmol/l

ECG

regular narrow complex tachycardia with ventricular rate of 150BPM; saw-tooth pattern particularly evident in leads I I, II I and aVF

9

Which of the following rhythms is it most likely to be? Idioventricular tachycardia with dissociated AV response Atrial flutter

Your answer

Atrial fibrillation AV nodal re-entrant tachycardia Atrioventricular re-entrant tachycardia

The saw tooth pattern which is particularly well characterised in leads II , III and aVF is typical of atrial flutter with 2:1 block. It is the second most common tachyarrhythmia after atrial fibrillation, prevalence in the 65-90 year age group being around 0.5-1%. Electrical cardioversion may be attempted; otherwise amiodarone is the drug of choice for chemical cardioversion. I n patients in whom cardioversion is unsuccessful, ventricular rate may be controlled with

(FACEBOOK) https/c

of 179

6/30/2011 4:56 PM

www.apnastudent.blogspot.com non-dihydropyridine calcium channel blockers such as verapamil or diltiazem, or cardioselective beta blockers such as

www.apnastudent.blogspot.com metoprolol. An 82-year-old man presents with syncopal attacks to the cardiology clinic. His GP has been treating him for worsening hypertensive heart failure. He takes 80mg of furosemide and 10mg of ramipril, with the recent addition of a small dose of spironolactone. Around 2 weeks ago he presented with an angina attack to the Emergency room after getting chest pain whilst out shopping at the supermarket. On examination his BP is 165/122 mmHg. His pulse is 92/min, heart sounds reveal a systolic murmur loudest in the aortic area, but no ejection click. He has bibasal crackles on auscultation of the chest. Investigations; Hb

12.9 g/dl

WCC

9 5.9 x10 / l

PLT

9 189 x10 / l

Na+

138 mmol/l

+

5.4 mmol/l

K

Creatinine

201 µmol/l

CXR

Bilateral basal infiltrates consistent with fluid, and cardiomegaly

Which of the following is the most likely cause of his aortic stenosis? Subacute bacterial endocarditis Rheumatic fever Bicuspid aortic valve Malformed tricuspid aortic valve Senile degenerative aortic stenosis An 82-year-old man presents with syncopal attacks to the cardiology clinic. His GP has been treating him for worsening hypertensive heart failure. He takes 80mg of furosemide and 10mg of ramipril, with the recent addition of a small dose of spironolactone. Around 2 weeks ago he presented with an angina attack to the Emergency room after getting chest pain whilst out shopping at the supermarket. On examination his BP is 165/122 mmHg. His pulse is 92/min, heart sounds reveal a systolic murmur loudest in the aortic area, but no ejection click. He has bibasal crackles on auscultation of the chest. Investigations; Hb

12.9 g/dl

WCC

9 5.9 x10 / l

PLT

9 189 x10 / l

Na+

138 mmol/l

+

5.4 mmol/l

K

Creatinine

201 µmol/l

CXR

Bilateral basal infiltrates consistent with fluid, and cardiomegaly

Which of the following is the most likely cause of his aortic stenosis? Subacute bacterial endocarditis Rheumatic fever Bicuspid aortic valve

Your answer

Malformed tricuspid aortic valve Senile degenerative aortic stenosis

Correct answer

Senile degenerative aortic stenosis involves progressive calcification of the valve leaflets, in response to long-standing haemodynamic stress. It represents the commonest cause of aortic valve replacement, usually presenting after the age of 75. Diabetes and hypercholesterolemia are risk factors for development of the lesion. Rheumatic fever related aortic stenosis tends to present earlier in patients in their sixties. Congenital bicuspid aortic stenosis presents in patients in

(FACEBOOK) https/c of 179

6/30/2011 4:56 PM

www.apnastudent.blogspot.com the 40-50 year age bracket. Patients with senile degenerative aortic stenosis may well have co-existent coronary artery

www.apnastudent.blogspot.com disease, in which case CABG is carried out at the same time as valve replacement. A 55-year-old man has infective endocarditis. He has refused dental work for a number of years and presented with night sweats and lethargy to the GP. Subsequent blood cultures demonstrated a S viridans bacteraemia. On examination he is o

pyrexial 37.8 C, his BP is 110/70 mmHg, pulse is 95/min. He has splinter haemorrhages on examination of his fingers. Auscultation reveals a murmur consistent with mitral regurgitation. His chest is clear. Investigations; Hb

10.7 g/dl

WCC

13.1 x10 /l

PLT

211 x10 /l

Na+

139 mmol/l

9

9

K+ Creatinine

4.9 mmol/l 139 µmol/l

ESR

72 mm/hr

ECG on admission

PR interval 180ms, otherwise no significant changes mitral regurgitation with evidence of vegetations

Echocardiogram

Which of the following would be considered an urgent indication for surgical intervention after 12 days of antibiotic therapy? Increasing PR interval Systolic murmur after 10 days of antibiotics Increasing fever Raised ESR Mobile vegetation >15mm in size

A 55-year-old man has infective endocarditis. He has refused dental work for a number of years and presented with night sweats and lethargy to the GP. Subsequent blood cultures demonstrated a S viridans bacteraemia. On examination he is pyrexial 37.8 o C, his BP is 110/70 mmHg, pulse is 95/min. He has splinter haemorrhages on examination of his fingers. Auscultation reveals a murmur consistent with mitral regurgitation. His chest is clear. Investigations; 10.7 g/dl

Hb

9

WCC

13.1 x10 /l

PLT

211 x10 /l

Na+

139 mmol/l

9

K+ Creatinine

4.9 mmol/l 139 µmol/l

ESR

72 mm/hr

ECG on admission

PR interval 180ms, otherwise no significant changes

Echocardiogram

mitral regurgitation with evidence of vegetations

Which of the following would be considered an urgent indication for surgical intervention after 12 days of antibiotic therapy? Increasing PR interval

Your answer

Systolic murmur after 10 days of antibiotics Increasing fever Raised ESR Mobile vegetation >15mm in size

(FACEBOOK) https/c of 179

6/30/2011 4:56 PM

www.apnastudent.blogspot.com

Increase of the PR interval suggests extension of the endocarditic infection into the myocardium. This also raises the possibility of abscess formation, particularly given the length of time of antibiotic treatment so far, as such urgent surgical referral is indicated. Other indications for urgent surgical intervention include mitral regurgitation or aortic regurgitation with heart failure, septal perforation and valvular obstruction. Rupture into the pericardium is an indication for same day surgical intervention. Heart. 2004 June; 90(6): 618–620

A 68-year-old woman recently diagnosed with multiple myeloma presents to her GP with progressively increasing breathlessness, exercise intolerance and ankle swelling. On examination, there is bilateral pitting leg oedema to her thighs, ascites and raised JVP. The apical impulse is impalpable. An ECG shows diffusely diminished voltage. Chest X-ray is normal and the echocardiogram shows small thick ventricles and dilated atria with a thickened interatrial septum. The ventricular myocardium has a granular sparkling texture on echo, and minimal fluid in the pericardial space is noted. What is the most likely diagnosis? Chronic pericardial effusion with tamponade Chronic pericardial effusion without tamponade Constrictive pericarditis Restrictive cardiomyopathy Congestive heart failure

A 68-year-old woman recently diagnosed with multiple myeloma presents to her GP with progressively increasing breathlessness, exercise intolerance and ankle swelling. On examination, there is bilateral pitting leg oedema to her thighs, ascites and raised JVP. The apical impulse is impalpable. An ECG shows diffusely diminished voltage. Chest X-ray is normal and the echocardiogram shows small thick ventricles and dilated atria with a thickened interatrial septum. The ventricular myocardium has a granular sparkling texture on echo, and minimal fluid in the pericardial space is noted. What is the most likely diagnosis? Chronic pericardial effusion with tamponade Chronic pericardial effusion without tamponade Constrictive pericarditis Restrictive cardiomyopathy

Your answer

Congestive heart failure

Cardiac involvement is the most common cause of death in patients with amyloidosis associated with an immunocyte dyscrasia – typically as restrictive cardiomyopathy. Physical examination reveals right heart failure with a raised jugular venous pressure (JVP), characteristically showing a prominent deep Y descent. The heart size is often normal. The physical findings are very similar in constrictive pericarditis (CCP), but the apex is frequently non-palpable due to the thick pericardium. The chest X-ray may show pericardial calcifications in patients with constrictive pericarditis. The most characteristic ECG finding of restrictive cardiomyopathy is diffusely diminished voltages. Echocardiography typically shows small thick ventricles and a thick interatrial septum due to amyloid deposits, which have a ‘granular sparkling’ appearance. Pericardial effusion is common, but rarely causes tamponade.

A 62-year-old-man with a blood pressure of 160/98 mmHg, total serum cholesterol of 6.5 mmol/l and HDL of 1.3 mmol/l is seen by his GP. On reviewing his records it appears the nurse has also documented hypertensive BPs at his last two well man checks. He is not diabetic and has never smoked. His family history is unknown as he was adopted. There is a history of exertional angina which is worse on hill climbing and walking his dog in cold weather. Apart from advice on lifestyle modification, which of the following combination of drugs should he receive under current guidelines? Aspirin, antihypertensive treatment Statin, aspirin

www.apnastudent.blogspot.com Clopidogrel, aspirin, statin

(FACEBOOK) https/c of 179

6/30/2011 4:56 PM

www.apnastudent.blogspot.com Antihypertensive treatment, aspirin, statin Antihypertensive treatment, clopidogrel, statin A 62-year-old-man with a blood pressure of 160/98 mmHg, total serum cholesterol of 6.5 mmol/l and HDL of 1.3 mmol/l is seen by his GP. On reviewing his records it appears the nurse has also documented hypertensive BPs at his last two well man checks. He is not diabetic and has never smoked. His family history is unknown as he was adopted. There is a history of exertional angina which is worse on hill climbing and walking his dog in cold weather. Apart from advice on lifestyle modification, which of the following combination of drugs should he receive under current guidelines? Aspirin, antihypertensive treatment Statin, aspirin Clopidogrel, aspirin, statin Antihypertensive treatment, aspirin, statin

Your answer

Antihypertensive treatment, clopidogrel, statin

Several treatments reduce the risk of coronary artery disease, the absolute benefits being proportional to the pre-treatment risk: note that individual patients may be eligible for more than one treatment. The most cost-effective preventive treatments are aspirin, statins, and anti-hypertensives, with ACE inhibition or calcium antagonists the first line agents for treating blood pressure.

A 50-year-old-professor of economics presents with a 6-week history of progressive breathlessness and bilateral ankle swelling. The ECG shows inverted p in V 1 and partial LBBB. Echocardiography confirms dilated cardiomyopathy. Which of the following statements is true? There is no relevance of history of alcohol abuse. A family history of similar problem is unrelated Past cytotoxic drug therapy is relevant Viral illness in the past is unrelated History of pulmonary tuberculosis in the past is relevant A 50-year-old-professor of economics presents with a 6-week history of progressive breathlessness and bilateral ankle swelling. The ECG shows inverted p in V 1 and partial LBBB. Echocardiography confirms dilated cardiomyopathy. Which of the following statements is true? There is no relevance of history of alcohol abuse. A family history of similar problem is unrelated Past cytotoxic drug therapy is relevant

Your answer

Viral illness in the past is unrelated History of pulmonary tuberculosis in the past is relevant

Dilated cardiomyopathy (DCM) is characterised by dilatation and impaired systolic function of the left and/or right ventricle. The aetiology is idiopathic in the majority of cases. DCM is familial (autosomal dominant) in at least 20% of cases and a role of autoimmunity is proposed in the pathogenesis of this disease. About 30–40% of patients with DCM have organ-specific antibodies and these may become negative with disease progression. There is an association with viral (coxsackievirus or HIV) infection, which may be immune-related.

www.apnastudent.blogspot.com Many patients with systemic heart disease present with features of DCM: · Alcohol abuse · Cytotoxic drug therapy, eg doxorubicin, cyclophosphamide · Primary heart muscle diseases, eg amyloidosis

(FACEBOOK) https/c of 179

6/30/2011 4:56 PM

www.apnastudent.blogspot.com · End-stage cardiovascular disease (ischaemic, rheumatic, congenital, systemic hypertension) · Generalised disease, eg haemachromatosis, sarcoidosis · Connective tissue disorders, eg systemic sclerosis, systemic lupus erythematosus

A 68-year-old-man with atrial fibrillation (AF) is admitted electively for DC cardioversion, to be performed as a day-case procedure. However, the procedure is postponed to a later date. Which one of the following reasons could be responsible for the delay? He had discontinued digoxin for the last 2 days He was taking amiodarone His I NR 3 weeks ago was 1.6 His serum potassium level was 4.2 mEq/l He had an episode of angina 2 days ago A 68-year-old-man with atrial fibrillation (AF) is admitted electively for DC cardioversion, to be performed as a day-case procedure. However, the procedure is postponed to a later date. Which one of the following reasons could be responsible for the delay? He had discontinued digoxin for the last 2 days

Your answer

He was taking amiodarone His I NR 3 weeks ago was 1.6

Correct answer

His serum potassium level was 4.2 mEq/l He had an episode of angina 2 days ago

Although the INR on the day of cardioversion is important, the I NR should be optimal in the preceding 3-4 weeks prior to cardioversion. External cardioversion is a safe and effective method for restoring sinus rhythm, and should be attempted at least once in every patient with chronic AF. Overt congestive heart failure, hypokalaemia and hyperthyroidism should be controlled as much as possible before cardioversion. Acute myocardial infarction is not a contraindication to cardioversion. If the patient has a slow ventricular response of AF in the absence of anti-arrhythmic drugs, cardioversion should be performed after the insertion of a temporary transvenous-pacing catheter. Electrical cardioversion is initially successful in 70–94% cases, but relapse is frequent (25–50% at 1 month and 70–90% at 1 year). The success depends on the duration of AF, transthoracic impedance, left atrial size and the age of the patient. Digoxin should be withheld on the day of cardioversion. However, if digoxin toxicity is suspected, the problem should be resolved before cardioversion is attempted. Pre-treatment with amiodarone or sotalol may prevent early recurrence. The initial shock strength should be 100 J, followed by a second 200-J shock and a third 360-J shock. If AF persists, a second 360-J shock with the paddles in the anteroposterior position can be attempted. I mmediate DC cardioversion, after the administration of intravenous heparin, is appropriate in an emergency or if AF has been present for less than 48 hours. In elective cases, patients should be established on warfarin to give an I NR of between 2 and 3 for a minimum of three weeks prior to cardioversion. Anticoagulation should be continued for four weeks after successful cardioversion.

A 44-year-old man presents with a 2-hour history of severe central chest pain. ECG shows ST elevation in the anterior leads. He was recently discharged following a laparotomy for intestinal obstruction. What would be the best line of treatment for him? Aspirin and clopidogrel Streptokinase Coronary angioplasty Intravenous heparin Alteplase

www.apnastudent.blogspot.com A 44-year-old man presents with a 2-hour history of severe central chest pain. ECG shows ST elevation in the anterior leads. He was recently discharged following a laparotomy for intestinal obstruction. What would be the best line of treatment for him? Aspirin and clopidogrel Streptokinase Coronary angioplasty

Your answer

Intravenous heparin Alteplase

The symptoms and investigations suggest an acute anterior myocardial infarction. The most appropriate approach here is angioplasty. Given the superiority for angioplasty over thrombolysis in the general MI population, and the contraindication because of recent surgery to thrombolysis, the correct answer is obvious.

Which of troponin?

the

following

is

a

characteristic

feature

of

It is an integral component of pericardial cells Levels rise immediately or even prior to the onset of chest pain due to myocardial infarction About 30% of infarct patients show a rise in levels at 12 hours from the onset of symptoms 1 ng/ml is the cut off above which a myocardial infarction is indicated Levels act as a prognostic factor following an acute coronary syndrome

Which of troponin?

the

following

is

a

characteristic

feature

of

It is an integral component of pericardial cells Levels rise immediately or even prior to the onset of chest pain due to myocardial infarction About 30% of infarct patients show a rise in levels at 12 hours from the onset of symptoms 1 ng/ml is the cut off above which a myocardial infarction is indicated Levels act as a prognostic factor following an acute coronary syndrome Correct answer

Your answer

The troponin complex is part of the cardiac myofibril and is released in myocardial damage. Levels rise about 4 hours after the onset of chest pain. 100% of patients are positive for troponin at 12 hours after the onset of pain. A level of > 0.1 ng/ml is considered as a significant rise. Levels of troponin have a strong relationship to clinical outcomes, such as progression to myocardial infarction and death.

A 36-year-old woman who is 8 weeks’ pregnant presents with a swollen left leg. Doppler studies confirm a deep vein thrombosis. What would be the management in this case? Commence intravenous heparin Start subcutaneous heparin throughout pregnancy and change to warfarin in the postpartum period Oral anticoagulation with warfarin daily throughout pregnancy and the postpartum period Aspirin 300 mg daily throughout pregnancy and the postpartum period Elastic band compress of her left leg, bedrest and foot elevation A 36-year-old woman who is 8 weeks’ pregnant presents with a swollen left leg. Doppler studies confirm a deep vein

(FACEBOOK) https/c

of 179

6/30/2011 4:56 PM

www.apnastudent.blogspot.com thrombosis. What would be the management in this case? Commence intravenous heparin Start subcutaneous heparin throughout pregnancy and change to warfarin in the postpartum period

Your answer

www.apnastudent.blogspot.com Oral anticoagulation with warfarin daily throughout pregnancy and the postpartum period Aspirin 300 mg daily throughout pregnancy and the postpartum period Elastic band compress of her left leg, bedrest and foot elevation

Anticoagulation is essential in this condition. Warfarin is contraindicated in the first trimester as it is teratogenic. Heparin is relatively safer and should be the drug of choice for anticoagulation. Either unfractionated or low molecular weight heparin can be used throughout pregnancy. I V heparin can be used around the time of delivery for easy reversibility. Warfarin may be given in the postpartum period. High doses of aspirin are contraindicated in pregnancy as it can cause premature ductal closure. See SIGN guidelines on anti-thrombotic therapy for full discussion of this topic. http://www.sign.ac.uk/guidelines/fulltext/36/section2.html

A young computer programmer suddenly develops dysphasia and right-sided weakness. Cardiac examination is normal and he is afebrile. Which investigation would confirm the underlying cardiological diagnosis? Chest X-ray 12-lead ECG 2-D echocardiography Carotid Doppler study Transoesophageal echocardiogram A young computer programmer suddenly develops dysphasia and right-sided weakness. Cardiac examination is normal and he is afebrile. Which investigation would confirm the underlying cardiological diagnosis? Chest X-ray 12-lead ECG 2-D echocardiography Carotid Doppler study Transoesophageal echocardiogram

Your answer

A young, otherwise healthy person who suddenly develops a stroke is likely to have a paradoxical embolism due to a patent foramen ovale (PFO or ostium secundum defect). PFOs are relatively common and may be present in 30% of the general population. PFOs greater than 4 mm and associated with shunting are more likely to be associated with systemic emboli. A chest X-ray may show enlargement of the heart and pulmonary artery as well as pulmonary plethora. Incomplete right bundle branch block is seen on ECG. Echocardiography may demonstrate the defect and show right ventricular dilatation and hypertrophy and pulmonary artery dilatation. However, PFOs may be missed on 2-D echocardiogram. The precise size and location of the defect can be shown on transoesophageal echocardiography. A transoesophageal echocardiogram with Doppler colour-flow imaging would be the investigation of choice in this case.

Right ventricular myocardial infarction is characterised by which of the following? ST-segment elevation in leads I I, I II and aVF with Q waves and T-wave inversion in these leads Occlusion of the left coronary artery Marked pulmonary vascular congestion A rise in systolic blood pressure Absent Kussmaul’s sign

of 179

(FACEBOOK) https/c 6/30/2011 4:56 PM

www.apnastudent.blogspot.com Right ventricular myocardial infarction is characterised by which of the following? ST-segment elevation in leads I I, I II and aVF with Q waves and T-wave inversion in these leads

Your answer

www.apnastudent.blogspot.com Occlusion of the left coronary artery Marked pulmonary vascular congestion A rise in systolic blood pressure Absent Kussmaul’s sign

Right ventricular myocardial infarction usually occurs in association with an inferior-wall left ventricular infarction, as revealed by the ECG. There is usually a right coronary occlusion. Characteristic clinical features include a low cardiac output syndrome with jugular venous distension but no pulmonary vascular congestion. Kussmaul’s sign (increased jugular venous distension with inspiration) may be evident.

An elderly man develops syncope. He is known to have ischaemic heart disease. Peripheral pulses are absent and ECG reveals wide-complex tachycardia. Immediate management consists of which of the following? Programmed stimulation Thrombolysis with TPA or streptokinase A bolus dose of intravenous lidocaine DC cardioversion Intravenous phenylephrine and carotid sinus pressure An elderly man develops syncope. He is known to have ischaemic heart disease. Peripheral pulses are absent and ECG reveals wide-complex tachycardia. Immediate management consists of which of the following? Programmed stimulation Thrombolysis with TPA or streptokinase A bolus dose of intravenous lidocaine DC cardioversion

Your answer

Intravenous phenylephrine and carotid sinus pressure

The patient has ventricular tachycardia and is haemodynamically unstable. This has caused loss of consciousness. Immediate treatment consists of cardioversion followed by a suitable anti-arrythmic such as lidocaine I V or amiodarone and correction of any electrolyte imbalance. Myocardial infarction (MI) may be the cause of the ventricular tachycardia, hence treatment for MI would be indicated if required. Programmed stimulation may only be carried out when the patient is stable .

A 20-year-old woman complains of recurrent syncope. Each attack has occurred after attending an aerobics class. On examination, a systolic murmur is heard which worsens with the Valsalva manoeuvre and improves on squatting. What could be the diagnosis? Epilepsy Hypertrophic obstructive cardiomyopathy Atrial fibrillation Aortic stenosis Vasovagal attack A 20-year-old woman complains of recurrent syncope. Each attack has occurred after attending an aerobics class. On examination, a systolic murmur is heard which worsens with the Valsalva manoeuvre and improves on squatting. What could be the diagnosis?

of 179

(FACEBOOK) https/c 6/30/2011 4:56 PM

www.apnastudent.blogspot.com Epilepsy Hypertrophic obstructive cardiomyopathy Atrial fibrillation

Your answer

www.apnastudent.blogspot.com

1 History of ischaemic-type chest discomfort Aortic stenosis Vasovagal attack 2 Evolutionary ECG changes 3

Rise and fall in sethe rummost cardiac markerscomplaint of patients with hypertrophic obstructive cardiomyopathy. However, Dyspnoea is usually common angina or syncope may also occur. A left ventricular apical impulse, a prominent S4 gallop and a harsh systolic ejection murmur are typical findings in these cases. Valsalva manoeuvre decreases venous return to the heart, which results in a smaller ventricular size. This leads to an increase in the murmur. An echocardiogram is the diagnostic procedure of choice. Most patients with pure or predominant aortic stenosis have gradually increasing obstruction for years but do not become symptomatic until their sixth to eighth decades.

A 50-year-old male is brought to A&E with acute-onset central chest pain that started 1½ hours ago. His ECG taken on arrival showed ST elevations of 4 mm in leads I I, II I, aV F and V 4–V 6 with hyperacute T waves. His pulse was 60 per minute and his BP was 146/60. The SaO2 was 92%. He was given O2 by mask and two large intravenous cannulas were inserted. Which of the following blood tests would be most useful to wait for prior to commencing definitive treatment? Myoglobin Troponin I CK-MB LDH None

A 50-year-old male is brought to A&E with acute-onset central chest pain that started 1½ hours ago. His ECG taken on arrival showed ST elevations of 4 mm in leads I I, II I, aV F and V 4–V 6 with hyperacute T waves. His pulse was 60 per minute and his BP was 146/60. The SaO2 was 92%. He was given O2 by mask and two large intravenous cannulas were inserted. Which of the following blood tests would be most useful to wait for prior to commencing definitive treatment? Myoglobin Troponin I CK-MB LDH None

Your answer

The WHO criteria for the diagnosis of acute myocardial infarction (MI ) requires the presence of at least two of three elements:

In this patient, a diagnosis of MI can be made on the basis of the first two criteria. No further time should be wasted before reperfusing an ST elevation MI (STEMI), generally by percutaneous transluminal coronary intervention (PTCI). In the case of non-ST elevation MI (NSTEMI), the serum cardiac markers become elevated. The commonly used cardiac markers are creatinine kinase (CK), creatinine kinase-myocardial type (CK-MB) and troponin (Trop)-I /Trop-T. The earliest time at which the various cardiac markers rise are shown in the table. Mark er

In i t i a l r i s e

Peak l evel s

B ac k t o norm al

Myoglobin

1-4 h

6-7 h

24 h

Troponin I

3-12 h

24 h

5-10 days

12 h-2 days

5-14 days

Troponin T 3-12 h

(FACEBOOK) https/c of 179

6/30/2011 4:56 PM

www.apnastudent.blogspot.com CK-MB

3-12 h

24 h

48-72 h

www.apnastudent.blogspot.com LDH

10 h

24-48 h

10-14 days

A 70-year-old woman is admitted with chest pain and breathlessness of 12 hours duration. On examination, her heart rate is 170 beats/min, her BP is 125/72 mmHg. ECG shows atrial fibrillation. What is the next step in her management? Administration of propranolol Administration of verapamil Asynchronous cardioversion Administration of warfarin Immediate heparinisation A 70-year-old woman is admitted with chest pain and breathlessness of 12 hours duration. On examination, her heart rate is 170 beats/min, her BP is 125/72 mmHg. ECG shows atrial fibrillation. What is the next step in her management? Administration of propranolol Administration of verapamil Asynchronous cardioversion

Your answer

Administration of warfarin Immediate heparinisation

Correct answer

The appropriate treatment for atrial fibrillation (AF) is determined by the patient’s relative risk from the arrhythmia. High-risk patients include those with a heart rate greater than 150 beats/min, chest pain, an unstable condition or shock. These patients require urgent treatment. Immediate heparinisation, to reduce the risk of systemic embolisation, and attempted cardioversion with synchronised DC shock should be carried out first. Warfarin treatment is indicated in the elderly and those with heart disease, and where the duration of AF is longer than 48 hours prior to considering cardioversion. Young patients with lone atrial fibrillation in the absence of heart disease may not need anticoagulation.

A 75-year-old man with congestive cardiac failure presents with atrial fibrillation. He is haemodynamically stable with a ventricular rate of 72. He has a good functional state, although ECHO cardiography revealed a dilated left atrium and mild mitral regurgitation. Which drug option would be most beneficial for this patient? Aspirin Digoxin Frusemide Lidocaine Warfarin A 75-year-old man with congestive cardiac failure presents with atrial fibrillation. He is haemodynamically stable with a ventricular rate of 72. He has a good functional state, although ECHO cardiography revealed a dilated left atrium and mild mitral regurgitation. Which drug option would be most beneficial for this patient? Aspirin Digoxin Frusemide Lidocaine Warfarin

Your answer

(FACEBOOK) https/c of 179

6/30/2011 4:56 PM

www.apnastudent.blogspot.com

Patients with atrial fibrillation who are stable pose an intermediate risk. The initial treatment in this case is anticoagulation with warfarin. This is indicated in valvular heart disease and in the elderly. Digoxin is effective in controlling the heart rate at rest, and may improve his symptoms but would not affect stroke risk. Given that his ventricular rate is relatively low at 72, and that he is in a relatively good functional state, anticoagulation is indicated above rate control.

What is the commonest cause of restrictive cardiomyopathy in the UK? Pompe’s disease Amyloidosis Endocardial fibroelastosis Carnitine deficiency Acute coxsackievirus infection What is the commonest cause of restrictive cardiomyopathy in the UK? Pompe’s disease Amyloidosis

Your answer

Endocardial fibroelastosis Carnitine deficiency Acute coxsackievirus infection

Amyloidosis is the commonest cause of restrictive cardiomyopathy in the UK. Pompe’s disease is characterised by a rapid onset of muscle hypotonia, weakness, cardiomyopathy, glossomegaly and normal cerebral development. Death is common in the first year of life. A restrictive cardiomyopathy develops from endocardial fibroelastosis, which is typified by a collagen layer on the endocardium especially the left ventricle. Most infants with isolated disease present by age 3 months with heart failure. A metabolic cardiomyopathy develops with carnitine deficiency. A viral myocarditis is possible after coxsackievirus infection, but a cardiomyopathy is usually uncommon.

A 54-year-old man suddenly develops weakness of the left side of his face and arm and difficulty in speech. This episode lasts for 15 minutes. He has a history of hypertension, which is well controlled on a calcium channel blocking agent. His brother had had a severe disabling stroke at the age of 50. Cholesterol level is 5.8mmol/l. CT scan performed the same day shows the presence of 2 old lacunar strokes in the right middle cerebral artery territory. CT angiogram of the carotid system shows a 60% stenosis of the right internal carotid artery. Which of the following factors is the strongest predictor of his being at a high risk of early recurrent stroke? Positive family history History of hypertension Hyperlipidaemia Presence of moderate carotid stenosis Presence of previous strokes on CT scan A 54-year-old man suddenly develops weakness of the left side of his face and arm and difficulty in speech. This episode lasts for 15 minutes. He has a history of hypertension, which is well controlled on a calcium channel blocking agent. His brother had had a severe disabling stroke at the age of 50. Cholesterol level is 5.8mmol/l.

www.apnastudent.blogspot.com CT scan performed the same day shows the presence of 2 old lacunar strokes in the right middle cerebral artery territory. CT angiogram of the carotid system shows a 60% stenosis of the right internal carotid artery.

(FACEBOOK) https/c of 179

6/30/2011 4:56 PM

www.apnastudent.blogspot.com Which of the following factors is the strongest predictor of his being at a high risk of early recurrent stroke? Positive family history History of hypertension Hyperlipidaemia Presence of moderate carotid stenosis

Correct answer

Presence of previous strokes on CT scan

Your answer

This is a transient ischaemic attack (TI A). About 15–20% of patients with stroke have a preceding transient ischaemic attack. The issue of subsequent stroke prevention is therefore paramount when managing such a “warning event”. The urgency of treatment of minor stroke or transient ischaemic attack should depend on the early risk of major stroke. The risk of recurrent stroke during the first few days after a transient ischaemic attack or minor stroke is much higher than previously estimated. Recent studies have identified potential risk factors for those at highest risk of subsequent stroke: age>60 years; hypertension; duration of symptoms >60 minutes; certain clinical features (unilateral weakness, speech impairment); presence of diabetes mellitus. Brain imaging also seems to be of prognostic value: the presence of infarction on CT brain scans in patients with transient ischaemic attack or minor stroke is associated with an increased risk of stroke recurrence. More research is needed to determine the optimal medical management according to individual risk factors. There are several treatments that are likely to be effective in preventing stroke in the acute phase after a transient ischaemic attack or minor ischaemic stroke including aspirin, possibly in combination with clopidogrel and anticoagulation in patients with atrial fibrillation, and possibly statins. The subgroup of patients with large-artery atherosclerosis (usually carotid bifurcation stenosis) accounts for the largest proportion of early recurrent strokes. A recent population-based study of prognosis of patients with transient ischaemic attack and =50% symptomatic carotid-artery stenosis reported risks of stroke of about 20% during the 2 weeks before endarterectomy and other studies have highlighted the high risk of stroke if endarterectomy is delayed, and hence the rapid decrease in benefit from surgery with increasing time since event. For neurologically stable patients with transient ischaemic attack and minor stroke, benefit from endarterectomy is greatest if done within 2 weeks of the event. NB. The risk benefit ratio of treating symptomatic carotid stenosis (secondary stroke prevention) differs from that of treating asymptomatic stenosis as part of primary prevention, where stenosis must be severe in order to justify the risk of surgery. From t h e A m eri can H eart A ss ociat ion g u ideli n es (200 6) : For patients with recent TIA or ischemic stroke within the last 6 months and ipsilateral severe (70 to 99%) carotid artery stenosis, CEA is recommended by a surgeon with a perioperative morbidity and mortality of 15 mmHg in those with LV diastolic dysfunction. There will be a focal pressure gradient in a lobar or larger pulmonary artery, intravascular filling defect or narrowing in patients with major pulmonary artery occlusion by clot or tumour. I n congenital heart disease, there will be a step up in oxygen saturation in the right heart and a step down in the left heart.

A 17-year-old young man presents with palpitations. His physical examination is normal except for a systolic murmur in the second left intercostal space and prominent precordial motion with a late systolic impulse. Which of the following conditions is he likely to have? Aortic stenosis Atrial septal defect Hypertrophic cardiomyopathy Mitral valve prolapse Mixed aortic valve disease A 17-year-old young man presents with palpitations. His physical examination is normal except for a systolic murmur in the second left intercostal space and prominent precordial motion with a late systolic impulse. Which of the following conditions is he likely to have? Aortic stenosis Atrial septal defect Hypertrophic cardiomyopathy

Your answer

Mitral valve prolapse Mixed aortic valve disease

A mid to late systolic impulse in the precordial motion (triple ripple) is seen in patients with HOCM. A hyperdynamic impulse is seen in mild to moderate AR. Conditions A, B, D and E can produce the murmur but not this characteristic finding.

A 25-year-old woman is seen in outpatients and is found to have loud first heart sound, an early diastole sound followed by a mid-diastolic murmur. What is the likely diagnosis? Mitral stenosis with a fourth heart sound Mitral stenosis with atrial fibrillation Mitral stenosis with mobile leaflets Mitral stenosis with pulmonary hypertension Mitral valve prolapse A 25-year-old woman is seen in outpatients and is found to have loud first heart sound, an early diastole sound followed by

(FACEBOOK) https/c

of 179

6/30/2011 4:56 PM

www.apnastudent.blogspot.com a mid-diastolic murmur. What is the likely diagnosis? Mitral stenosis with a fourth heart sound

www.apnastudent.blogspot.com Mitral stenosis with atrial fibrillation Mitral stenosis with mobile leaflets

Correct answer

Mitral stenosis with pulmonary hypertension Mitral valve prolapse

Your answer

From the information given, the diagnosis is mitral stenosis with mobile leaflets. There is no information regarding atrial fibrillation or pulmonary hypertension. There is a loud first heart sound and when an opening snap is heard this indicates that the mitral valve is mobile. The snap occurs when the superior systolic bowing of the anterior mitral valve leaflet is rapidly reversed towards the left ventricle in early diastole, due to the high left atrial pressure. S4 is classically late diastolic or presystolic. It occurs when augmented atrial contraction causes presystolic ventricular distension so that the ventricle then contracts with greater force. Atrial myxoma can produce an early diastolic sound (tumour plop), which is due to the abrupt diastolic seating of the tumour within the right or left atrioventricular orifice.

Which of the following microanatomical structures within the heart interacts with conventional calcium-channel blockers? L type Calcium-channels Calcium-channel T type T tubules Titin Tropomyosin Which of the following microanatomical structures within the heart interacts with conventional calcium-channel blockers? L type Calcium-channels

Correct answer

Calcium-channel T type

Your answer

T tubules Titin Tropomyosin

The T tubules are a tubular network formed by the invagination of the sarcolemma of the myocyte. Sarcolemmal calcium channels are located on the T tubules; there are two main types of channels – T and L types. The T (transient) channels do not interact with conventional calcium-channel blockers. Calcium-channel blockers interact with the L-type calcium channels. Titin tethers the myosin molecule to the Z line, and its elasticity explains the stress–strain elastic relation of striated muscle. It is the largest protein molecule yet described. The thin actin filaments intertwine and are carried on a heavier tropomyosin molecule that functions as a backbone. At regular intervals along this structure is a group of three regulatory proteins called the ‘troponin complex’, which is composed of troponin C, troponin I and troponin M.

Which of the following antiarrhythmic agents works primarily by its action on SA and AV nodes? Amiodarone Atenolol Flecainide Sotalol Verapamil Which of the following antiarrhythmic agents works primarily by its action on SA and AV nodes? Amiodarone

(FACEBOOK) https/c of 179

6/30/2011 4:56 PM

www.apnastudent.blogspot.com Atenolol Flecainide Sotalol

Your answer

www.apnastudent.blogspot.com Verapamil

Correct answer

Calcium-channel blockers act mainly on the sinoatrial and atrioventricular nodes (direct membrane effect), as these structures are almost exclusively depolarised by the slow calcium channels. Flecainide binds to the sodium channel and decreases the speed of depolarisation (in other words, decreases conduction velocity). Atenolol decreases sympathetic tone. Amiodarone and sotalol increase the action-potential duration and therefore the refractory periods. They have little effect on conduction velocity.

Which of the following best describes the mechanism of action of flecainide as an antiarrhythmic agent? Slows the upstroke of the action potential Increases the action-potential duration Has a direct membrane effect Increases vagal tone Affects SA and AV nodes Which of the following best describes the mechanism of action of flecainide as an antiarrhythmic agent? Slows the upstroke of the action potential

Your answer

Increases the action-potential duration Has a direct membrane effect Increases vagal tone Affects SA and AV nodes

Flecainide, a class Ic agent slows the upstroke of the action potential and is its main mechanism of action. I t has minimal effects on action-potential duration. In other words, it causes a marked decrease in conductivity, with little effect on refractoriness. The antiarrhythmic group that mainly affects sinoatrial and atrioventricular nodes, and thus has a direct membrane effect, is the calcium-channel blockers. Class V agents (digitalis agents) affect SA and AV nodes by increasing vagal tone.

Which of the following antiarrhythmics have the highest risk of producing torsades de pointes? Flecainide Lidocaine Phenytoin Propafenone Sotalol Which of the following antiarrhythmics have the highest risk of producing torsades de pointes? Flecainide Lidocaine Phenytoin Propafenone Sotalol

Your answer

(FACEBOOK) https/c of 179

6/30/2011 4:56 PM

www.apnastudent.blogspot.com

Among these agents sotalol, which is a class II I agent, has the highest risk of producing torsades, which is a polymorphic, pause-dependent ventricular tachycardia causing syncope and sudden cardiac death. Lidocaine and phenytoin (which are class Ib agents) and flecainide and propafenone (class Ic) have an almost zero risk of producing TdP.

Urinary hesitancy as a sign of drug-induced toxicity is characteristic of which of the following antiarrhythmics? Amiodarone Sotalol Disopyramide Flecainide Verapamil Urinary hesitancy as a sign of drug-induced toxicity is characteristic of which of the following antiarrhythmics? Amiodarone Sotalol Disopyramide Flecainide

Correct answer

Your answer

Verapamil

Amiodarone causes hepatic effects, peripheral neuropathy, proximal myopathy, thyroid dysfunction, skin discoloration and pneumonitis, among others. Sotalol ( β-blockers) and flecainide have negative inotropy and CNS effects. Verapamil causes bradycardia.

A 25-year-old medical student noticed that he had a murmur when he tested his new stethoscope. On assessment in the cardiology clinic, he was found to have a harsh systolic murmur over his precordium, which did not change with inspiration. ECG showed features of biventricular hypertrophy. What is the most likely diagnosis? Aortic stenosis Hypertrophic cardiomyopathy Mitral regurgitation Tricuspid regurgitation Ventricular septal defect A 25-year-old medical student noticed that he had a murmur when he tested his new stethoscope. On assessment in the cardiology clinic, he was found to have a harsh systolic murmur over his precordium, which did not change with inspiration. ECG showed features of biventricular hypertrophy. What is the most likely diagnosis? Aortic stenosis Hypertrophic cardiomyopathy Mitral regurgitation Tricuspid regurgitation Ventricular septal defect

Your answer

www.apnastudent.blogspot.com

Right-sided murmurs increase with inspiration (eg tricuspid regurgitation, TR), whereas left-sided murmurs show no

(FACEBOOK) https/c of 179

6/30/2011 4:56 PM

www.apnastudent.blogspot.com change. The clue to diagnosis is in the ECG finding. Aortic stenosis and mitral regurgitation produce left ventricular hypertrophy, TR produces right ventricular hypertrophy and a ventricular septal defect produces biventricular hypertrophy.

In the cardiology outpatients’ department, a 50-year-old obese woman was noticed to have a systolic murmur loudest at the apex. I sometric exercise by handgrip intensified the murmur. What is the most likely cause of her murmur? Aortic stenosis Hypertrophic obstructive cardiomyopathy Mitral regurgitation Mitral valve prolapse Tricuspid regurgitation In the cardiology outpatients’ department, a 50-year-old obese woman was noticed to have a systolic murmur loudest at the apex. I sometric exercise by handgrip intensified the murmur. What is the most likely cause of her murmur? Aortic stenosis

Your answer

Hypertrophic obstructive cardiomyopathy Mitral regurgitation

Correct answer

Mitral valve prolapse Tricuspid regurgitation

The murmur of mitral regurgitation is intensified by isometric exercise and thus helps to differentiate it from other systolic murmurs. Sudden standing and amylnitrite decrease the murmur. The murmurs of aortic stenosis, mitral prolapse and HOCM are reduced by handgrip exercises.

A 30-year-old woman with a previous history of deep vein thrombosis is expecting her first child. During which phase of her pregnancy and puerperium does she have the greatest risk of venous thrombosis? First trimester Second trimester Third trimester During delivery First 6 weeks after delivery A 30-year-old woman with a previous history of deep vein thrombosis is expecting her first child. During which phase of her pregnancy and puerperium does she have the greatest risk of venous thrombosis? First trimester Second trimester Third trimester

Your answer

During delivery First 6 weeks after delivery

Correct answer

There is an increase in thromboembolic complications because of the hypercoagulability that exists postpartum. Anticoagulants may be necessary during pregnancy to prevent or control the following: venous thrombosis, pulmonary embolism, rheumatic mitral valve disease, prosthetic heart valves, peripartum cardiomyopathy, primary pulmonary hypertension and Eisenmenger’s syndrome.

www.apnastudent.blogspot.com A 25-year-old man presents to the emergency department with a 1-week history of fever and myalgia. He had travelled to Chile 8 weeks ago. On examination there are no positive findings, although the patient recollects that his right eyelid was

(FACEBOOK) https/c of 179

6/30/2011 4:56 PM

www.apnastudent.blogspot.com swollen for a few weeks after he left Chile. ECG reveals non-specific, T-wave changes in all leads. What is the most likely diagnosis? Echinococcosis Falciparum malaria Schistosomiasis Toxoplasmosis Trypanosomiasis A 25-year-old man presents to the emergency department with a 1-week history of fever and myalgia. He had travelled to Chile 8 weeks ago. On examination there are no positive findings, although the patient recollects that his right eyelid was swollen for a few weeks after he left Chile. ECG reveals non-specific, T-wave changes in all leads. What is the most likely diagnosis? Echinococcosis Falciparum malaria Schistosomiasis Toxoplasmosis Trypanosomiasis

Your answer

Trypanosoma cruzi. causes American trypanosomiasis or Chagas disease and is quite common in South America. The vectors are reduvid bugs. The trypanosomes are transmitted by scratching infected faeces of the bug into skin abrasions caused by the bug during blood sucking. I n acute trypanosomiasis, the patient presents with fever, myalgia, hepatosplenomegaly and myocarditis. Unilateral periorbital oedema and swelling of the eyelid can result from a bug bite around the eyes. This is called Romana’s sign. The other conditions listed can cause myocarditis, but the best choice is trypanosomiasis.

A 30-year-old woman presents with pleuritic chest pain and haemoptysis. Her blood pressure is stable at 130/80 mmHg. A ventilation/perfusion scan shows minor mismatch at the lung bases. There is no evidence of RV dysfunction, clinically and on echocardiography. In addition to oxygen, which of the following is the appropriate management for this patient? Heparin and consideration for surgery Heparin and paracetamol Heparin plus mechanical intervention Heparin plus thrombolytic therapy Supportive

A 30-year-old woman presents with pleuritic chest pain and haemoptysis. Her blood pressure is stable at 130/80 mmHg. A ventilation/perfusion scan shows minor mismatch at the lung bases. There is no evidence of RV dysfunction, clinically and on echocardiography. In addition to oxygen, which of the following is the appropriate management for this patient? Heparin and consideration for surgery Heparin and paracetamol Heparin plus mechanical intervention Heparin plus thrombolytic therapy Supportive

Your answer

www.apnastudent.blogspot.com This patient has potentially had up to two small to moderate pulmonary embolisms, probably associated with pulmonary infarction. The management in this case would be heparin and simple analgesics to control her chest pain. I f there were signs of a large/massive PE (hypotension, right ventricular dysfunction), the ideal management would include

(FACEBOOK) https/c of 179

6/30/2011 4:56 PM

www.apnastudent.blogspot.com thrombolytic therapy or mechanical intervention.

A 50-year-old man underwent coronary artery bypass grafting 2 days ago. A routine liver function test result now shows that both the direct and indirect bilirubin are elevated. All the other liver function tests are normal. Which of the following is the most likely cause? ‘Shock liver’ syndrome Anaesthetic-induced Haemolysis on bypass Narcotic-induced Right heart failure A 50-year-old man underwent coronary artery bypass grafting 2 days ago. A routine liver function test result now shows that both the direct and indirect bilirubin are elevated. All the other liver function tests are normal. Which of the following is the most likely cause? ‘Shock liver’ syndrome

Your answer

Anaesthetic-induced Haemolysis on bypass

Correct answer

Narcotic-induced Right heart failure

Isolated elevation of direct and indirect bilirubin, indicates haemolysis on the cardiopulmonary bypass and can be confirmed by increased plasma free-haemoglobin levels. There is no specific treatment. Markedly raised enzyme levels are seen in patients with the ‘shock liver’ syndrome, and the treatment is aimed at maximising cardiac output and oxygenation. Right heart failure is another cause of hyperbilirubinaemia in the immediate post-bypass period, and, in this case, the direct bilirubin and alkaline phosphatase are increased without enzyme elevation. Treatment is as for right heart failure.

In an asymptomatic patient, a permanent pacemaker is indicated in which of the following conditions? First-degree block at the AV node First-degree block in the distal conduction system with an HV interval < 100 ms Second-degree block at the AV node Second-degree block at the distal conduction system Right bundle branch block In an asymptomatic patient, a permanent pacemaker is indicated in which of the following conditions? First-degree block at the AV node First-degree block in the distal conduction system with an HV interval < 100 ms Second-degree block at the AV node Second-degree block at the distal conduction system

Your answer

Right bundle branch block

In an asymptomatic patient, a permanent pacemaker (PPM) is indicated in second- and third-degree heart block at the distal conduction system. I f the third-degree block at the AV node had been associated with symptoms, it would have been an indication for PPM. Also, a PPM is indicated for cases of first-degree AV block in the distal conduction system with an HV (Bundle of His to ventricular depolarisation) interval of greater than 100 ms associated with symptoms.

www.apnastudent.blogspot.com An 80-year-old man with a history of intermittent atrial fibrillation presents with syncope. ECG documents a type I I,

(FACEBOOK) https/c of 179

6/30/2011 4:56 PM

www.apnastudent.blogspot.com second-degree AV block. Which of the following types of pacemaker is best indicated for him? DDD DDDR VOO VVI VVIR An 80-year-old man with a history of intermittent atrial fibrillation presents with syncope. ECG documents a type I I, second-degree AV block. Which of the following types of pacemaker is best indicated for him? DDD DDDR

Your answer

VOO VVI VVIR

Correct answer

There is a suspicion here that this patient probably has sinus node disease with intermittent AF and periods of heart block. In a patient with long periods of AF, a single-chamber device (VVI) is indicated. The addition of rate modulation (VVIR) makes it even better. A DDD (atrial sensing) device is not indicated in this case, since it will inappropriately sense all the fibrillating P waves and can defeat the objective. A VOO (ventricular pacing only) device will simply pace the ventricle, and there is a risk that if the pacing falls in the relative refractory period of the ventricle, it can induce the R on T phenomenon, which can lead on to ventricular tachyarrhythmia (VT) and ventricular fibrillation (VF).

A 60-year-old man suffered an anterior myocardial infarction. He had all the risk factors for coronary artery disease. Which of the following non-pharmacological interventions will be most helpful in reducing his risk of a future ischaemic event? Alcohol reduction Diet control Regular exercise Stopping smoking Weight reduction A 60-year-old man suffered an anterior myocardial infarction. He had all the risk factors for coronary artery disease. Which of the following non-pharmacological interventions will be most helpful in reducing his risk of a future ischaemic event? Alcohol reduction Diet control Regular exercise Stopping smoking

Your answer

Weight reduction

All these interventions are effective but stopping smoking is the single most effective, non-pharmacological intervention that will help to reduce the risk of a future event. There are trials showing that smoking cessation is as effective as having a coronary artery bypass graft.

www.apnastudent.blogspot.com A 30-year-old man with known hypertrophic obstructive cardiomyopathy (HOCM) presents to casualty with an episode of

(FACEBOOK) https/c of 179

6/30/2011 4:56 PM

www.apnastudent.blogspot.com witnessed collapse: a passer-by provided initial resuscitation when he felt no pulse and the man was making no effort to breathe. On admission he is unwell with pulse rate of 160 bpm, blood pressure 70/40 mmHg and decreased conscious level. ECG confirms ventricular tachycardia. Sinus rhythm is restored with a DC shock. What would be the most appropriate strategy for the long term? Amiodarone Automatic implantable cardioverter defibrillator Dual-chamber pacemaker Sotalol Verapamil A 30-year-old man with known hypertrophic obstructive cardiomyopathy (HOCM) presents to casualty with an episode of witnessed collapse: a passer-by provided initial resuscitation when he felt no pulse and the man was making no effort to breathe. On admission he is unwell with pulse rate of 160 bpm, blood pressure 70/40 mmHg and decreased conscious level. ECG confirms ventricular tachycardia. Sinus rhythm is restored with a DC shock. What would be the most appropriate strategy for the long term? Amiodarone Automatic implantable cardioverter defibrillator

Your answer

Dual-chamber pacemaker Sotalol Verapamil

This man has survived an out-of-hospital cardiac arrest and therefore an automatic implantable cardioverter defibrillator (AICD) is warranted. Overall, patients with HOCM have an annual mortality rate of around 1%. I dentifying those at greatest risk of sudden cardiac death (SCD) is challenging. However, several factors have been identified that are associated with an increased risk: maximum wall thickness > 30 mm; non-sustained ventricular tachycardia on a 48-hour tape; a history of SCD in a relative under 45 years of age and a history of syncope; resting, left ventricular outflow-tract gradient > 30 mmHg; abnormal blood-pressure response to exercise. Although a single risk factor does not, on its own, have a particularly high positive-predictive accuracy, the presence of two or more risk factors does identify a much higher risk population. Dual-chamber pacing, b-blockers or verapamil may be used to reduce symptoms in patients with a left ventricular outflow-tract obstruction.

A 60-year-old woman presents with exertional dyspnoea on rushing upstairs or going to the local shops. She is in sinus rhythm at 80 bpm and her blood pressure is 160/80 mmHg, without evidence of fluid overload. She is already taking the maximum dose of an angiotensin-converting enzyme inhibitor and frusemide 40 mg once daily for long-standing hypertension. Echo confirms significantly impaired left ventricular systolic function (ejection fraction 35%). Her coronary arteries and renal function are normal. What additional medication should be considered for symptomatic and prognostic benefit? Amiodarone Amlodipine Bisoprolol Isosorbide mononitrate Losartan A 60-year-old woman presents with exertional dyspnoea on rushing upstairs or going to the local shops. She is in sinus rhythm at 80 bpm and her blood pressure is 160/80 mmHg, without evidence of fluid overload. She is already taking the maximum dose of an angiotensin-converting enzyme inhibitor and frusemide 40 mg once daily for long-standing hypertension. Echo confirms significantly impaired left ventricular systolic function (ejection fraction 35%). Her coronary arteries and renal function are normal. What additional medication should be considered for symptomatic and prognostic benefit?

www.apnastudent.blogspot.com Amiodarone Amlodipine Bisoprolol

Your answer

(FACEBOOK) https/c of 179

6/30/2011 4:56 PM

www.apnastudent.blogspot.com )

Isosorbide mononitrate Losartan

This woman has chronic heart failure (CHF) with compatible symptoms and objective evidence of left ventricular dysfunction at rest. There is overwhelming evidence for the long-term prognostic and symptomatic benefit of an angiotensin-converting enzyme inhibitor (ACE inhibitor) in patients with CHF. Recent trials support the prognostic and symptomatic benefit of certain b-blockers in CHF patients in addition to ACE inhibitors. I n the UK, bisoprolol and carvedilol are the two licensed agents for use in this condition, trials of both bisoprolol and carvedilol suggest around a reduction of 30% in mortality, particularly in patients with severe cardiac failure. I sosorbide mononitrate and amlodipine are safe to use in patients with CHF either for symptomatic treatment of angina or associated hypertension. There is no evidence that they influence outcome.

A 70-year-old man, previously fit and well, is referred to out-patients with exertional chest pain that comes on at around 0.8 km (0.5 mile) on the flat. Examination reveals him to be in sinus rhythm at 80 bpm, blood pressure 100/70 mmHg, a diminished carotid upstroke and a loud ejection systolic murmur over the aortic area. Echo confirms left ventricular hypertrophy, preserved systolic function with peak aortic valve gradients of 80 mmHg. Subsequent coronary angiography reveals significant disease in the left anterior descending artery. What is the optimal treatment strategy? Angioplasty and stenting to the left anterior descending artery Aortic valve replacement and bypass graft Aspirin and review in 3 months’ time with a repeat echo Aspirin and a b-blocker, and review in 3 months’ time with a repeat echo Percutaneous aortic valvotomy

A 70-year-old man, previously fit and well, is referred to out-patients with exertional chest pain that comes on at around 0.8 km (0.5 mile) on the flat. Examination reveals him to be in sinus rhythm at 80 bpm, blood pressure 100/70 mmHg, a diminished carotid upstroke and a loud ejection systolic murmur over the aortic area. Echo confirms left ventricular hypertrophy, preserved systolic function with peak aortic valve gradients of 80 mmHg. Subsequent coronary angiography reveals significant disease in the left anterior descending artery. What is the optimal treatment strategy? Angioplasty and stenting to the left anterior descending artery Aortic valve replacement and bypass graft

Correct answer

Aspirin and review in 3 months’ time with a repeat echo Aspirin and a b-blocker, and review in 3 months’ time with a repeat echo Percutaneous aortic valvotomy

Your answer

This patient has severe (aortic valve gradient > 70 mmHg), symptomatic aortic stenosis and as such valve replacement is indicated, unless precluded by co-morbidity. Coronary angiography is performed to assess the need for concomitant coronary artery bypass grafting. With acquired aortic stenosis, patients most commonly present in their sixth decade with symptoms of angina, syncope or heart failure. The development of symptoms is associated with a poor outcome if left untreated. The average life expectancy from the onset of symptoms to death is 2 years in patients with heart failure, 3 years in those with syncope and 5 years in those with angina.

A 52-year-old woman, with a prior history of rheumatic fever, presents with shortness of breath on strenuous exertion while working as a landscape gardener. She is in permanent atrial fibrillation and is on long-term warfarin and digoxin (125 µg once daily). Clinical examination reveals her to be in atrial fibrillation at a rate of around 150 bpm. Echo demonstrates preserved left ventricular function, a heavily calcified mitral valve with moderate mitral stenosis (mitral valve area 1.5 cm and moderate mitral regurgitation. Her left atrium is dilated. What is the most appropriate initial treatment option?

2

www.apnastudent.blogspot.com Amiodarone Atenolol

(FACEBOOK) https/c of 179

6/30/2011 4:56 PM

www.apnastudent.blogspot.com )

DC shock Mitral valve replacement Percutaneous mitral valvotomy

A 52-year-old woman, with a prior history of rheumatic fever, presents with shortness of breath on strenuous exertion while working as a landscape gardener. She is in permanent atrial fibrillation and is on long-term warfarin and digoxin (125 µg once daily). Clinical examination reveals her to be in atrial fibrillation at a rate of around 150 bpm. Echo demonstrates preserved left ventricular function, a heavily calcified mitral valve with moderate mitral stenosis (mitral valve area 1.5 cm and moderate mitral regurgitation. Her left atrium is dilated. What is the most appropriate initial treatment option? Amiodarone Atenolol

Correct answer

DC shock Mitral valve replacement

Your answer

Percutaneous mitral valvotomy

This woman has moderate, mixed mitral valve disease and therefore surgery is not currently indicated. It may however be required later if features of left heart failure progress. Even if the mitral stenosis were to be severe the presence of heavy calcification of the valve and concomitant mitral regurgitation would preclude percutaneous valvotomy. She is in permanent atrial fibrillation and as such, by definition, sinus rhythm cannot be restored (as opposed to persistent or paroxysmal). Better rate control is required. Digoxin on its own may not control catecholamine-driven tachycardia (eg during exertion). Amiodarone, while effective, would not be the first choice for this young patient working outdoors because of its side-effect profile.

2

View more...

Comments

Copyright ©2017 KUPDF Inc.
SUPPORT KUPDF